112
Universidad La Salle. Facultad Mexicana de Medicina. Curso de Extensión Universitaria para la Preparación del Examen Nacional para Aspirantes a Residencias Médicas. Examen Módulo V – Subespecialidades. Examen del Módulo V. 24 de Agosto del 2011. 1.- Una niña de 9 años presenta astenia, adinamia, decaimiento y pérdida de 3 kilogramos de su peso habitual. Desde hace 15 días ha padecido fiebre de 39°C, disfagia y epistaxis. Al explorarla se encuentra pálida, con adenomegalia cervical y axilar; se palpa hepatoesplenomegalia y se observan quimosis en las piernas. El diagnostico más probable es: a) Mononucleosis infecciosa. b) Linfoma de hodgkin. c) Tuberculosis extrapulmonar. d) Leucemia aguda. La EH se presenta habitualmente con adenopatías cervicales o supraclaviculares, indoloras, más consistentes que las inflamatorias y de lento crecimiento. Al menos, dos tercios de casos asocian adenopatías mediastínicas. Síntomas sistémicos: Síntomas sistémicos no específicos pueden ser fatiga, anorexia y ligera pérdida de peso. Síntomas específicos (fiebre inexplicada, pérdida de peso de 10% en los 6 meses previos y sudor nocturno abundante) tienen significado pronóstico. Algunos pacientes tienen prurito. Exámenes de laboratorio En el hemograma, se observa leucocitosis con neutrofilia, linfopenia, eosinofilia y monocitosis. La anemia puede indicar la presencia de enfermedad avanzada. Se ha observado anemia hemolítica asociada a EH, con test de Coombs positivo, reticulocitosis e hiperplasia normoblástica de médula ósea. La asociación de púrpura trombopénica idiopática (PTI) también ha sido referida y la respuesta al tratamiento de la PTI va a depender del estadio en que se encuentre la EH. La elevación de VSG, cupremia y ferritinemia reflejan la activación del sistema mononuclear fagocítico, que puede servir como test inespecífico para control de seguimiento de la EH. Las anomalías del

Examen Subespecialidades

Embed Size (px)

DESCRIPTION

Evaluacion La salle ENARM de subespecialidades.

Citation preview

Page 1: Examen Subespecialidades

Universidad La Salle.

Facultad Mexicana de Medicina. Curso de Extensión Universitaria para la Preparación del Examen Nacional para

Aspirantes a Residencias Médicas. Examen Módulo V – Subespecialidades.

Examen del Módulo V. 24 de Agosto del 2011.

1.- Una niña de 9 años presenta astenia, adinamia, decaimiento y pérdida de 3 kilogramos de su peso habitual. Desde hace 15 días ha padecido fiebre de 39°C, disfagia y epistaxis. Al explorarla se encuentra pálida, con adenomegalia cervical y axilar; se palpa hepatoesplenomegalia y se observan quimosis en las piernas. El diagnostico más probable es: a) Mononucleosis infecciosa. b) Linfoma de hodgkin. c) Tuberculosis extrapulmonar. d) Leucemia aguda. La EH se presenta habitualmente con adenopatías cervicales o supraclaviculares, indoloras, más consistentes que las inflamatorias y de lento crecimiento. Al menos, dos tercios de casos asocian adenopatías mediastínicas. Síntomas sistémicos: Síntomas sistémicos no específicos pueden ser fatiga, anorexia y ligera pérdida de peso. Síntomas específicos (fiebre inexplicada, pérdida de peso de 10% en los 6 meses previos y sudor nocturno abundante) tienen significado pronóstico. Algunos pacientes tienen prurito. Exámenes de laboratorio En el hemograma, se observa leucocitosis con neutrofilia, linfopenia, eosinofilia y monocitosis. La anemia puede indicar la presencia de enfermedad avanzada. Se ha observado anemia hemolítica asociada a EH, con test de Coombs positivo, reticulocitosis e hiperplasia normoblástica de médula ósea. La asociación de púrpura trombopénica idiopática (PTI) también ha sido referida y la respuesta al tratamiento de la PTI va a depender del estadio en que se encuentre la EH. La elevación de VSG, cupremia y ferritinemia reflejan la activación del sistema mononuclear fagocítico, que puede servir como test inespecífico para control de seguimiento de la EH. Las anomalías del

Page 2: Examen Subespecialidades

sistema inmune en el momento del diagnóstico de EH pueden persistir durante y después del tratamiento. 1.*** Fernández-Teijeiro A. Enfermedad de Hodgkin. En: Madero López L y Muñoz Villa A editores. Hematología y Oncología pediátricas. Madrid: Ergon; 1997. p. 467-82. El autor hace un recuerdo histórico de EH, señalando aspectos epidemiológicos, etiológicos e histológicos, resaltando la presentación clínica y el diagnóstico diferencial, así como los estudios de diagnóstico. Describe con amplitud aspectos terapéuticos de radioterapia, quimioterapia y de efectos secundarios del tratamiento. 2.* Ferrís Tortajada J, García Castell J, Berbel Tornero O, Clar Gimeno S. Factores de riesgo para los linfomas no hodgkinianos. An Esp Pediatr 2001; 55: 230-8. Exposición de factores de riesgo para LNH, revisados de la literatura médica, con el objetivo, según los autores, de divulgar entre los pediatras dichos factores de riesgo. 3.* Ferrís Tortajada J, García Castell J, López Andreu JA, Berbel Tornero O, Clar Gimeno S. Factores de riesgo para los linfomas de Hodgkin. An Esp Pediatra 2001; 55: 239-43. Exposición de factores de riesgo para EH, revisados de la literatura médica, con el objetivo, según los autores, de divulgar entre los pediatras dichos factores de riesgo. 2.- Paciente con infección por VIH, diagnosticado hace 2 años, asintomático. Acude a consultarlo porque no ha podido ir con el infectólogo que lo ha revisado en los últimos 6 meses y quien había solicitado como control una carga viral que se reportó con 100,000 copias, y una cuenta de CD4 de 33 células. Usted le recomienda. a) Inicio de tratamiento con AZT y 3TC. b) Ir con su infectólogo. c) Repetir el estudio. d) Iniciar TMP/SMZ y claritromicina.

Page 3: Examen Subespecialidades

El conteo de CD4 tan bajo pone al paciente en riesgo de desarrollar un evento por infección oportunista potencialmente mortal en el siguiente año por arriba del 80%. Es indispensable, en tanto recibe atención por el especialista, iniciar tratamiento profiláctico contra Pneumocystis jiroveci y Mycobacterium avium-intracellulare. Kasper DL, Braunwald E, Fauci AS, Hauser SL, Longo DL, Jameson JL. Harrison´s Principles of Internal Medicine. McGraw Hill. 16 Ed. 1076 p. 3.- Se trata de masculino de 40 años de edad con leucemia mieloblástica aguda recibió su tercer ciclo de quimioterapia. Una semana después inicia con fiebre y un súbito empeoramiento del estado general. Al realizar una exploración física se presenta una lesión nodular equimótica y dolorosa, con centro ulcerado y rodeada de eritema y edema, en el MPI. Presentó menos de 100 leucocitos/mm3, hemoglobina 7 gr/dL y 30.000 plaquetas/mm3. ¿El germen que es aislado con mayor frecuencia en un caso como éste es? a) Aeromonas hydrophila. b) Klebsiella pneumoniae. c) Staphyloccocus aureus. d) Pseudomonas aeruginosa.

Pseudomonas aeruginosa (o Pseudomonas pyocyanea) es una bacteria Gram-negativa, aeróbica, con motilidad unipolar.1 Es un patógeno oportunista en humanos y también en plantas.2

Como otras Pseudomonas, P. aeruginosa secreta una variedad de pigmentos como piocianina (azul verdoso), fluoresceína (amarillo verdoso fluorescente) y piorubina (rojo pardo). King, Ward, & Raney desarrollaron "Pseudomonas Agar P" (también conocido como "medio King A") para mejorar la producción de piocianina y piorubina; y "Pseudomonas Agar F" (también conocido como "medio King B") para la fluoresceína.3

Este patógeno oportunista de individuos immunocomprometidos, P. aeruginosa infecta el tracto pulmonar, el urinario, tejidos, heridas, y también causa otras infecciones de sangre Pseudomonas puede causar neumonías a grupos, necesitando a veces ayuda mecánica para superar dichas neumonías, siendo uno de los más comunes agentes aislados en muchos estudios. La piocianina es un factor de virulencia de la bacteria y se ha conocido que puede hasta causar muerte en C. elegans por estrés oxidativo. Sin embargo, la investigación indica que el ácido salicílico puede inhibir la producción de piocianina. Uno en diez hospitales se infectan con pseudomonas. La fibrosis quística está también predispuesta a la infección con

Page 4: Examen Subespecialidades

P. aeruginosa de los pulmones. P. aeruginosa es el causante de dermatitis, causada por disminución del control de la calidad del agua de bebida. El más común causante de altas fiebres en infecciones es P. aeruginosa. También ha estado involucrado en foliculitis de tinas de agua caliente, en especial aquellas sin un control higiénico continuo.8

1. Ryan KJ; Ray CG (editors) (2004). Sherris Medical Microbiology, 4th ed. edición, McGraw Hill ISBN 0-8385-8529-9..

2. Iglewski BH (1996). Pseudomonas. In: Baron's Medical Microbiology (Barron S et al, eds.), 4th ed. edición, Univ of Texas Medical Branch (via NCBI Bookshelf) ISBN 0-9631172-1-1..

3. King EO, Ward MK, Raney DE (1954). «Two simple media for the demonstration of pyocyanin and fluorescin.» J Lab Clin Med. Vol. 44. n.º 2. pp. 301-7. PMID 13184240.

4. Todar's Online Textbook of Bacteriology 5. Fine et al, JAMA 1996: 275: 134 6. Diekema DJ et al. Clin Infect Dis 1999;29:595 7. Prithiviraj B, Bais H, Weir T, Suresh B, Najarro E, Dayakar B, Schweizer H, Vivanco

J (2005). «Down regulation of virulence factors of Pseudomonas aeruginosa by salicylic acid attenuates its virulence on Arabidopsis thaliana and Caenorhabditis elegans.» Infect Immun. Vol. 73. n.º 9. pp. 5319-28. PMID 16113247.

8. MedlinePlus - Enciclopedia Médica: Foliculitis de la tina.

4.- Una mujer de 36 años es valorada porque presenta dolor de garganta y adenopatía cervical. Su temperatura es de 37°C., pulso de 90/min. y presión arterial de 110/70 mmHg. El ganglio cervical anterior derecho mide 2.5 x 3 cm. No hay adenopatía supraclavicular, axilar, epitroclear o inguinal palpable. La exploración abdominal no es relevante. En los siguientes seis meses la paciente es revalorada por infecciones recurrentes de las vías respiratorias superiores. Durante este período hubo alguna regresión de ganglios cervicales, pero al parecer el tamaño aumenta y disminuye en respuesta a los antibióticos. Por su adenopatía persistente, la enferma es enviada con un cirujano y se realiza una biopsia del ganglio. ¿Cuál de los siguientes describe la histopatología correspondiente al diagnóstico de la enferma? a) Linfoma folicular de células pequeñas hendidas. b) Linfoma de células grandes difusas. c) Linfoma inmunoblástico. d) Ganglio linfático normal.

Page 5: Examen Subespecialidades

Allen R. M. MMS Medicina Interna. 5ª. Edición. National Medical Series. Mc. Graw Hill. 2006. (capítulo 4 XVI C 2, D 2). El linfoma folicular de células hendidas pequeñas, llamado antes linfoma linfocítico nodular mal diferenciado, es una de las variedades más comunes de linfomas no hodgkinianos indolentes. A menudo los pacientes presentan enfermedad en etapa III o IV. Aun sin tratamiento, los individuos pueden tener adenopatía que aparece y desaparece, pero con el tiempo la enfermedad progresa y se requiere quimioterapia. Un pequeño porcentaje de sujetos presenta un linfoma más agresivo, casi siempre de variedades de células grandes o inmunoblásticos. 5.- En la hiponatremia intensa (inferiores 120 mEq/L se debe de tratar con soluciones salinas hipertónicas mediante el calculo del déficit, reponiendo de la siguiente manera: a) Pasar el déficit de sodio en las primeras 6 hr. b) Pasar el déficit de sodio en las primeras 12 hr. c) Pasar el déficit de sodio, la mitad en las primeras 12-24 hr y se revalora. d) Pasar el déficit de sodio en las primeras 3 hr y después se revalora. Farreas. Medicina Interna. Cap. 222, Decimoquinta edición 2004, Pp. 1844-1847. La hiponatremia con volumen extracelular disminuido se trata con la administración de soluciones salinas isotónicas (al 0.9%). La cantidad necesaria de miliequivalentes de sodio se calcula de acuerdo a la siguiente formula: Na (mEq)= Na x agua total corporal, es decir Na (mEq)= (140- Na actual) x (0.6 x peso en Kg) Donde: Agua total corporal= 60% del peso = 0.6 x peso en kg En la practica, suele administrase la mitad de esa cantidad en el transcurso de las primeras 12- 24 h y luego se efectúan reevaluaciones correctoras. 6.- Su frecuencia es de 1/10,000, bajo peso al nacimiento, LPH (Labio paladar hendido) microftalmia, holoprosencefalia, aplasia cutis vertex, riñones poliquísticos, malrotación del colon, y en niñas puede existir útero bicorne, son manifestaciones de:

a) Trisomía 21 b) Trisomía 13 c) Trisomía 18 d) Trisomía 11

Page 6: Examen Subespecialidades

La trisomía 13 representa la tercera aneuploidía autosómica viable más frecuente en la especie humana, sólo superada en frecuencia por la trisomía 21 y trisomía 181. Aunque su etiología es aún desconocida, estudios citogenéticos en individuos afectados han podido determinar que aproximadamente un 75% de los casos corresponden a no-disyunción meiótica, un 20% a translocaciones y un 5% a mosaicismo debido a nodisyunción post-cigótica. La expresión fenotípica de la trisomía 13 es característica y consiste en anomalías faciales, esqueléticas y del sistema nervioso central, siendo también frecuentes las malformaciones estructurales del aparato cardiovascular, genitourinario y gastrointestinal (Tabla I).1 La mayoría de los defectos estructurales asociados a trisomía 13 pueden ser identificados prenatalmente por ultrasonografía.2-6 Esto permite ofrecer estudio citogenético prenatal en casos seleccionados, establecer el diagnóstico definitivo y manejar el embarazo afectado en forma racional, ya que esta anomalía se asocia a una mortalidad perinatal cercana al 100%.

Page 7: Examen Subespecialidades
Page 8: Examen Subespecialidades

Bibliografía: 1. Jones KL. Smith’s Recognizable Patterns of Human Malformation. 5th Edition. WB Saunders, Philadelphia. 1997, pp 18-23. 2. Benacerraf BR, Miller WA, Frigoletto FD. Sonographic detection of fetuses with trisomies 13 and 18: accuracy and limitations. Am J Obstet Gynecol 1988; 158:404-9. 3. Dicke JM, Crane JP. Sonographic recognition of major malformations and aberrant fetal growth in trisomic fetuses. J Ultrasound Med 1991,10:433-8. 4. Nicolaides KH, Snijders RJ, Gosden CM, et al. Ultrasonographically detectable markers of fetal chromosomal abnormalities. Lancet 1992;340:704-7. 5. Lehman CD, Nyberg DA, Thomas CW, et al. Trisomy 13 syndrome: prenatal US findings in a review of 33 cases. Radiology 1995;194:217-22. 6. Sepúlveda W. Marcadores sutiles de anomalías cromosómicas en el feto: detección ultrasonográfica, significado clínico y manejo. Rev Chil Obstet Ginecol 1996;61:395-403. 7. Sepúlveda W, Be C, Bravo M. Anomalías cromosómicas en el primer trimestre del embarazo: Correlación entre hallazgos ultrasonográficos y cariograma por biopsia de vellosidades coriales. Rev Chil Obstet Ginecol 1997;62:268-74. Lectura recomendada: Revista Chilena de Ultrasonografía. Vol 2/Nº1/1999 Rendimiento de la translucencia nucal en 120 biopsias de vellosidades coriales Trisomía 13: Diagnóstico citogenético prenatal y hallazgos ultrasonográficos Trisomía 13: Diagnóstico citogenético prenatal y hallazgos ultrasonográficos. 7.- Se trata de paciente femenino de 28 que acude por presentar en la muñeca una lesión pápuloescamosa, pruriginosa, violácea y poligonal, con una red de líneas grisáceas (estrías de Wickham). Esta lesión es compatible con: a) Pitiriasis rosada. b) Liquen plano. c) Tiña versicolor. e) Melanoma.

Liquen plano

El liquen plano es una enfermedad dermatológica consistente en una erupción papular, prurítica, inflamatoria, de curso crónico que afecta a la piel y mucosas. Los cambios clínicos y histológicos son característicos de esta enfermedad, pudiéndose presentar con diversas formas clínicas. El liquen plano afecta a 1,5-5% de los pacientes de consultas externas de un servicio de dermatología. Es de causa desconocida, existen datos que muestran que el liquen plano representa una reacción autoinmune, mediada por los linfocitos T y dirigida contra los queratinocitos basales que expresan autoantígenos en su superficie que han sido

Page 9: Examen Subespecialidades

modificados por diversas causas como son infecciones virales (hepatitis C), medicaciones, alergenos de contacto, neoplasias internas. Esta reacción de los linfocitos T

contra los autoantígenos modificados por los agentes externos se vería seguida de la reacción cruzada frente autoantígenos previamente ignorados dando lugar a una perpetuación de la reacción autoinmune mediada por los linfocitos T que caracteriza el liquen plano. La lesión clínica elemental cutánea es la existencia de una pápula poligonal, brillante, con una depresión central cubierta de una escama blanquecina que deja entrever unas líneas blanquecinas que se conocen como estrías de Wickham. Las lesiones se distribuyen de forma simétrica afectando a las caras extensoras de las extremidades, especialmente en las muñecas, región lumbar, cara lateral de cuello y área genital. La mayor parte de los pacientes refieren un inicio brusco de su enfermedad, que se desarrolla en un período corto de unas semanas. El prurito suele ser muy intenso. El liquen plano, al igual que la psoriasis, desarrolla el fenómeno de Koebner, siendo frecuente el desarrollo de nuevas lesiones en áreas de rascado o traumatismo. Las lesiones de liquen plano curan espontáneamente o con tratamiento dejando una hiperpigmentación residual que es más intensa en fototipos de piel IV, V y VI. Existen muchas variantes clínicas de liquen plano que varían con relación a la zona de aparición y su morfología.

• Bligard CA, Millikan LE. Acute exanthems in children. Postgrad Med 1986;79:150-67.

• Bodemer C, de Prost Y. Unilateral laterothoracic exanthem in children: a new disease? J Am Acad Dermatol 1992;27:693-96.

• Drago F, Rampini P, Rampini E, Rebora A. Atypical exanthems: morphology and laboratory investigations may lead to an aetiological diagnosis in about 70% of cases. Br J Dermatol 2002;147:255-60.

• Frieden IJ. Childhood exanthems. Curr Opin Pediatr 1995;7:411-14. Gelmetti C, Caputo R. Asymmetric periflexural exanthem of childhood: who are you? J Eur Acad Dermatol Venereol 2001;15:293-94.

8.- Masculino de 25 años con un gran melanoma maligno en la espalda. No hay linfadenopatía aparente por lo que está en estadio 1. ¿Cuál es el factor pronóstico más importante en su caso? a) Grosor del tumor. b) Diámetro del tumor. c) Localización del tumor. d) El género del paciente.

Page 10: Examen Subespecialidades

Factores de riesgo:

Exposición solar. Número de nevos (5-6 =3 veces + riesgo) Nevos congénitos. Historia previa de melanoma. Historia familiar. Transplante e inmunosupresión. Xeroderma pigmentoso.

Ante la presencia de una lesión pigmentada presente desde el nacimiento o de nueva aparición, deberán de tenerse en cuenta una serie de parámetros clínicos (Regla: ABCDE)

A = ASIMETRIA B = BORDE C = COLOR D = DIAMETRO.

FACTORES PRONOSTICOS. MM E.C. I Y II.

GROSOR. Fx Px más importante de sobrevida. Correlaciona con riesgo de recurrencia local. Riesgo de metástasis en transito. Determina márgenes de resección quirúrgica.

1. Fitzpatrick TB, Jonson RA, Wolf K, Suurmond D. Color Atlas and Synopsis of Clinical Dermatology. Ed. McGraw-Hill-Interamericana, 2001. 2. Malvehy J, Puig S, Martí R, Castel T, Mascaró JM. Melanoma maligno cutáneo (I): epidemiología, patogenia y diagnóstico. Med Cután Iber Lat Am 1998;6:283-99. 3. Prorok PC, Hankey B, Bundy B. Concepts and problems in the evaluation of screening programmes. J Chronic Dis 1981;34:159-71. 4. Spratt JS. Epidemiology of screening of cancer. Cancer 1982;6:1-58. 5. Kopf AW, Rigel DS, Friedman RJ. The rising incidence and mortality rates of malignant melanoma. J Dermatol Surg Oncol 1982;8:760-1. 6. Schreiber MM, Bozzo PD, Moon TE. Malignant melanoma in southern Arizona: increasing incidence ans sunlight as an etiologic factor. Arch Dermatol 1981;117:6-11. 7. Makie RM. Secondary prevention of malignant melanoma. Pigment cell 1999;11:22-30. 8. Setlow RB, Woodhead AD. Temporal changes in the incidence of malignant melanoma: explanation from action espectra. Mut Res 1994;307:365-74.

Page 11: Examen Subespecialidades

Lectura: Melanoma maligno: Consideraciones sobre su diagnóstico precoz, prevención y tipos clínicos. Dermatólogo Correspondencia: Xavier Jeremías Torruella Hospital del Mar Servicio de Dermatología. Facultad de Medicina Universidad Autónoma de Barcelona Passeig Marítim, 25. 08003 Barcelona 9.-Masculino de 35 años con datos clínicos de importancia para diagnóstico de íleo mecánico simple, los datos radiográficos de esta patología son: a) Retención de contenido en colon, sin niveles hidroaereos. b) Liquido y aire libres en cavidad peritoneal. c) Niveles hidroaéreos a diferentes alturas, con dilatación de asas. e) Asas fijas y lisas, con niveles largos. ILEO MECANICO A/ Extraluminal Adherencias postquirúrgicas (Causa más frecuente 35-40%) Hernias externas (inguinales, crurales, umbilicales, laparotómicas, etc.) Hernias internas Torsiones Vólvulos Invaginaciones Efecto masa extraluminal (tumoración, masa inflamatoria ó abceso) B/ Parietal Neoplasias Alteraciones congénitas (atresias, estenosis, duplicaciones, etc.) Procesos inflamatorios (Crohn, postradiación, etc.) C/ Intraluminal Ileo biliar Bezoar Parasitosis Cuerpo extraño Impactación fecal

Page 12: Examen Subespecialidades

Tumoraciones

Íleo mecánico

Los signos radiológicos de una obstrucción intestinal simple son:

• Asas distendidas proximales por retención de líquido y gas. • Niveles hidroaéreos. • Reducción o ausencia de gas y materia fecal en colon. Ante el hallazgo de gas en

colon se descarta la existencia de una obstrucción de intestino delgado.

BIBLIOGRAFÍA: 1) Di Lorenzo C. Pseudo-obstrucction. Current approaches. Gastroenterology 1999;116:980-987. 2) Stanghellini V, Cammilleri M, Malagelada JR.Chronic idiopatic intestinal pseudobstruction: clinical and intestinal manometric findings. Gut 1987; 28: 5-12. 3) Scolapio JS, Ukleja A, Bouras EP et al. Nutritional management of chronic intestinal pseudo-obstruction. J Clin Gastroenterol 1991; 28:306-312. 4) Malagelada JR, Distrutti E. Managementof gastrointestinal motility disorders. A practical guide to drug selection and appropiate ancillary measures. Drugs 1996;52:494-506. 10.- Cuando se observa una imagen histológica de acantosis con elongación de las crestas interpapilares-que incluso se fusionan entre sí, hiperparaqueratosis y acúmulos epidérmicos de leucocitos polimorfonucleares, estamos hablando de: a) Icitiosis. b) Dermatitis (eccema). c) Psoriasis. d) Epidermólisis.

Page 13: Examen Subespecialidades

Psoriasis La psoriasis es una enfermedad cutánea caracterizada por un curso crónico, que cursa a brotes y tiene hallazgos clínicos variables. Las manifestaciones cutáneas de esta enfermedad son tan características (Tabla 1) que el diagnóstico suele realizarse con facilidad. En la actualidad se contempla la psoriasis como una enfermedad de base inmunológica, mediada por los linfocitos T, que asocia inflamación dérmica y secundariamente hiperplasia epidérmica. Es probablemente la enfermedad dermatológica mediada inmunológicamente más prevalente. Tabla 1 Características clínicas de la psoriasis: 1. Placa eritematosa. 2. Escamas blanquecinas. 3. Buena delimitación. 4. Signo de Auspitz. 5. Fenómeno de Koebner.

Psoriasis. Hiperparaqueratosis con acúmulos intracórneos de neutrófilos (microabscesos de Munro). HE, x 400. Cambios histológicos de la psoriasis:

• Infiltración de células mononucleares • Hiperplasia epidermica • Acantosis epidermica regular, con hipogranulosis, hiperqueratosis y paraqueratosis • Atrofia epidermica suprapapilar

Page 14: Examen Subespecialidades

• Infiltracióninflamatoria • Pustula espongiforme de Kogog • Microabscesos de Munro • Infiltrado de linfocitos CD4 y CD8 en dermis • Cambios vasculares

• Proliferación de vasos capilares dilatados ocupando las papilas. Bibliografía:

• Arnold HL, James WD, Odom RB. Andrews : tratado de dermatología. 1993. 4ª ed. Barcelona: Masson; 1993

• Ashcroft DM, Po AL, Williams HC, Griffiths CE. Systematic review of comparative efficacy and tolerability of Calcipotriolin treating chronic plaque psoriasis. BMJ 2000; 320:963-967. [Medline][Texto completo]

• Chalmers RJG, O'Sullivan T, Owen CM, Griffiths CEM. Intervenciones para la psoriasis guttata (Revisión Cochrane). En: La Cochrane Library Plus, Número 2, 2002. Oxford: Update Software

• Ferrándiz Foraster C. Dermatosis eritematoescamosas (I). Psoriasis. Eritrodermias. En: Ferrándiz C, ed. Dermatología Clínica.2ª ed. Madrid: Harcourt; 2001. p. 165-175.

• Freedberg I, Eisen A, Katz SI, Wolff K, Fitzpatrick TB, Goldsmith LA et al, eds. Dermatology in general medicine. 5th ed. New York: McGraw-Hill; 1999

• Riffiths C E, Clark C M, Chalmers R J, Li Wan Po A, Williams H C. A systematic review of treatments of severe psoriasis. Health Technology Assessment, 2000;4(40):1-125.

• Naldi L, Rzany B. Chronic plaque psoriasis. Clin Evid 2002; 7: 1488-1507 11.- Si el estudio de triple marcador en suero materno indica una elevación de AFP (alfa feto-proteína) respecto a la edad gestacional, usted debe sospechar como primera opción: a) Trisomía 21 (Down) b) Trastorno monogénico c) Defecto de cierre de tubo neural o de pared abdominal d) Labio hendido

Alfa Fetoproteina (AFP) es una glicoproteína de PM 70.000 D, cuya secuencia de aminoácidos presenta una homología del 40 % con la albúmina. Se sintetiza inicialmente en el saco vitelino y posteriormente en el hígado fetal. Su concentración en sangre fetal aumenta hasta alcanzar un máximo de 300 mg/100ml a las 10-13 semanas de embarazo. A partir de este momento, disminuye progresivamente a menos de 100 mg/100ml a término y sigue disminuyendo hasta los 5 mg/100ml a los 2 años de edad, permaneciendo en estos niveles hasta la vida adulta.

Page 15: Examen Subespecialidades

La AFP puede migrar de la circulación fetal a la materna a través de dos mecanismos:

1. Difusión transplacentaria 2. Difusión transamniótica desde la orina fetal.

La concentración en sangre materna es cinco veces inferior a la fetal y va aumentando durante el segundo trimestre debido al incremento de la permeabilidad placentaria. Podemos encontrar por lo tanto aumento de la AFP sérica materna por defectos de la barrera feto-amniótica ( Defectos abiertos del tubo neural , defectos abiertos de la pared abdominal , higroma quístico , teratoma fetal , amputaciones fetales , muerte fetal , síndrome nefrótico fetal ) o por defectos de la barrera placentaria ( Hemorragia feto-materna , tumores o infartos placentarios, placentas hipertróficas o quísticas ). Cualquier mínimo compromiso de la integridad placentaria produce repercusiones importantes en los niveles maternos de AFP, debido al gran gradiente de concentración existente entre el suero materno y el fetal.

La causa menos frecuente de elevación de la AFP en suero materno, es la secundaria a la disminución de la eliminación renal fetal por patología obstructiva o displásica.

Los valores de AFP en suero materno están disminuidos en pacientes portadoras de fetos con Síndrome de Down. Existen diversas etiologías acerca de la causa de esta disminución siendo la más aceptada la propuesta por Cukcle y colaboradores quienes abogan por una disminución de la síntesis hepática fetal.

La concentración de AFP en el suero de fetos con trisomía 13, 18 y monosomía X, es más baja que la de fetos cromosómicamente normales. Los valores bajos de AFP en suero materno no son exclusivos por tanto del Síndrome de Down.

TAMIZAJE BIOQUIMICO DE CROMOSOMOPATIAS. 12.- Una paciente de 25 años con una historia de contactos sexuales con distintas parejas en los últimos meses, consulta por fiebre elevada, cefalea, adenopatías inguinales y ulcerosas dolorosas en vulva. ¿Cuál es el diagnóstico más adecuado? a) Vaginitis candidiásica. b) Enfermedad pélvica inflamatoria por Chlamydia. c) Infección por Citomegalovirus. d) Primoinfección por Herpes simple tipo 2.

Page 16: Examen Subespecialidades

Los virus Herpes simplex (VHS) tipos 1 (VHS-1) y 2 (VHS-2), dos alfaherpesvirus estrechamente relacionados, causan el herpes genital, un problema de salud pública global, de rango epidémico, cuya verdadera dimensión empezamos a advertir. En nuestro entorno, el herpes es la tercera enfermedad de transmisión sexual (ETS) por orden de frecuencia y la primera causa de úlceras anogenitales. Se estima que en el mundo hay alrededor de 90 millones de individuos que sufren la enfermedad crónica recurrente sintomática y, lo que resulta más inquietante, el número de infectados que desconocen serlo podría exceder esa cifra. En España, al igual que en otros países de la Europa mediterránea, la prevalencia de la infección por VHS-2 en la población general se sitúa en torno al 5-10% y alcanza el 25-40% cuando se considera únicamente a individuos atendidos en clínicas de ETS.

El VHS-2 causa la mayoría de los herpes genitales, si bien, durante los últimos años, se ha producido un incremento del número de casos debidos al VHS-1, particularmente en occidente, en adolescentes y adultos jóvenes; este hecho es atribuible al aumento de los adolescentes susceptibles al VHS-1 y a un cambio sustancial en el modelo de conducta sexual de la población.

La primoinfección genital por ambos VHS es, frecuentemente, asintomática o "subclínica". En su forma característica, el herpes anogenital sintomático cursa con lesiones vesiculosas y ulcerativas dolorosas situadas, generalmente, en los labios y el introito vaginales, glande y prepucio, adenopatías regionales, y, especialmente en la mujer, con disuria; no es infrecuente que se acompañe de síntomas generales tales como fiebre y cefalea. La infección previa por el VHS-1 suele aminorar la sintomatología asociada a la primoinfección por el VHS-2. Las reactivaciones son asintomáticas en más del 80% de los casos, independientemente del VHS causal, y los episodios de recurrencia sintomáticos son, en general, menos llamativos clínicamente que los vinculados a las primoinfecciones.

Los VHS-1 y VHS-2 difieren en cuanto a su capacidad de causar lesiones anogenitales recurrentes; el VHS-2 las genera con mayor frecuencia que el VHS-1, particularmente durante el año siguiente a la primoinfección, en una proporción de 5:1. Es de gran interés epidemiológico, por otra parte, el hecho de que mientras que el VHS-1 se excreta infrecuentemente en ausencia de lesiones, el VHS-2 lo hace habitualmente, de forma intermitente (entre 4-75% de los días del año) e impredecible (no hay manera de prever quiénes lo harán, en qué medida y con qué frecuencia), de modo que la mayoría de infecciones de novo por el VHS-2 son consecuencia de contactos sexuales con individuos que excretan el virus de forma "subclínica" o asintomática; no hay duda de que la población sexualmente activa que ignora estar infectada por el VHS-2 es el reservorio más trascendente del virus y la fuente de diseminación de la infección.

Page 17: Examen Subespecialidades

BIBLIOGRAFÍA:

• Ashley R, Wald A. Genital herpes: review of the epidemic and potential use of type-specific serology. Clin Microbiol Rev 1999; 12:1-8.

• Aurelian L. Herpes simplex viruses. En: Specter S, Hodinka RL, Young SA (eds). Clinical Virology Manual, 3ª ed. Washington DC: ASM Press, 2000; pp 384-409.

• Guerry SL, Bauer HM, Klausner JD, et al. Recommendations for the selective use of herpes simplex virus type 2 serological tests. Clin Infect Dis 2005; 40:38-45.

• Kimberlin DW, Rouse DJ. Genital herpes. N Engl J Med 2004; 350:1970-1977. • Patel R. Progress in meeting today`s demands in genital herpes: an overview of

current management. J Infect Dis 2002; 186 (Supl 1):S47-S56. • Ramaswamy M, Smith M, Geretti AM. Detection and typing of herpes simplex DNA

in genital swabs by real-time polymerase chain reaction. J Virol Methods 2005; 126:203-206.

• Slomka MJ. Current diagnostic techniques in genital herpes: their role in controlling the epidemic. Clin Lab 2000; 11: 591-607.

• Stevenson J, Hymas W, Hillyard D. Effect of sequence polymorphisms on performance of two real-time PCR assays for detection of herpes simplex virus. J Clin Microbiol 2005; 43:2391-2398.

13.- Masculino de 18 años que desde la infancia padece anemia, esplenomegalia e ictericia. El frotis de sangre periférica muestra eritrocitos pequeños con palidez central y una fragilidad osmótica muy incrementada. Ud. sospecha de: a) Esferocitosis hereditaria

b) Anemia perniciosa c) Talasemia mayor e) Leucemia linfoblástica

La esferocitosis hereditaria (EH) es una enfermedad caracterizada por anemia hemolítica de severidad variable, con presencia de esferocitos en sangre periférica y una respuesta clínica favorable a la esplenectomía. Con el desarrollo de nuevas técnicas se encontraron las primeras alteraciones bioquímicas de las proteínas de la membrana eritrocitaria, y

Page 18: Examen Subespecialidades

posteriormente, se han podido precisar las alteraciones moleculares mediante las técnicas del ADN recombinante. La EH es una enfermedad muy heterogénea que se produce por un defecto intrínseco del glóbulo rojo, y existen otras alteraciones secundarias a esta afección. La prueba más utilizada para el diagnóstico de la EH es la fragilidad osmótica del glóbulo rojo. Se ha demostrado que esta enfermedad es producida por defectos de las proteínas que intervienen en las interacciones verticales entre el esqueleto de la membrana y la bicapa lipídica. El tratamiento de elección en la EH es la esplenectomía, ya que es el más efectivo en el control de la anemia, aunque la sobrevida de los glóbulos rojos permanece acortada y los esferocitos no desaparecen. Este proceder se indica en pacientes con anemia hemolítica severa o en individuos moderadamente asintomáticos pero que presentan litiasis vesicular.

Formas Clínicas

La EH es una enfermedad muy heterogénea desde el punto de vista clínico. Se puede observar desde el portador asintomático hasta pacientes que presentan una anemia hemolítica crónica con grandes requerimientos transfusionales.24,25.

Dependiendo de la severidad del cuadro clínico, de las cifras de hemoglobina, los niveles de bilirrubina y el conteo de reticulocitos, esta enfermedad se clasifica en 4 formas: portador asintomático, EH ligera, EH típica y EH severa.26,27.

Portador asintomático. En algunas familias se ha señalado un patrón de herencia autosómico recesivo. En estos casos, los padres de un paciente afectado no presentan ninguna alteración. En ocasiones la afectación es muy leve, como ligero incremento de las cifras de reticulocitos, escasos esferocitos en periferia o fragilidad osmótica incubada alterada y puede no ser detectada por los exámenes de rutina. Debe tenerse en cuenta también que pueden ocurrir nuevas mutaciones dentro de una familia aparentando un patrón de herencia autosómico recesivo, por lo que siempre es importante un estudio minucioso de todos los miembros de la familia.3,4.

EH ligera. Comprende entre el 20 y 30 % de todos los pacientes con EH autosómica dominante, los que pueden presentar una hemólisis ligera compensada.3,27 Los individuos son frecuentemente asintomáticos y algunos casos son difíciles de diagnosticar, ya que la anemia y la esplenomegalia son muy ligeras y en ocasiones pueden estar ausentes.28 Muchos de estos pacientes se diagnostican durante estudios familiares o cuando en la etapa adulta aparece el íctero y la esplenomegalia. Episodios hemolíticos pueden presentarse en el curso de algunos procesos infecciosos como mononucleosis, parvovirus o citomegalovirus, así como durante el embarazo, por esfuerzos físicos intensos o por sangramientos.27-30.

EH típica. Entre el 50 y 60 % de los pacientes con EH autosómica dominante tienen esta forma clínica. Presentan una hemólisis compensada incompleta y una anemia de ligera a

Page 19: Examen Subespecialidades

moderada. El íctero es común en niños, aunque se puede ver también en los adultos y está asociado con infecciones virales ligeras, debido a la estimulación reticuloendotelial y a un aumento de la hemólisis. Los requerimientos transfusionales son esporádicos. La esplenomegalia está presente en el 50 % de los niños y en el 75 % de los adultos.2,3,31,32.

EH severa. Estos pacientes (5-10 %) evolucionan con una hemólisis severa y presentan frecuentes requerimientos transfusionales. La mayoría de estos casos tienen una forma autosómica recesiva de la enfermedad. Pueden presentar crisis aplásticas, retardo del

crecimiento y de la maduración sexual. La esplenectomía es el tratamiento de elección en esta forma clínica.33,34 Generalmente la enfermedad debuta al nacimiento con ictericia y hemólisis y se requiere, en muchas ocasiones, de exanguinotransfusión.30,32.

REFERENCIAS BIBLIOGRÁFICAS: 1. Berga L, Vives-Corrons JLl, Feliú E, Woessner S, Rozman C. Hemorreología. Bases teóricas y aplicaciones clínicas. Barcelona: Salvat, 1983. 2. Delaunay J. Genetic disorders in the red cell membrane. Crit Rev Oncol Hematol 1995;19:79-110. 3. Iolascon A, Miraglia del Giudice E, Perrotta S, Alloisio N, Morle L, Delaunay J. Hereditary spherocytosis: from clinical to molecular defects. Haematologica 1998; 83:240-57. 4. Scriver SR, Beaudet AL, Sly WS, Valle D. The metabolic and molecular bases of inherited disease. Philadelphia: McGraw-Hill Company, CD-ROM, 1997. 5. Yu J, Steck TL. Isolation and characterization of band 3, the predominant polypeptide of the human erythrocyte membrane. J Biol Chem 1975;250:9170-6. 6. Kay MMB, Folwers N, Goodman J. Alterations in membrane protein band 3 associated with accelerated erythrocyte ageing. Proc Nall Acad Sci USA 1989;86:5834-8. 7. Marsh WL. Molecular defects associated with McLeod blood group phenotype. En. Salmon E, ed. Blood groups and other red cells surface markers in health and disease 1982:17-82. 8. Tse WT, Lux SE. Red blood cell membrane disorders. Br J Haematol 1999;104:2-13. 9. Speicher DW, DeSilva TM, Speicher KD, Ursitti JA, Hembach P, Weglarz L. Location of human red cell spectrin tetramer binding site and detection of a related “closed” hairpin loop dimer using proteolytic footprinting. J Biol Chem 1993;268:4227-31. 10. Jordan C, Puschel B, Koob GR. Identification of a binding motif for ankirin on the a subunit of Na+ K+ ATPase. J Biol Chem 1995;270:29971-4.

Page 20: Examen Subespecialidades

14.- Masculino de 34 que acude por fatiga y palidez. Su Hto es del 32% y su Hb de 10.3g/100mL. El frotis de sangre periférica revela macrocitosis. Su nivel de vitamina B12 sérica es 90 pg/mL (normal 170-940). Su diagnóstico es: a) Leucemia linfocítica b) Anemia microcítica c) Púrpura tromboctiopénica d) Anemia megaloblástica La anemia megaloblástica es un tipo de anemia en donde existe una disminución de la síntesis del ADN con detención de la maduración que compromete las tres líneas celulares de la médula ósea (glóbulos rojos, glóbulos blancos y plaquetas). Las causas que la producen son numerosas, pero aproximadamente el 95% de los casos es consecuencia de una deficiencia de vitamina B 12 y/o de ácido fólico. Las manifestaciones clínicas y hematológicas son similares en ambos casos, pero las manifestaciones neurológicas se presentan sólo en los casos de deficiencia de vitamina B12. El tratamiento está ligado a la causa que la produce. Hemograma: En la serie roja: - Macrocitosis con un Volumen corpuscular medio > 100 fL, y generalmente la hemoglobina corpuscular media está elevada (12) - Ovalositos, dacriositos, cuerpos de inclusión: en el frotis de sangre periférica (Howell- Jolly y anillos de Cabot) - Incremento del índice de anisocitosis. En la serie blanca: - Leucopenia en casos severos - Un signo precoz de megaloblastosis carencial es la hipersegmentación de los Neutrófilos. - Recuento de plaquetas: no suele alterarse pero puede haber trombocitopenia severa (4, 12). El aspirado de médula ósea: es hipercelular, con aumento relativo de los precursores eritroides, núcleos de aspecto inmaduro y citoplasma hemoglobinizado. Los precursores granulocíticos de gran tamaño (metamielocitos gigantes) así como megacariocitos(12). Bioquímica: La determinación de cobalamina en suero debe ser menor de 200 pg/ml, valor que debe demostrarse en al menos dos determinaciones separadas (13) (Normal: 150- 900 pg/ml). BIBLIOGRAFIA:

Page 21: Examen Subespecialidades

1. De Paz R, Hernández- Navarro F. Manejo, prevención y control de la anemia perniciosa. Nutr Hosp. Scielo Chile [en línea] 2005 [fecha de acceso 12 de Febrero de 2007]; 20 (6): URL 2. Forrellat Barrios M, Gómis Hernández I, du Défaix Gómez HG. Vitamina B 12: metabolismo y aspectos clínicos de su deficiencia. Rev Cubana Hematol Inmunol Hemoter. Biblioteca virtual de salud [en línea] 1999 [fecha de acceso 12 de Febrero de 2007]; 15 (3): URL. 3. Brees MH, Berlow RW. El Manual Merk de diagnóstico y tratamiento. 10ª ed. España: Elsevier España SA, 1999; 869- 873. 4. Ruiz Franco O. Síndromes anémicos (Primera parte) Anemias carenciales. Rev Diagnóstico. Fundación instituto Hipólito Unanue [en línea] Abril- Junio de 2005 [fecha de acceso 13 de Febrero de 2007]; 44 (2); URL. 15.- Acude masculino de 6 años de edad al departamento de Urgencias debido a que presenta un cuadro de dificultad respiratoria. Sus padres refieren que por la mañana estaba bien y al regresar de jugar en el jardín presenta dificultad respiratoria. A la exploración física se encuentra letárgico, taquipneico, diaforético; y existen sibilancias y estertores en ambos campos pulmonares, con abundantes secreciones, y pupilas reactivas de 1mm. Signos vitales: FC 70 por minuto, FR 50 por min, TA de 90/ palpable, Temp. 37.5° C, peso de 25 Kg. ¿Qué clase de exposición tóxica presenta este paciente? a) Hidrocarburos b) Organofosforados c) Anticolinergicos d) Salicilatos

Los efectos agudos aparecen inmediatamente o poco tiempo después de la exposición: En la absorción dérmica los signos y síntomas se manifiestan a las 2-3 horas después de la exposición. Sin embargo es posible observar los efectos dentro de l/2-1 hora, dependiendo de las circunstancias de la intoxicación. Algunos compuestos pueden retenerse en el tejido graso resultando en un retraso de los síntomas hasta por 24 horas.

Los síntomas de intoxicación se dividen en:

Page 22: Examen Subespecialidades

1. Efectos muscarínicos que incluyen diaforesis, pupilas puntiformes, sialorrea, epífora, broncoconstríccíón y aumento de la secreción de las glándulas bronquiales, espasmo abdominal con vómito y diarrea, bradicardia.

2. Efectos nicotínicos que incluyen taquicardia, fasciculaciones musculares o contracciones espasmódicas de los músculos finos, en los casos más severos del diafragma y músculos respiratorios.

3. Las manifestaciones en el sistema nervioso central incluyen cefalea, fatiga, vértigo, ansiedad, confusión, convulsiones, depresión del centro respiratorio, coma.

El inicio y la intensidad de los síntomas varía dependiendo del compuesto (inhibidor directo/indirecto), la ruta y el nivel de exposición.

Los primeros síntomas son usualmente nausea, cefalea, fatiga, vértigo, visión borrosa -muchas veces descrita "como un velo sobre los ojos"- y constricción pupilar. Dependiendo de la severidad de la intoxicación estos síntomas pueden agravarse agregándose vómito, dolor abdominal, diarrea, diaforesis y sialorrea. El empeoramiento progresivo se caracteriza por espasmos musculares que usualmente inician en la lengua y los párpados, progresando a crisis convulsivas y finalmente parálisis. También hay broncoconstricción e hipersecreción bronquial y en la etapa final se observa parálisis, convulsiones, depresión respiratoria y coma. En la intoxicación fatal por organofosforados la causa inmediata de muerte es generalmente la asfixia como resultado de depresión respiratoria.

Para la mayoría de los plaguicidas organofosforados, la exposición dermal y la subsecuente absorción a través de la piel intacta representa la ruta de entrada más importante en caso de exposición ocupacional.

La ruta oral es importante en caso de ingestión accidental o con fines suicidas. La ingestión ocupacional accidental ocurre como resultado de un deficiente adíestramento en el trabajo y la falta de higiene personal.

La ruta respiratoria es generalmente la menos importante. La inhalación de organofosforados depende de la volatilidad del compuesto, del tipo de formulación y de la técnica de aplicación.

Page 23: Examen Subespecialidades

16.- Mujer de 25 años presenta epistaxis, sangrado transvaginal, equimosis y petequias. Refiere que ha presentado hemorragias en las encías. Desde hace tres días ha tenido artralgias leves. A la exploración física, se encuentran huellas de sangrado en la nariz y no hay visceromegalias. El diagnóstico m+as probable es: a) Enfermedad de von willebrand. b) Púrpura trombocitopénica autoinmune. c) Púrpura trombocitopénica trombótica. d) Púrpura trombocitopénica idiopática. La EVW es el trastorno hemorrágico hereditario más común en los seres humanos. La característica central de todos los tipos de EVW es la presencia de cantidades reducidas de FVW o de formas anormales del FVW en el torrente sanguíneo. Papel del factor von Willebrand en la hemostasia: • Facilita la adhesión plaquetaria a la pared del vaso sanguíneo lesionado. • Participa en la agregación plaquetaria. • Es la proteína portadora del factor VIII.

Page 24: Examen Subespecialidades
Page 25: Examen Subespecialidades

ENFERMEDAD DE VON WILLEBRAND: INTRODUCCIÓN PARA MÉDICOS DE ATENCIÓN PRIMARIA David Lillicrap Department of Pathology and Molecular Medicine Queen’s University, Kingston, Canadá Paula James Department of Medicine Queen’s University, Kingston, Canadá TRATAMIENTO DE LA HEMOFILIA MARZO DE 2009 • NO 47 17. - The following conditon is commonly seen in the magnesium-amonium-phosphate (struvite) stones: a) Recurrent P. mirabilis infection b) Double J stent placement c) Resorptive hypercalciuria d) Renal azotemia

Esta bacteria de colonias redondeadas tiene la habilidad de producir grandes niveles de ureasa. La ureasa hidroliza urea a amoníaco, (NH3) y eso hace a la orina más alcalina. Y al subir la alcalinidad puede liderar la formación de cristales de estruvita, carbonato de calcio, y/o apatita. Esta bacteria puede encontrarse en cálculos, y esas bacterias escondidas allí, pueden reiniciar una infección post tratamientos antibióticos. Al desarrollarse los cálculos, después de un tiempo pueden seguir creciendo más y causar obstrucción dando fallas renales.

Page 26: Examen Subespecialidades

Proteus también puede producir infecciones de heridas, septicemia y neumonías, sobre todo en pacientes hospitalizados.

• Esipov, Sergei E. and J. A. Shapiro (1998). «Kinetic model of Proteus mirabilis swarm colony development». Journal of Mathematical Biology 36 (3). doi 10.1007/s002850050100.

• Frénod, Emmanuel (2006). «Existence result for a model of Proteus mirabilis swarm». Differential and integral equations 19 (6): pp. 697-720. http://arxiv.org/abs/math.FA/0702761.

18.- En la enfermedad por reflujo los siguientes pacientes tiene una indicación más clara de tratamiento quirúrgico: a) Paciente de 68 años de edad con esofagitis erosiva y antecedentes de infarto de miocardio hace 2 años con insuficiencia cardiaca. b) Paciente de 79 años con hernia hiatal de mediano tamaño y pirosis frecuente que responde bien a 20 mg/día de omeprazol en 6 semanas. c) Femenino de de 45 años con molestias epigástricas tipo flatulencia y pirosis inter-mitente que responde sólo ligeramente al tratamiento con omeprazol 20 mg.

Page 27: Examen Subespecialidades

d) Masculino de 56 años con pirosis diaria diurna y nocturna de más de 10 años de evolución que durante el tratamiento con 40 mg de esomeprazol se presenta asintomático y a la supresión o disminución de dosis reinciden de forma inmediata los mismos síntomas. ¿QUÉ ENFERMOS DEBEN SER ENVIADOS A TRATAMIENTO QUIRÚRGICO? 1. Aquellos con sintomatología persistente, que afecta su calidad de vida y que dependen de la ingesta constante de medicamentos. 2. Falta de apego al tratamiento médico.1 3. Pacientes que para el control de sus síntomas requieren de un aumento progresivo de las dosis habituales de omeprazol (20 mg), rabeprazol (20 mg), lansoprazol (30 mg), pantoprazol (40 mg) o esomeprazol (40 mg). 4. Esofagitis erosiva que requiere tratamiento a largo plazo y con riesgo de complicaciones. 5. El enfermo con síntomas recurrentes y que decida de manera expresa la cirugía, siempre que no exista una contraindicación. La cirugía antirreflujo es una alternativa de tratamiento a largo plazo en enfermos bien seleccionados. Es indispensable establecer con absoluta certeza el diagnóstico de ERGE con o sin hernia hiatal. El fundamento del tratamiento quirúrgico es aumentar la eficacia de la barrera antirreflujo Las complicaciones de la enfermedad como: estenosis, úlcera o esófago de Barrett no son indicaciones absolutas de cirugía. Los predictores de buena respuesta al tratamiento quirúrgico son: 1. Respuesta adecuada a tratamiento farmacológico. 2. Pacientes con pHmetría anormal. En los enfermos con pHmetría normal y ERGE se han informado malos resultados en comparación con los que son llevados a funduplicatura con pHmetría anormal. 3. Personas menores de 50 años.2,3 La efectividad de la cirugía se ve reducida en sujetos con reflujo refractario a tratamiento médico, enfermedad documentada de manera deficiente. Enfermedad por reflujo no erosiva (ERNE)

Page 28: Examen Subespecialidades

En los casos de ERNE no se ha determinado bien el papel de la cirugía antirreflujo. Aunque podría considerarse en caso de algunos síntomas extraesofágicos a nivel laríngeo o pulmonar, o en el caso de un defecto anatómico grande como hernia hiatal.4 El grupo consideró que la cirugía antirreflujo laparoscópica representa un avance terapéutico significativo, sin embargo, los resultados dependen sustancialmente de la experiencia del cirujano. No hay en la actualidad estudios que comparen el costo-efectividad de la Terapa éutica médica con la quirúrgica. Por lo tanto, la elección dependerá de la preferencia del paciente en el caso de contar con un grupo quirúrgico entrenado. (Nivel de Evidencia 2, Grado de Recomendación B). 1. Armstrong D, Marshall J, Chiba N, Enns R, Fallone C, et al. Canadian Consensus Conference on the management of gastroesophageal reflux disease in adults. Update 2004. Can J Gastroenterol 2005; 19: 15-35. 2. De Vault K, Castell D. Update Guidelines for the diagnosis and treatment of Gastroesophageal reflux disease. AJG 2005: 100: 190-200. 3. Catarci M, Gentileschi P, Papi C, Carrara A, Marrese R, et al. Evidence- Based Appraisal of Antireflux Fundoplication. Ann Surg 2004; 239: 325- 37. 4. Westcot C, Hopkins B, Bach K, Postman G, Belafisky P, et al. Fundoplicationfor laringo-faringeal reflux disease. Am J Coll Surg 2004; 199:23-30. 5. Brooks DC, Rattner D. Patient satisfaction following laparoscopic andopen antireflux surgery. Arch Surg 1995; 130: 289-94. 6. Catarci M, Gentileschi P, Papi C, Carrara A, Marrese R, et al. Evidence- Based Appraisal of Antireflux Fundoplication. Ann Surg 2004; 239: 325-37. 7. Zoring C, Strate U, Fibbe C. Nissen vs Toupet laparoscopic fundoplication. Surg End 2002; 16: 758-66. 8. Freston JW, Triadafilopoulos G. Review article: approaches to the longterm management of adults with GERD-proton pump inhibitor therapy, laparoscopic fundoplication or endoscopic therapy. Aliment Pharmacol Ther 2004; 19(Suppl. 1): 35-42.

Page 29: Examen Subespecialidades

19.-Femenino de 34 años la cual labora en lavandería, hace varios años presenta enrojecimiento de los pliegues proximales de las uñas de varios dedos de las manos, que ocasionalmente le supuran. ¿Cuál es el diagnóstico más probable? a) Dermatitis de contacto. b) Paroniquia candidiásica crónica. c) Liquen plano. d) Dermatoficia. Infección por Candida del lecho ungueal que se presenta con mayor frecuencia como una onicolisis asociada con paroniquia, aunque también se observa la destrucción completa del lecho ungueal y la erosión de la zona distal y lateral de las uñas de los dedos, sin distrofia ungueal total. La perionixis candidiásica se trata de la inflamación con eritema, edema, dolor y salida ocasional de pus blanquecino escaso y espeso del reborde ungueal de uno o varios dedos de la mano. Con frecuencia, se acompaña de onicopatía (discoloración, onicolisis, distrofia total, etc.).

Predisponentes: Todas las labores manuales que generen humedad. (muy frecuente en amas de casas, trabajadores de restaurantes, lavanderas, etc.)

BIBLIOGRAFÍA: 1. Crespo Erchiga V. Protocolo diagnóstico de contaminantes. En "Micología para dermatólogos" Ed. Janssen, Madrid, 1994, pp:49-70. 2. Crespo V, De Luís B, Delgado V, Crespo A y Vera. Espectro etiológico de las onicomicosis en nuestro medio. CO7. II Congreso Nacional de Micología. Santiago de Compostela. Junio, 1994.

Page 30: Examen Subespecialidades

3. Crespo Erchiga V, Delgado Florencio V y Martínez García S. Micología dermatológica. Ed. M.R.A. Barcelona, 2006. 4. Daniel III CR. The Diagnosis of Nail Fungal Infection. Editorial Arch Dermatol 1991;127:1566-1567. 5. Delgado Florencio V. Protocolo de identificación de dermatofitos. En "Micología para dermatólogos". Ed. Janssen, Madrid, 1994, pp:27-41. 6. Delgado Florencio V. Estrategia en el diagnóstico y tratamiento de las micosis superficiales. Ed. Aula Médica, Madrid, 1994. 7. Delgado V, Abad Romero-Balmas J, Armijo Moreno M y Dulanto F. Scopulariopsis brevicaulis como agente de onicomicosis. Actas Deermo-Sif. 1976; 9-10:693-700. 8. English MP. Nails and fungi. Br J Dermatol 1976; 94:697-701. 9. Fevilhade de Chauvin M. Onicomicosis. Dermatología práctica. 1994; 9:1-2. 20.-Masculino de 12 años que inicia con dolor intraescrotal intenso de aparición brusca al encontarse realizando actividad física, se tiene la sospecha de una torsión de testículo, ¿Cuál es, entre las siguientes, la prueba diagnóstica de elección? a) Ecografía-doppler. b) Ecografía. c) Radiografía escrotal. d) Palpación bimanual. Ecografía eco-doppler color: Es la prueba diagnóstica de elección al permitir la visualización de la vascularización testicular pudiendo distinguir si el flujo circulatorio es normal, ausente o aumentado. En la torsión lo normal es que esté reducido o ausente. Tiene una especificidad del 100% y una sensibilidad del 80%, aunque tiene sus limitaciones en las subtorsiones. Gammagrafía isotópica testicular con TC 99: Es también un método sensible y específico, pero del que no siempre se puede disponer en urgencias. Se verá un área de captación disminuida o ausente, rodeado de un área de captación aumentada por la hiperemia reactiva. Bibliografía: 1) Sesions AE et al: Testicular torsión: direction, cadem, duration and desinformation. J. Urol 2003, 169: 663-665. 2) Cummings JM el al: Adult testicular torsión. J.Urol. 202, 167: 2109, 167: 2109-2110 3) Bedos F et al: Manual de Urología. Ed Masson SA. Barcelona 1996. 297-335.

Page 31: Examen Subespecialidades

21.- De las dermatosis reaccionales, el padecimiento que se caracteriza afección de piel y mucosas con ampollas en blanco de tiro, signo de Nikolski positivo y en la biopsia de piel se observa ampolla con necrosis epidérmica es: a) Psoriasis b) Síndrome de Stevens-Johnson c) Eritema Multiforme d) Pénfigo Vulgar La necrólisis epidérmica tóxica (NET) y el síndrome de Stevens-Johnson (SSJ) constituyen un espectro de la misma enfermedad, compartiendo aspectos etiológicos, patogenéticos, histológicos y terapéuticos. Casi todos (si no todos) los casos son inducidos por fármacos. NET y SSJ pueden distinguirse clínicamente del eritema multiforme, que debe considerarse una enfermedad distinta. La patogenia de la NET y del SSJ es poco conocida, pero se acepta que intervienen reacciones inmunológicas y un mecanismo final de apoptosis masiva de queratinocitos epidérmicos. El tratamiento consiste en la retirada del fármaco causal y medidas de soporte, evitando la administración de corticosteroides. Se han descrito tratamientos que pretenden detener la evolución del cuadro, entre ellos ciclofosfamida, ciclosporina, plasmaféresis, pentoxifilina e inmunoglobulinas i.v. El cuadro típico de NET, descrito por Lyell (8), se caracteriza por la aparición brusca, tras un pródromo «catarral», de lesiones eritematosas, dolorosas, inicialmente distribuidas de forma simétrica en cara y tronco, aunque luego se puedan extender hacia las partes acras. Las lesiones comienzan siendo máculas eritematosas, pero pronto aparece el signo característico de la enfermedad: la necrosis y desprendimiento de la epidermis que produce ampollas flácidas y signo de Nikolsky con despegamiento de amplias zonas epidérmicas que dejan erosiones exudativas. La extensión de este despegamiento es uno de los factores pronósticos principales. Un pequeño porcentaje de enfermos presenta sólo eritema confluente y zonas erosivas, pero el 90% tienen además lesiones aisladas, salpicadas en la proximidad de las erosiones. Éstas son máculas de borde mal definido, con forma irregular, y pueden tener un centro más oscuro o ampolloso. Casi todos los pacientes presentan lesiones mucosas, incluyendo erosiones dolorosas orales y faríngeas, lesiones oculares (que conllevan un riesgo elevado de secuelas) y genitales. El daño de otros epitelios, como el respiratorio o digestivo, y las complicaciones de la insuficiencia cutánea aguda agravan el cuadro Signo de NIKOLSKY: desprendimiento de las capas de la piel, aparentemente sana, por efecto de la presión tangencial del dedo, con una ventosa o con un esparadrapo.

Page 32: Examen Subespecialidades

1. Avakian R, Flowers FP, Araújo OE, Ramos-Caro FA. Toxic epidermal necrolysis: a review. J Am Acad Dermatol 1991;25:69-79. 2. Roujeau J-C, Chosidow O, Saiag P, Guillaume J-C. Toxic epidermal necrolysis (Lyell syndrome). J Am Acad Dermatol1990;23:1039-58. 3. Champion RH. Disorders of blood vessels. En: Champion RH, Burton JL, Ebling FJG, eds. Textbook of dermatology, 5.a ed. Oxford: Blackwell Scientific Publications; 1992. p. 18.34-18.38. 4. Fritsch PO, Elias PM. Erythema multiforme and toxic epidermal necrolysis. En: Fitzpatrick TB, Eisen AZ, Wolff K, Freedberg IM, Austen KF, eds. Dermatology in general medicine, 4.a ed. Nueva York: McGraw-Hill; 1993. p. 585-600. 5. Böttiger LE, Strandberg I, Westernholm B. Drug induced febrile mucocutaneous syndrome. Acta Med Scand 1975. 22.- Una mujer de 35 años de edad, afroamericana, acude a consulta debido a que presenta lesiones de piel acrómicas que han ido desarrollándose en los últimos años. Una vez que una placa se desarrolla, parece ser que nunca vuelve a colorearse normalmente de nuevo, y se quema fácilmente a la exposición solar. La EF demuestra 6 placas hipopigmentadas de piel en cuello y tórax que varían en tamaño de 1 a 6cm. La piel afectada no presenta otras alteraciones. ¿Cuál de los siguientes es el diagnóstico más probable? a) Albinismo b) Lentigo c) Vitiligo d) Hipocromía idiomática El vitiligo es una leucodermia adquirida de causa desconocida que se caracteriza por manchas acrónicas y cuyo tratamiento en ocasiones es difícil. Se debe a la desaparición selectiva de los melanocitos epidérmicos en el área de las lesiones. Tiene una prevalencia del 1-2% a nivel mundial; en México ocupa el entre el 3º y 5º lugar entre las dermopatías,

Page 33: Examen Subespecialidades

sin embargo, el impacto psicosocial es importante en muchos de los casos. Se presenta a cualquier edad, principalmente entre los 20 y 40 años, aunque puede verse incluso en niños de 3-4 años; tiene un ligero predominio en el sexo femenino. Es poligénico, el patrón de herencia no ha sido bien establecido, sin embargo, existe una elevada incidencia en parientes de primer grado de pacientes que padecen vitiligo. De manera ocasional se relaciona con padecimientos internos subyacentes, donde destacan factores hormonales y autoinmunitarios, tales como tiroideos (hipotiroidismo, hipertiroidismo, enfermedad de Graves y tiroiditis), que se pueden presentar hasta en el 14% de los pacientes con vitiligo; la evolución de ambos padecimientos es independiente. Otros padecimientos relacionados son: disfunción poliglandular, diabetesmellitus, anemia perniciosa. Se desconoce la etiología del vitiligo pero se tienen diversas hipótesis tales como la neural, la citotóxica, la inmunitaria, la psicosomática y la bioquímica. Elementos diagnósticos. Se caracteriza por mancha acrómicas o hipocrómicas bien delimitadas, que se pueden presentar en cualquier parte del cuerpo, pero es más común en cara (párpados, peribucal), dorso de manos, muñecas, axilas, ombligo, pezones, cintura, región sacra e inguinal; puede ser simétrico, aunque ésta no es una característica. Pueden aparecer nuevas lesiones en sitios de presión o secundarias a un traumatismo (fenómeno de Koebner), éste último se considera como signo de progresión. Las manchas varian en número y tamaño, la superficie es lisa. Otras manifestaciones clínicas son: Nevo en halo o vitiligo perinévico (nevo pigmentado rodeado por una mancha acrónica), alopecia areata y leucotriquia; esta última indica un mal pronóstico para la repigmentación.

Su evolución generalmente es lenta, insidiosa, crónica y son asintomáticas.

Bibliografía: 1) Luger T, Paul C. Potential new indications of topical calcineurin inhibitors. Dermatology. 2007;215 Suppl 1:45-54. 2) Lotti TM. Vitiligo: problemas y soluciones. DermatologíaCMQ 2003;3: 3) Njoo MD, Spuls PI, Bos JD et al. Nonsurgical repigmentation therapies in vitiligo. Arch Dermatol 1998;134:1532-1540. 4) Drake LA, Dinehart SM, Farmer ER et al. Guidelines of care for vitiligo. American Academy of Dermatology. J Am Acad Dermatol 1996;35:620-626. 23.- Se trata de paciente masculino de 66 años con antecedentes de tabaquismo positivo consumiendo más de 30 cigarrillos al día desde hace mas de 40 años, hipertensión arterial descontrolada, acude a consulta por presentar hematuria franca y alteraciones en la función renal, a la exploración física se aprecia masa abdominal en flanco derecho se

Page 34: Examen Subespecialidades

realiza una TAC demostrando masa sólida de aproximados 9 cm de diámetro en riñón derecho. ¿Cuál es el diagnóstico presuncional más probable? a) Nefroblastoma. b) Adenocarcinoma. c) Carcinoma epidermoide. d) Liposarcoma.

La incidencia de cáncer de riñón ha aumentado sustancialmente en el mundo. El carcinoma de células renales (CCR) representa 80 a 85% de todos los tumores malignos de riñón. Varios estudios epidemiológicos indican que el tabaquismo constituye un importante factor de riesgo. Un estudio reciente mostró que dejar de fumar se asocia con una reducción lineal del riesgo de CCR. El tabaquismo pasivo ha sido relacionado con varias consecuencias sobre la salud incluyendo cáncer (de pulmón, estómago y mama). Además, recientemente se ha establecido la asociación causal entre tabaquismo activo y cáncer de riñón. El cigarrillo parece ser el factor de riesgo más importante para cáncer de riñón y explica la mayoría de los casos en casi todo el mundo.

Presentación clínica

La mayoría de los casos de carcinoma renal presenta una tríada clásica de hematuria, dolor abdominal en el flanco y una masa palpable. La hematuria gruesa se encuentra en un 45% de los pacientes. Un 40% de ellos presentan un cuadro clínico llamado síndrome paraneoplásico que comprende anemia, dolor óseo, hipercalcemia, fiebre, pérdida de peso.

La evaluación inicial de un paciente con cáncer renal debe incluir un buen historial y examen físico, laboratorio de rutina y una tomografía computarizada (CT scan) del abdomen y la pelvis, así como una radiografía de pecho. Si hay evidencia o sospecha de trombosis de la vena cava o trombosis tumoral venosa (que ocurre en un 5% a 10% de los casos), se recomienda hacer una resonancia magnética (MRI).

En general, no se recomienda realizar una biopsia percutánea de la masa renal, a menos que se sospeche que se trate de una metástasis de otra neoplasia, como cáncer de colon o linfoma, entre otros.

La resección quirúrgica sigue siendo el tratamiento de elección para los tumores localizados, ya que se sabe que dicha neoplasia es resistente a la quimioterapia y a la radioterapia.

Page 35: Examen Subespecialidades

Klein EA, RM Bukowski, JH Finke: Renal Cell Carcinoma. Immunotherapy and cellular Biology. MARCEL DEKKER, Inc. 1993.

2. RE Lenhard, Jr, RT Osteen, T Gansler: The American Cancer Society’s Clinical Oncology. American Cancer Society, 2001.

3. 2008 Genitourinary Cancers Symposium: A Multidisciplinary Approach. February 14-18 2008.

4. Managing the Complex Journey of Renal Cell Carcinoma. Institute for medical Education & Research. March 2008.

5. Highlights of the NCCN 13th Annual Conference. Clinical Practice Guidelines & Quality Cancer Care. March 5-9, 2008.

6. JS Lam, J Bergman, A Breda, P Schulam: Importance of Surgical Margins in the Management of Renal Cell Carcinoma. Nature Clinical Practice Urology, Medscape.com. May 13, 2008.

7. R. Nelson: Renal cell carcinoma is being increasingly diagnosed at early stages. Medscape Medical News. Medscape.com. May 22, 2008.

24.- Paciente masculino de 40 años de edad, empleado en una fábrica, homosexual, con infección por VIH diagnosticada hace 2 años, acude al servicio de urgencias por presentar fiebre de 15 días de evolución, ataque al estado general, pérdida de peso, disnea progresiva y tos productiva. Se realizó tinción de la expectoración, que reportó bacilos ácido-alcohol resistente. Su cuenta de CD4 realizada hace 1 mes fue de 100 cél/mL y una carga viral reciente fue de 250,000 copias. ¿En qué estadio CDC se encuentra el paciente?

a) B3 b) C3 c) B2 d) A3

Page 36: Examen Subespecialidades

El estadio C3 está definido por una cuenta de CD4 menor de 200 cél/mL y la presencia de una condición clínica indicativa de Síndrome de Inmunodeficiencia Adquirida, entre las que se encuentra la infección por Mycobacterium tuberculosis en cualquier variedad.

5

CLASIFICACION DE INFECCION VIHVIGILANCIA, ADULTOS Y ADOLESCENTES

CDC 1993

A B CCategoría CD4 Asintomático Sintomático* Indicador de

Infxn aguda (no A o C) SIDA

> 500/mm3 A1 B1 C1

200-499/mm3 A2 B2 C2

< 200/mm3 A3 B3 C3

* muguet oral, candidiasis vaginal persistente, displasia cervical, fiebre-diarreade >1mes, leucoplasia vellosa, zoster (2 episodios o >1 dermatoma), PTI, PID

NO TOMA EN CUENTA LA CARGA VIRAL

Kasper DL, Braunwald E, Fauci AS, Hauser SL, Longo DL, Jameson JL. Harrison´s Principles of Internal Medicine. McGraw Hill. 16 Ed. 1076 p. 25.- Se trata de femenino de 22 años, que presenta lesiones eritematoescamosas, edema y alguna vesícula en la cara, escote, dorso de las manos y antebrazos. Las lesiones tienen 12 horas de evolución y han aparecido tras una escursión al campo. Entre los antecedentes personales destaca acné vulgar en tratamiento con retinoides tópicos y doxiciclicina oral. El diagnóstico más probable es: a) Erupción lumínica poliforma.

Page 37: Examen Subespecialidades

b) Eritrodermia por fármacos. c) Urticaria solar. d) Reacción fototóxica. DEFINICIÓN Enfermedades cutáneas que se producen por el aumento de capacidad de reacción de la piel a las radiaciones lumínicas tras la administración de una sustancia fotosensibilizante. Se conocen como reacciones de fotosensibilidad y pueden desencadenarse tanto por contacto como por la administración sistémica del agente fotosensibilizante. Si existe implicación inmunológica se denomina dermatitis fotoalérgica y si no dermatitis fototóxica. Dermatitis fototóxica No existe un mecanismo inmunológico, puede afectar a muchas personas siempre que exista dosis elevada de irradiación y cantidad suficiente de sustancia química. Las lesiones aparecen tras la primera exposición, son monomorfas, y se caracterizan por eritema intenso, edema y vesiculación en áreas de piel fotoexpuestas, marcando claramente los bordes de las zonas descubiertas, y onicolisis ungueal . Formas particulares de fototoxia: fitofotodermatitis (dermatitis de los prados, apio) (Fig. 3), dermatitis de Berloque, fotosensibilidad en tatuajes (sulfuro de cadmio), fármacos (tetraciclinas, AINEs, amiodarona (color azulado), clorpromacina (color gris…).

Page 38: Examen Subespecialidades

Figura 3. Fitofotodermatitis 1. Litt Jz. Drug eruption reference manual 2001. New York: Parthenon, 2001. 2. Sullivan JR, Shear NH. Drug eruptions and other adverse drug effects in aged skin. Clinics in geriatric medicine 2002;18(1). 3. Lim HW, Gigli. Complement-derived peptides in phototoxic reaction. En: Daynes RA, Spikes JD, editors. Experimental and clinical photoimmunology. Boca Raton: CRC Press, 1983:81-93. 4. Torinuki W, Tagami H. Role of complement in chlorpromazine-induced phototoxicity. J Invest Dermatol 1986;86:142-4. 5. Hearst JE, Issacs ST, Kanne D, Rapoport H, Straub K. The reaction of the psoralens with deoxyribonucleic acid. Q Rev Biophys 1984;45:891-5. 6. Athar M, Elmets CA, Bickers DR, Mukhtar H,. A novel mechanism for the generation of superoxide anions in hematoporphyrin derivative-mediated cutaneous photosensitization. Activation of the xantine oxidase pathway. J Clin Invest 1989;83:1137-43. 7. Matsuo I, Inukai N, Fujita H, Ohkido M. Possible involvement of oxidationmof lipids in inducing griseofluvin photosensitivity. Photodermatol Photoimmunol Photomed 1990;7:213-7. 8. Harber LC, Bickers DR. Photosensitivity diseadses. Principles of diagnosis and treatment. Ontario: BC Decker Inc, 1989:160-202. 9. Kockevar IE. Phototoxicity of nonsteroidal inflammatory drugs. Coincidence or specific mechanism?. Arch Dermatol 1989;125:824-6.

Page 39: Examen Subespecialidades

26.- El diagnóstico más probable en un paciente masculino de 17 años de edad que desconoce sus antecedentes familiares. Desarrolla dolor abdominal progresivo, posterior a un traumatismo directo en la región lumbar; se documenta un hematoma en el músculo psoas izquierdo. La Bh con 11 gr. de HB, reticulocitos 35, LEUCOCITOS DE 13500, 350 mil plaquetas, TTP: 60”/35”, TP: 12”/12”, TT: 26”/26” fibrinógeno 350 MG/dl. ¿Cuál es su sospecha diagnóstica?

a) Enfermedad de Von Willebrand b) Hemofilia c) Trombocitopatía d) Deficiencia de factor XII

¿Cuáles son los síntomas de la hemofilia?

El síntoma más común de la hemofilia es la hemorragia incontrolable y excesiva por causa del factor de coagulación que falta o está en bajos niveles en la sangre. Puede producirse una hemorragia incluso cuando no haya ninguna lesión. La mayoría de las veces se produce en las articulaciones y en la cabeza.

A continuación, se enumeran los síntomas más frecuentes de la hemofilia. Sin embargo, cada individuo puede experimentarlos de una forma diferente. Los síntomas pueden incluir:

• Equimosis (Moretones) Pueden producirse moretones por pequeños accidentes, que pueden producir un hematoma grande (una acumulación de sangre debajo de la piel que causa inflamación).

• Sangra con facilidad La tendencia a sangrar por la nariz, la boca y las encías por un traumatismo sin importancia, cepillarse los dientes y, o trabajo dental es a menudo una indicación de hemofilia.

• Hemorragia en una articulación La hemartrosis (hemorragia en una articulación) puede producir dolor, inmovilidad y, con el tiempo, deformación si no se trata médicamente de la forma adecuada. ésta es la zona más común de complicaciones debido a la hemorragia por hemofilia. Estas hemorragias de las articulaciones pueden producir la artritis crónica y dolorosa, deformaciones y parálisis si se repiten.

• Hemorragia en los músculos La hemorragia en los músculos puede causar hinchazón, dolor y enrojecimiento. La hinchazón por el exceso de sangre en estas zonas puede producir un aumento de la presión en los tejidos y nervios de la zona, provocando daño y, o deformación permanente.

Page 40: Examen Subespecialidades

• Hemorragia por lesiones o hemorragia cerebral

La hemorragia por lesión o espontánea en el cerebro es la causa más común de muerte en los niños que tienen hemofilia y la complicación hemorrágica más grave.

• Otras fuentes de hemorragia La sangre en la orina o en las heces puede ser también un síntoma de hemofilia.

Los síntomas de la hemofilia pueden parecerse a los de otros trastornos de la sangre o problemas médicos. Siempre consulte a su médico para el diagnóstico. Roberts HR. Hemophilia A and Hemophilia B. In: Lichtman, MA, et al., eds. Williams Hematology. 7th ed. The McGraw-Hill Companies, Inc.; 2006:chap 115. 27.- Femenino de 45 años de edad que se encuentra en terapia intensiva por manejo de sépsis abdominal. Lleva dos días con tratamiento a base de ceftriaxona, metronidazol y amikacina, el cual provee de una cobertura conveniente para los microorganismos probablemente involucrados. Después de revisar al paciente que ha tenido una evolución favorable, usted sugiere que se aplique el aminoglucósido en monodosis ya que:

a) De esta forma aumenta su potencia. b) Como tiene un efecto post antibiótico corto su efecto es mejor. c) Disminuyen sus efectos adversos. c) Hace sinergismo con el Metronidazol.

Una característica farmacológica de los aminoglucósidos es su actividad dependiente de concentración, por lo que su aplicación en bolos, con lo que se logran niveles pico más altos, es de mayor conveniencia para aprovechar este fenómeno. Además, el importante efecto post-antibiótico que tiene, permiten que la actividad dure más allá del tiempo en que sus concentraciones se encuentran por arriba de la mínima inhibitoria, favoreciendo una posología cómoda cada 24 horas. Por otra parte, se ha visto que los efectos adversos (nefrotoxicidad y ototoxicidad), se encuentran más relacionados con los niveles valle.

Page 41: Examen Subespecialidades

Estos se mantienen a niveles bajos cuando se utiliza el esquema en monodosis, disminuyendo la probabilidad de la presentación de estos efectos adversos. Kasper DL, Braunwald E, Fauci AS, Hauser SL, Longo DL, Jameson JL. Harrison´s Principles of Internal Medicine. McGraw Hill. 16 Ed. 789-806 pp. 28.- Un varón de 18 años presenta cefalea persistente y fiebre, y después de cinco días, un cuadro de crisis convulsiva tónica focal. Una tomografia computadorizada de la cabeza muestra una lesión con reforzamiento anular en el lóbulo frontal derecho y un nivel hidroaéreo en el seno frontal del mismo lado. Lo más probable es que un aspirado neuroquirúrgico de la lesión muestre: a) Células mononucleares pequeñas sugestivas de linfoma de Burkitt. b) Quistes de Toxoplasma gondii y taquizoítos (trofozoítos) c) Escherichia coli y Bacteroides fragilis. d) Estreptococo hemolítico a y mezcla de anaerobios. El absceso cerebral es una patología que se da entre personas jóvenes o de edad media (entre la segunda y cuarta décadas de la vida), con una inexplicable predilección por el sexo masculino (Doble de frecuencia). El 25% de los abscesos cerebrales ocurren en niños menores de 15 años. De forma rara se presentan en neonatos pero tienen una alta morbi-mortalidad (Wu, Lai et al.2006). Localización: La localización de los abscesos está muy relacionada con la etiología de la infección, siendo la predilección de los abscesos de origen otógeno por el lóbulo temporal o cerebelo, los de senos paranasales por el lóbulo frontal, o la frecuente multiplicidad de los de origen hematógeno.

Page 42: Examen Subespecialidades

Etiología: El absceso cerebral se inicia cuando los gérmenes alcanzan el parénquima cerebral, lo cual se produce a través de tres vías fundamentales: por inoculación directa o fístulas, en el caso de traumatismos o cirugía; por una infección en contigüidad (bien por invasión directa, bien por tromboflebitis de las venas emisarias); o por embolismos sépticos procedentes de infecciones a distancia. Tradicionalmente las infecciones crónicas del oído y de los senos paranasales han sido la causa más frecuente de abscesos cerebrales, pero su incidencia ha disminuido con la mejora del tratamiento de estas infecciones, produciéndose así un relativo incremento de la infección de origen hematógeno. Microorganismos más frecuentes: Estreptococo, Estafilococo, Enterobacterias. Los microorganismos más frecuentes son los estreptococos, 33%-50% anaerobios o microaerófilos. Generalmente, se obtienen muchos microorganismos en 10%-30% de los cultivos, pero esta cifra puede aproximarse a 80%-90%, y suelen hallarse cepas anaerobias (son frecuentes las Bacteroides sp.) En los abscesos secundarios a sinusitis frontoetmoidal, se observan streptococcus milleri y Streptococcus anginosus. Cuando son secundarios a otitis media, mastoiditis o abscesos pulmonares, por lo general, se hallan varios tipos de microorganismos: estreptococos anaerobios, Bacteroides, enterobacterias (proteo o proteus).Los abscesos postraumáticos suelen deberse a S. aureus o a enterobacterias. En los pacientes inmunodeprimidos transplantados (tanto de médula ósea como de órganos sólidos), son frecuentes las infecciones fúngicas, la mayoría por Aspergillus fumigatus a partir de una infección pulmonar primaria. Lactantes: son frecuentes los bacilos gramnegativos porque la IgM no pasa a la placenta. Microorganismos que pueden generar abscesos en pacientes inmunodeficientes: 1.toxoplasmosis. 2.nocardia.

RM de un absceso multilobulado

Page 43: Examen Subespecialidades

Bibliografía: 1. Awad I, Bay JW, Petersen JM: Nocardial osteomyelitis of the spine with epidural spinal cord compression--a case report. Neurosurgery 15:254-256, 1984. 2. Byrne E, Brophy BP, Perrett LV: Nocardia cerebral abscess: New concepts in diagnosis, management, and prognosis. J Neurol Neurosurg Psychiatry 42:1038-1045, 1979. Tonon, E., P. G. Scotton, et al. (2006). "Brain abscess: clinical aspects of 100 patients." Int JInfectDis10 (2):103-9. 3.Tseng, J. H. and M. Y. Tseng (2006). "Brain abscess in 142 patients: factors influencing outcome and mortality." Surg Neurol 65(6): 557-62; discussion 562. 4.Visani, P., E. Schmutzhard, et al. (2006). "Subcortical deficit pattern after brain abscess: a neuropsychological study." Eur J Neurol 13(6): 599-603. 5.Wandroo, F., P. Stableforth, et al. (2006). "Aspergillus brain abscess in a patient with acute myeloid leukaemia successfully treated with voriconazole." Clin Lab Haematol 28(2): 130-3. 29.- Femenino que desde hace varias semanas le han aparecido de forma eruptiva unas máculas y placas eritematosas en el tronco, refiere que hubo una lesión más grande que precedió a las demás. Las lesiones presentan una descamación fina en la periferia y son dis-cretamente pruriginosas. No existe afectación palmoplantar. La serología luética es negativa. ¿Qué tipo de pitiriasis es la más probable? a) P. liquenoide crónica. b) P. rubra pilaris. c) P. rosada. d) P. versicolor

La pitiriasis rosada es una enfermedad exantemática autolimitada de causa desconocida que cursa con lesiones maculopapulosas de aspecto asalmonado en áreas proximales de las extremidades y en tronco, donde se distribuyen de forma paralela a las líneas de la hendidura (Stulberg DL, 2004).

Existen algunas características que sugieren una etiología vírica (pródromos, predominio estacional, evolución variable, ausencia de recaídas, hallazgos histológicos...) que algunos autores relacionan con el herpes virus humano 7, aunque otros lo desmienten (Chuh A, 2004).

Page 44: Examen Subespecialidades

Afecta con más frecuencia a niños mayores y jóvenes adultos (10-35 años) y es ligeramente más frecuente en mujeres (Chuang TY, 1982).

El cuadro típico se inicia con una lesión redondeada u oval, de 4-8 cm. única, generalmente situada en el tronco, de color asalmonado, discretamente descamativa y en ocasiones ligeramente pruriginosa, llamada “madre”. Puede acompañarse de síntomas generales leves: astenia, anorexia, odinofagia, cefalea y artralgias.

En 1-2 semanas esta lesión se torna más escamosa y clara en el centro al tiempo que aparecen otras de menor tamaño que se diseminan por las extremidades (áreas proximales) y por el tronco distribuyéndose simétricamente en forma arbórea a lo largo de las hendiduras costales. En la espalda es más evidente (“signo del árbol de Navidad”) (Stulberg DL, 2004)

La evolución de las lesiones es similar a las de la lesión “madre” desapareciendo en 5-6 semanas, aunque en algunos casos puede permanecer más tiempo y en otros verse áreas hiper o hipo pigmentadas durante algún tiempo. La tasa de recurrencia a los 5 años es inferior al 2% (Chuang TY, 1982).

Pitiriasis Rosada

Chuh A, Chan H, Zawar V. Pityriasis rosea--evidence for and against an infectious aetiology. Epidemiol Infect. 2004 Jun;132(3):381-90.

Chuh AA, Chan HH. Prospective case-control study of chlamydia, legionella and mycoplasma infections in patients with pityriasis rosea.

Chuh AA, Dofitas BL, Comisel GG, Reveiz L, Sharma V, Garner SE, Chu F. Interventions for pityriasis rosea. Cochrane Database Syst Rev. 2007 Apr 18;(2):CD005068.

Chuh AA. Quality of life in children with pityriasis rosea: a prospective case control study. Pediatr Dermatol. 2003 Nov-Dec;20(6):474-8.

Hsu S, Le EH, Khoshevis MR. Differential diagnosis of annular lesions. Am Fam Physician. 2001 Jul 15;64(2):289-96] Eur J Dermatol. 2002 Mar-Apr;12(2):170-3.

Miranda SB, Lupi O, Lucas E. Vesicular pityriasis rosea: response to erythromycin treatment. J Eur Acad Dermatol Venereol. 2004 Sep;18(5):622-5

Page 45: Examen Subespecialidades

30.- Recibe el siguiente perfil de hepatitis viral: anticuerpos contra Hepatitis C negativos (-); Hepatitis B: Antígeno de superficie positivo (+), Anticuerpos contra antígeno de superficie negativos (-), inmunoglobulina G contra Antígeno core positiva (+), Antígeno e (Anti-HBe) positivo (+). ¿Con lo anterior diagnostica?

a) Infección aguda C. b) Infección antigua C. c) Infección aguda por hepatitis B. d) Infección crónica por hepatitis B, con replicación viral.

La infección aguda de hepatitis B se caracteriza por la presencia de inmunoglobulina M contra antígeno del core, que no tiene este paciente. La presencia de inmunoglobulina G contra éste mismo antígeno la caracteriza como crónica, y la presencia de antígeno e determina que existe replicación activa. CRITERIOS DIAGNÓSTICOS 1. Determinar la presencia de infección crónica por el VHB, que vendrá dictada por la positividad del HBsAg durante más de 6 meses. 2. Determinar la existencia de replicación viral activa, caracterizada por la presencia en suero del ADN-VHB. Existen distintos métodos para determinar el ADN-VHB sérico; si se utilizan métodos poco sensibles, como la hibridación molecular, se pueden obtener resultados falsamente negativos y por el contrario, técnicas muy sensibles, como la reacción en cadena de la polimerasa, pueden ofrecer resultados positivos en portadores del VHB en los que la replicación del virus no tiene repercusión clínica relevante. 3. Determinar la existencia de enfermedad hepática y conocer su estadío. La elevación de los valores de transaminasas constituye el marcador más sencillo de la existencia de enfermedad hepática activa. En tales casos, la realización de una biopsia hepática permitirá confirmar el diagnóstico de hepatitis crónica, conocer su grado de actividad, así como el estadio de fibrosis existente. 4. Descartar otras causas de enfermedad hepática. Otros virus de la hepatitis (VHD o VHC), o incluso otras causas de enfermedad hepática pueden ser los responsables, en asociación o no con el VHB, de la lesión hepática en portadores crónicos de este virus. El conocimiento de estas otras posibles causas de enfermedad hepática es imprescindible para planificar el tratamiento oportuno. La infección crónica por el VHB es un proceso dinámico, sujeto a la interacción entre el propio virus y el sistema inmune, que tiene como resultado la existencia de distintas fases, con datos virológicos y clínicos diferenciales:

Page 46: Examen Subespecialidades

HEPATITIS CRÓNICA POR VIRUS DE LA HEPATITIS B (Tabla 1). Tabla 1. Fases de la infección crónica por el VHB HBsAg HBeAg ADN-VHB Transaminasas Histología Inmunotolerancia Positivo Positivo Positivo Normales Normal Cambios Mínimos Hepatitis crónica Positivo Positivo Positivo Elevadas Hepatitis HBeAg+ crónica Hepatitis crónica Positivo Negativo Positivo Elevadas Hepatitis HBeAg - crónica Portador sano Positivo Negativo Negativo Normales Normal del VHB hepatitis "residual" BIBLIOGRAFÍA Kasper DL, Braunwald E, Fauci AS, Hauser SL, Longo DL, Jameson JL. Harrison´s Principles of Internal Medicine. McGraw Hill. 16 Ed. 1835 p. Benhamou Y, Bochet M, Thibault V et al. Long-term incidence of hepatitis B virus resistance to lamivudine in human immunodeficiency virusinfected patients. Hepatology 1999; 30: 1302-1306. Chayama K, Suzuki Y, Kobayashi M et al. Emergence and takeover of YMDD motif mutant hepatitis B virus during long-term lamivudine therapy and re-takeover by wild type after cessation of therapy. Hepatology 1998; 27: 1711-1716.

Page 47: Examen Subespecialidades

Chien R-N, Liaw Y-F, Atkins M. Pretherapy alanine transaminase level as a determinant for hepatitis B e antigen seroconversion during lamivudine therapy in patients with chronic hepatitis B. Hepatology 1999; 30: 770-4. Dienstag JL, Schiff ER, Mitchell M et al. Extended lamivudine retreatment for chronic hepatitis B: manteinance of viral suppression after discontinuation of therapy. Lai C-L, Chien R-N, Leung NWY et al. A one year trial of lamivudine for chronic hepatitis B. N Engl J Med 1998; 339: 61-8. 31.- Una niña de 2 años de edad presenta lesiones rojas, con costras melicéricas en la cara, cuero cabelludo, área de pañal y extremidades, desde aproximadamente los 2 meses de edad. El evitar sustancias irritantes no ha funcionado en el tratamiento. La niña se rasca constantemente las áreas afectadas. Hay una historia familiar positiva para asma y fiebre. ¿Cuál es el diagnóstico más probable? a) Dermatitis atópica b) Celulitis c) Dermatitis de contacto d) Liquen cimple crónico

Page 48: Examen Subespecialidades

La dermatitis atopica (DA) (Wise-Sulzberger, 1993), llamada neurodermatitis diseminada, por las escuelas Europea, es un estado reaccional de la piel, intensamente pruriginosa, más frecuente en los niños, multifactorial, en la que combinan factores constitucionales y factores ambientales, por lo tanto de difícil tratamiento, muy frecuente en la consulta diaria del pediatra y del dermatólogo. DATOS EPIDEMIOLOGICOS La enfermedad originalmente conocida como prúrigo de Besnier y eccema constitucional, es ubicua, afecta a todas las razas y existe en todos los países. Se señala su existencia hasta en el 2% de la población general y en el 14% de los niños. La enfermedad se inicia antes del primer años de la vida en más del 60% de los casos, la curva desciende hacia los 12 años de la cifra de inicio es apenas del 5%-y es excepcional que la enfermedad se inicie en la edad adulta. La enfermedad sufre exacerbaciones en las temporadas de calor o de frío cuando hay sequedad de la atmósfera. CUADRO CLÍNICO Tradicionalmente se han descrito tres etapas que en la actualidad ya no se presentan pues se suman unas a otras debido a los tratamientos que reciben los pacientes desde su inicio. Etapa de lactante. La enfermedad suele iniciarse en los primeros meses de vida, a veces casi desde el nacimiento. Las lesiones afectan la cara: mejillas, frente (respetando su centro); afectan también la piel cabelluda, los pliegues, las nalgas y el dorso de los pies. Las lesiones son de tipo eccematosos (piel llorosa): eritema, vesícular y costras melicéricas con costras hemáticas como signo del rascado. Estas lesiones altamente pruriginosas evolucionan por brotes y en general si el paciente no es yatrogénicamente tratado, tienen tendencia a involucionar al año de edad. Fase del escolar. Se inicia hacia los 3 a 7 años, cuando el niño empieza a ir a la escuela. Las lesiones son preferentemente flexurales: Cuello, pliegues de codo y huecos poplíteos y la morfología corresponde a la de una dermatitis crónica: zonas de eritema y liquenificación (índice de rascado crónico) y costras hemáticas periódicamente sufren un proceso de eccematización sobre todo por los tratamientos indebidos. La enfermedad evoluciona por brotes, hay temporadas en que no existen lesiones aun cuando persiste a veces una piel xerótica (seca) y pigmentada con intenso prurito. Etapa del adulto. Se veía con poca frecuencia y ahora es habitual en las consultas diarias debido a los malos tratamientos. En este caso además de las zonas flexurales, se presenta lesiones periorbitarias; y peribucales y lesiones vesiculosas en las manos. Son lesiones tanto liquenificadas como eccematosas, muy pruriginosas, que alternan con periodos asintomáticos.

Page 49: Examen Subespecialidades

Con el uso inmoderado de los corticoesteroides no hay diferenciación entre las etapas de la enfermedad y se ven casos que arrastran su enfermedad casi desde que nacen hasta la edad adulta. DIAGNÓSTICO Es clínico y relativamente sencillo, aunque no toda dermatitis flexural es necesariamente de origen atópico. Deben tomarse en cuenta los antecedentes del propio enfermo y los familiares. En los lactantes debe diferenciarse de la dermatitis seborreica que afecta sobre todo la piel cabelluda y el centro de la cara. En ocasiones hay mezcla de las dos dermatitis: la atópica y la seborreica y es difícil diferenciarlas. La dermatitis del pañal predomina en los lactante; en las zonas glúteas y genitales, se presenta con eritema y lesiones vesículopapulosas muy pruriginosas. La pitiriasis alba corresponde a los clásicos “jiotes” de los niños: manchas hipocrómicas cubiertas de fina escama en la cara. Estos procesos pueden coincidir con manifestaciones de DA, pero nunca se ha demostrado la relación de causa efecto. TRATAMIENTO Medidas generales. Es conveniente una explicación amplia al paciente y su familia señalando la naturaleza y evolución de la enfermedad y lo que se espera del tratamiento que vamos a indicar. Evitar el sol excesivo, el agua clorada de las albercas, el uso de jabón (indicar jabones neutros o sustitutos de jabón), prohibir el uso de pomadas y remedios caseros así como los cosméticos que irritan a la piel. Las dietas restrictivas han sido y siguen siendo un tema muy controversial, pues mientras los alergistras siguen insistiendo sin muchas bases en el beneficio de estas dietas, los más hemos comprobado su inutilidad. Existe un grupo muy limitado en que se puede demostrar que una dieta restrictiva de huevo, leche, fresas, etc. puede mejorar los brotes de DA y en tales casos (sólo en esos casos) se aconsejaría la supresión de tales medicamentos; en lo general se permite al paciente que coma de todo. En la actualidad se están limitando por los pediatras ciertos alimentos como el huevo, las fresas, el plátano y la leche de vaca en el primer año de vida por ser muy alergénicos y se discute la utilidad de la leche materna en estos niños. Es necesario hacer ver que estos pacientes requieren de una atención más personal, que sienten la necesidad de cariño por lo que es de recomendarse que el niño sea atendido personalmente por la madre.

Page 50: Examen Subespecialidades

Tratamiento tópico. Depende del estado de la piel; si está eccematosa debe secarse antes de aplicar cualquier pomada que será rechazada por la piel llorosa. El uso de fomentos con agua de manzanilla o suero fisiológico es útil. Los fometos con agua de végeto (subacetato de plomo) al 20% son potentes antiexudativos, pero deben limitarse a áreas restringidas y no usarse en niños. Estando la piel ya seca, se usan pasta inertes que llevan óxido de zinc y calamina en una base de vaselina y lanolina que son protectoras a la vez que antiprutiginosas. Si la piel está muy seca y liquenificada, los fomentos y baños serán emolientes, con almidón y aceites seguidos de cremas o pomadas más grasosas que llevan vaselina, coldcream y óxido de zinc. En el comercio existen numerosos preparados humectantes que ayudan a mantener el manto ácido grasa-agua, que se pierde en la DA. Si hay dermatitis por contacto o impétigo hay que tratar primero estas complicaciones con sulfato de cobre al 1: 1000 y pomadas con vioformo o mupirocín. En el caso de eritrodermia, los baños emolientes y el uso del petrolato (vaselina) será lo indicado. Cuando hay mucha liquenificación, pueden usarse cremas con alquitrán de hulla al 3% en base de coldcream por tiempo y zonas limitadas. Los corticoesteroides tópicos son los medicamentos más usados en esta enfermedad y muchas veces causa de las complicaciones que se presentan. Estos medicamentos no curan nada, solo engañan al paciente y al médico haciéndoles creer que la enfermedad va curando cuando sólo se oculta y modifica. Al pasar el efecto de estos medicamentos, invariablemente se presenta el rebote y más tarde la corticodependencia, haciendo a la enfermedad incontrolable. Nunca deben usarse los corticoesteroides fluorinados en niños y en la cara y zonas genitales o en los pliegues por su posibilidad de absorción. La hidrocortisona es de baja potencia, hace menos daño, pero también es menos efectiva; algunos la recomiendan. Tratamiento sistémico. El uso de antihistamínicos sobre todo de la primera generación que son sedantes como la clorfeniramina y la hidroxicina ayudan a mejorar el prurito; el ketotifeno y la oxotamida por su acción dual: inhiben la producción de histamina por los mastocitos y bloquean los receptores Hl, también son de ayuda, al igual que los sedantes suaves tanto para el paciente como para la madre que está en perenne angustia que transmite al pequeño paciente. La talidomida ha mostrados ser de ayuda en casos de DA conticoestropeada, en su fase eritrodérmica, no tanto en los niños, a dosis de 100 mg al día. Los antibióticos tipo dicloxacilina, serán necesarios cuando haya infección o simplemente eccematización por el papel que tiene el estafilococo dorado.

1. Criterios Diagnóstico para Dermatitis Atópica (DA), MedicalCriteria.com. 2. Dr. Luciano Domínguez-Soto, Dr. Amado Saúl Cano, Dermatitis atópica o

neurodermatitis diseminada, Dermatología, Parte C, Libro ) "

Page 51: Examen Subespecialidades

32.- Masculino de 48 años de edad, presenta erupción cutánea descamativa asintomática, con afectación del cuero cabelludo, cejas, pestañas, eminencias malares y pliegues nasolabiales. En la exploración se pone de manifiesto una descamación seborreica sobre una piel moderadamente eritematosa en las áreas afectadas. En la región anterior del tórax se observa un moderado enrojecimiento con descamación. El diagnóstico más probable es: a) Dermatitis seborreica. b) Psoriasis. c) Lupus eritematoso. d) Tiña corporis. CLÍNICA La dermatitis seborreica se puede clasificar en: 1) Dermatitis Seborreica en la infancia: a) Costra láctea. B) Dermatitis seborreica infantil. C) Falsa tiña amiantácea. D) Eritrodermia descamativa de Leiner-Mousses. 2. Dermatitis Seborreica en el adulto: Pitiriasis cápitis. B) Pitiriasis esteatoide. C) Dermatitis seborreica facial, mediotóracica y medioescapular y púbica. 3. Dermatitis Seborreica en enfermos inmunocomprometidos. La costra láctea se manifiesta en los primeros meses de vida y se manifiesta mediante placas escamosas, untuosas al tacto, de color grisáceo que localizan en cuero cabelludo (Fig. 1). También pueden existir lesiones eritemato-descamativas centrofaciales y mediotorácicas. La falsa tiña amiantácea se manifiesta como gruesas escamocostras en cuero cabelludo, de color blanco-grisáceo que al despegarse dejan zonas húmedas. La eritrodermia descamativa de Leiner Mousses comienza de forma repentina, a los pocos meses de vida, con lesiones en cuero cabelludo, evolucionando de forma rápida a lesiones eritematosas y descamativas que ocupan casi todo el tegumento. Suele acompañarse de candidiasis de pliegues, infecciones locales y sistémicas, diarreas. A veces el pronóstico es malo. Las manifestaciones clínicas de la dermatitis seborreica infantil o clásica del adulto sólo se manifiestan en la edad de presentación. Conforman lesiones eritemato-descamativas que localizan en la zona de inserción de cuero cabelludos, pliegues retroauriculares, conducto auditivo externo, surcos nasolabiales y nasogeniamos, cejas, glabela, zona mediotorácia e interescapular, axilas, ingles, ombligo, pubis . Suele ser pruriginosa. En pacientes HIV (+) las manifestacionesclínicas de la dermatitis seborreica son mucho más intensas. DERMATITIS SEBORREICA Dres. E. Herrera y G. Ruíz del Portal.

Page 52: Examen Subespecialidades

33.- Femenino de 52 años con antecedente: Dx. De síndrome anémico sin causa aparente. Exámenes de Laboratorio Reportan: Hb de 8,5 g/dl, VCM de 75 fl, y HCM de 25 pg. El examen de laboratorio que conforma el origen del diagnóstico de esta paciente es la determinación de:

a) Sideremia. b) Saturación de la transferrina. c) Hemoglobina A2. d) Ferritina.

La anemia por déficit de hierro ¿es un proceso que aparece de forma rápida o de forma más bien lenta? La anemia por falta de hierro se va estableciendo de forma gradual, cuando el hierro que se pierde supera al hierro aportado por la dieta; lo primero que ocurre es que el organismo empieza a utilizar las reservas de hierro. En esta etapa la ferritina empieza a descender, pero el hierro sérico y la TIBC generalmente permanecen inalterados no estableciéndose todavía anemia. En el momento en que el déficit de hierro empeora, el hierro sérico disminuye mientras que la transferrina y la TIBC aumentan, empezando las células de la serie roja (hematíes) a palidecer (hipocromia) y a disminuir de volumen (microcitosis), existiendo todavía un número suficiente de células de la serie roja. Si la deficiencia de hierro continúa o empeora aparece entonces la anemia. Mourey L. Manual de procedimientos del Laboratorio Clínico. México: IMSS, 1978: 158. 34.- Femenino acude a su consultorio con diagnóstico de e anemia microcítica e hipocrómica, Resultados de laboratorio reportan: hierro sérico y ferritina bajos, aumento de la capacidad de fijación del hierro y de la protoporfirina eritrocitaria, con hemoglobina A2 disminuida. La entidad responsable de los resultados de laboratorio de esta paciente es:

a) Anemia por deficiencia de hierro. b) Anemia de los trastornos crónicos. c) Anemia sideroblástica. d) Talasemia beta.

Page 53: Examen Subespecialidades

Frotis de sangre periférica: hipocromía, microcitosis, policromatofilia o punteado basófilo(eventualmente). Reticulocitos: Generalmente normales. Si están aumentados, investigar pérdidas por hemorragia o posibilidad de otro diagnóstico Plaquetas: normales. Si están elevadas, investigar pérdidas por hemorragia. Leucocitos: normales. Índices hematimétricos: volumen corpuscular medio (VCM) y concentración de hemoglobina corpuscular media (CHCM) disminuidos. Los límites inferiores normales (X – 2 DE) que se deben considera a distintas edades Pruebas que evalúan el hierro del Compartimiento funcional: Sideremia: disminuida (< 60 μg/dl). Capacidad total de saturación del Hierro (TIBC): aumentada, a meno que coexista proceso infeccioso, inflamatorio o tumoral. Porcentaje de saturación: disminuido (< 16%). Protoporfirina libre eritrocitaria: aumentada (> 70 μg/dl). Receptores solubles de transferrina: aumentados (> 30 nMol/l). c3. Pruebas que evalúan el hierro del compartimiento de depósito: Ferritina sérica: Disminuida (< 12 ng/ml), excepto que coexista proceso infeccioso o inflamatorio. Medulograma: hierro de depósito (SRE) ausente. Anemia ferropénica. Normas de diagnóstico y tratamiento / 163 Arch.argent.pediatr 2001; 99(2) / 162 35.- Masculino de 28 años, acude a consulta con resultado de exámenes de laboratorio: microcitosis, hematíes 5280000/microl, hemoglobina 12,3 g/dl, Hcto 36,6%, VCM 69,4 fl, HCM 23,3 pg, serie roja microcítica e hipocroma. Sideremia 78 microg/dl, transferrina 312 microg/dl, IST 25%, ferritina 71 ng/ml, HbA2 2,0%, HbF 0,6%, Antecedentes: 3 años antes, de melenas, hematoquecia por hemorroides externas. Tratado con hierro. El diagnóstico más probable en este paciente es:

a) Anemia ferropénica parcialmente tratada. b) Anemia sideroblástica adquirida. c) Portador de beta talasemia. d) Rasgo alfa talasémico.

Talasemia Talasemia es el nombre genérico con el que se conoce a un grupo de enfermedades hereditarias de la sangre que incluyen anomalías en la hemoglobina, el componente de los

Page 54: Examen Subespecialidades

glóbulos rojos encargado de transportar el oxígeno. La hemoglobina está compuesta principalmente por dos clases de proteínas denominadas globina alfa y globina beta. Las personas que padecen talasemia no producen suficiente cantidad de una de estas proteínas (y, en ocasiones, de las dos). En consecuencia, sus glóbulos rojos pueden ser anormales y no estar en condiciones de transportar suficiente oxígeno por todo el cuerpo.

Los dos tipos principales de talasemia se denominan talasemia alfa y talasemia beta. Las personas afectadas por talasemia alfa no producen suficiente cantidad de globina alfa y las afectadas por talasemia beta no producen suficiente cantidad de globina beta. Existen distintos tipos de talasemia alfa y beta, con síntomas que van de leves a graves.

La talasemia se encuentra entre los trastornos genéticos más comunes en todo el mundo.1 Cada año nacen en todo el mundo más de 100,000 bebés afectados con formas graves de talasemia.2 Esta enfermedad ocurre con mayor frecuencia en personas de ascendencia italiana, griega, asiática y africana, así como en aquellas de familias oriundas del Medio Oriente.3

¿Qué es la talasemia alfa?

Existen al menos cinco tipos principales de talasemia alfa, que afectan principalmente a personas de ascendencia hindú, africana, del sur de China, de Medio Oriente y del sudeste asiático.4 Los genes que controlan la producción de globina alfa son cuatro. La gravedad de la enfermedad se determina de acuerdo con el número de genes anormales o faltantes.

• El portador silencioso, la forma más leve de la enfermedad, tiene un solo gen de globina alfa anormal o faltante. Por lo general, las personas afectadas no presentan síntomas pero pueden transmitir la anomalía genética a sus hijos.

• Las personas con talasemia alfa menor (también denominada rasgo talasémico alfa) tienen dos genes de globina alfa anormales o faltantes. Por lo general, las personas afectadas no presentan síntomas o presentan una anemia leve pero pueden transmitir la enfermedad a sus hijos.

• La enfermedad de la hemoglobina H es causada por tres genes de globina alfa faltantes o anormales (las personas afectadas tienen un gen de globina alfa normal). La enfermedad trae como consecuencia anomalías en los glóbulos rojos y la rápida destrucción de los mismos. La mayoría de las personas afectadas padece anemia de leve a moderada y puede llevar una vida relativamente normal. La anemia puede empeorar temporalmente cuando la persona afectada contrae infecciones virales o cuando se trata con ciertos medicamentos (tales como medicamentos sulfa).5 Algunas personas afectadas pueden desarrollar complicaciones con el tiempo como, por ejemplo, el agrandamiento del bazo y cálculos biliares.5 Las personas que padecen la enfermedad de la hemoglobina H deben recibir atención médica regular para detectar y tratar estas complicaciones. Algunas incluso pueden necesitar transfusiones de sangre ocasionalmente.6

Page 55: Examen Subespecialidades

• La hemoglobina H-Constant Spring es una forma más grave de la enfermedad de

hemoglobina H. Las personas afectadas tienen un solo gen de globina alfa normal y una mutación específica denominada Constant Spring en uno de sus tres genes anormales. Las personas afectadas por esta enfermedad generalmente padecen

• anemia de moderada a grave y a menudo desarrollan complicaciones, tales como el agrandamiento del bazo. En algunas oportunidades, algunas necesitan recibir transfusiones de sangre, por ejemplo, cuando desarrollan una enfermedad con fiebre, mientras que otras necesitan recibir transfusiones con mayor frecuencia.5,6

• La talasemia alfa mayor, el tipo más grave, se da cuando no hay genes para la producción de globina alfa. Los fetos afectados padecen anemia grave, insuficiencia cardíaca y acumulación de líquido. Por lo general, nacen sin vida o mueren pocas horas después del parto. En casos poco frecuentes, han sobrevivido bebés a los que se diagnosticó la enfermedad antes de nacer y que recibieron tratamiento mediante transfusiones de sangre. Estos bebés necesitan recibir transfusiones de sangre de por vida.4,5

¿Qué es la talasemia beta?

Existen tres formas principales de talasemia beta, que afectan principalmente a personas con ascendencia griega, italiana, africana, de Medio Oriente, del sudeste asiático y del sur de China.4 El control de la producción de globina beta está a cargo de dos genes y las mutaciones en uno o ambos genes pueden provocar este trastorno. La gravedad de la enfermedad depende de si uno o ambos genes de globina beta son portadores de una mutación y de la gravedad de la mutación.

• La talasemia menor (también denominada rasgo talasémico) es causada por una mutación en un gen de globina beta. La mayoría de las personas afectadas no presenta síntomas aunque algunas padecen anemia leve. Las personas afectadas pueden transmitir el gen anormal a sus descendientes.

• La talasemia intermedia es resultado de anomalías en ambos genes de globina beta. Por lo general, estas anomalías genéticas son menos graves que las que causan la talasemia mayor. Por lo general, los niños afectados padecen anemia de leve a moderada y pueden desarrollar algunas de las complicaciones que se observan en la talasemia mayor, como agrandamiento del brazo y anomalías en los huesos. Muchas de las personas afectadas necesitan, ocasionalmente o con mayor frecuencia, transfusiones de sangre para reducir las complicaciones.1

• La talasemia mayor, la forma más grave, es resultado de mutaciones graves en ambos genes de globina beta. También se denomina “anemia de Cooley”, como homenaje al médico que la describió por primera vez en el año 1925. La mayoría de los niños afectados parecen saludables al nacer. No obstante, durante el primer o el segundo año de vida se vuelven pálidos e irritables y pierden el apetito. Su crecimiento es lento y a menudo tienen ictericia (su piel y sus ojos adquieren un

Page 56: Examen Subespecialidades

color amarillento). Sin el tratamiento adecuado, estos niños desarrollan un bazo y un hígado de mayor tamaño, huesos débiles que se quiebran con facilidad, huesos faciales anormales, infecciones frecuentes y problemas cardíacos y mueren dentro de los primeros diez años de vida. Los niños afectados requieren transfusiones de sangre periódicas desde la lactancia.

Referencias:

1. Rund, D. and Rachmilewitz, E. Medical Progress: Beta-Thalassemia. New England Journal of Medicine, volumen 353, número 11, 15 de septiembre de 2005, págs. 1135-1146.

2. New York Academy of Sciences. Cooley’s Anemia Eighth Symposium. Publicado 22 de julio de 2005, consultado 2 de mayo de 2008, www.nyas.org/cooleys.

3. National Heart, Lung and Blood Institute. Thalassemias. Publicado 1/08, www.nhlbi.nih.gov/health/dci/Diseases/Thalassemia/Thalassemia_All.html.

4. Cooley’s Anemia Foundation. About Thalassemia. Actualizado 2007, http://www.cooleysanemia.org/.

5. Northern California Comprehensive Thalassemia Center. Alpha Thalassemia. Consultado 2 de mayo de 2008, www.thalassemia.com/alpha_thal.html.

6. Cohen, A.R., et al. Thalassemia. Hematology 2004, American Society of Hematology, págs. 14-34.

7. Food and Drug Administration (FDA). FDA Approves First Oral Drug for Chronic Iron Overload. FDA News, 9 de noviembre de 2005. 36. - A 19 year old man with acute non lymhocytic leukemia is admitted to he hospital 2 weeks after hi first round of chemotherapy. His temperature is 39.2 C, and physical examination shows a no localized abnormalities. Chest radiograph shows a Hickman catheter with its tip in the right atrium. The white blood cell is 300/uL with no polymorphonuclear or band cells in the differential count. Blood cultures are obtained. The next step is to�

a) Initiate antistaphylococcal treatment for the possibility of Hickman catheter related bacteremia.

b) Administer broad spectrum antibiotics with excellent activity for enteric gramnegative rods ad Pseudomonas aeruginosa.

c) Await results of blood cultures and other diagnostic tests because infection could be caused by almost any microorganism.

Page 57: Examen Subespecialidades

d) Administer parenteral antifungal therapy.

Pseudomonas aeruginosa es un patógeno oportunista que causa infecciones del aparato urinario y respiratorio, de los tejidos blandos, endocarditis y una variedad de infecciones sistémicas, particularmente en pacientes quemados, con cáncer, fibrosis quística o compromiso del sistema inmune. Pseudomonas aeruginosa ha emergido como uno de los más importantes patógenos hospitalarios causantes de infecciones graves. Las infecciones asociadas a catéteres son una importante causade morbi-mortalidad. Características en pacientes con infección relacionada a catéteres: Infeccion local; fiebre de origen desconocido en paciente con catéter de más de 3 días; con hemocultivos positivos sin otro foco probable; normalización de la temperatura luego de la retirada del dispositivo. IDSA Guidelines for the management of intravascular catheter related infections. Clinical Infectious Diseases 2001; 32:1249. Centers for Disease Control/ Hospital Infection Control Practices Advisory Committee. “Guidelines for Prevention of Intravascular Device-Related Infections” American Journal of Infection Control 1996: 24: 262-293. 37.- Es una de las enfermedades tiroideas autoinmunes organo-específica más frecuentes. Es más común en la mujer, posee una asociación directa con otras enfermedades autoinmunes y, por lo general, se presenta con hipertiroidismo, bocio difuso, oftalmopatía, además, en algunos casos coincide con mixedema pretibial:

a) Carcinoma basocelular b) Enfermedad de Graves c) Adenoma tóxico d) Síndrome del eutiroideo enferm

Introducción

La enfermedad de Graves-Basedow (EGB) constituye la entidad más importante, por su frecuencia, entre las enfermedades que producen hipertiroidismo. Se caracteriza por la presentación de la tríada sintomática constituida por hipertiroidismo, bocio difuso y oftalmopatía (50%). Es más frecuente en la 3ª y 4ª década de la vida y afecta con más

Page 58: Examen Subespecialidades

frecuencia a mujeres (7/1 en zonas no bociógenas y 3/1 a partir de los 45 años en zonas de bocio endémico) 1.

Etiopatogenia

Aunque no se conocen exactamente los factores que inician y mantienen la enfermedad, hay elementos que permiten encuadrarla en el marco de la patología autoinmunitaria, como la presencia de autoanticuerpos antitiroglobulina y antitiroperoxidasas (TPO), así como la presencia de inmunoglobulinas antireceptor de la TSH (TRAb) que son estimuladoras de la función y del crecimiento del tiroides.

Para el desarrollo de la enfermedad parece que es necesaria la intervención de factores genéticos y ambientales (exceso de Yodo) 2.

Clínica

La enfermedad se caracteriza por la presencia de síntomas de hipertiroidismo (nerviosismo 99%, sudoración 91%, intolerancia al calor y palpitaciones 89%, pérdida de peso 85%, aumento de apetito 69% y síntomas oculares 55%) y signos clínicos (aumento uniforme del tamaño de la glándula 97%, oftalmopatía 60%, taquicardia en reposo 90%, temblor 70% y retracción palpebral 38%) relacionados con el aumento de receptores adrenérgicos que determinan las hormonas tiroideas 1,2.

La historia natural de la enfermedad se caracteriza por fases cíclicas de exacerbación y remisión, de duración y presentación imprevistas; aunque, hoy en día, está artefactada por los tratamientos utilizados. En aproximadamente el 25% de los pacientes, especialmente en aquellos con una forma leve de enfermedad, el proceso se autolimita al año o más, regresando espontáneamente a un estado eutiroideo 3.

En cuanto a la oftalmopatía, hay una forma no infiltrativa o pálpebro-retráctil (afecta más frecuentemente al sexo femenino, afecta simétricamente a ambos ojos y que evoluciona paralelamente al hipertiroidismo) y una forma infiltrativa (infiltración del tejido retroorbitario por mucopolisacáridos y linfocitaria con protusión ocular asimétrica)1.

Otras manifestaciones menos frecuentes son la aparición de mixedema pretibial o dermopatía infiltrativa y, la acropaquia tiroidea 1.

Referencias

1.- Foz M. Enfermedades del tiroides. En Rozman C (ed): Farreras-Rozman Medicina interna. Editorial Doyma. Barcelona, 1992:1997.

Page 59: Examen Subespecialidades

2.- Wartofsky L. Diseases of the thyroid. En Braunwald E, Isselbacher KJ, Wilson JD, Martin JB, Fauci AS, Kasper DL (eds): Harrison’s principles of internal medicine. Editorial McGraw-Hill. EEUU, 1994: 1930.

3.- De Groot. Graves’ diseases and the manifestations of thyrotoxicosis. En De Groot LJ, Reed Larsen P, Hennemann G.(eds): The thyroid and its diseases. 1996: 371.

4.- Haynes RC, Murad F. Drogas tiroideas y antitiroideas. En Goodman LS, Gilman A (eds): Las Bases farmacológicas de la terapéutica. Editorial Panamericana. México, 1982: 1376.

5. - Feliciano DV. Everything you wanted to know about Graves’ disease. Am J Surg 1992, (164): 404. 38.- Se trata de masculino de 70 años que consulta por pirosis, disfagia leve ocasional y episodios de regurgitación nocturna desde hace 15 días. Comenta que desde hace 2 años viene presentando por ocasiones pirosis y regurgitación. Se reporta endoscopia alta con esofagitis erosiva grave. ¿Qué tratamiento farmacológico entre los siguientes, es el más adecuado? a) Antagonistas de los receptores H2. b) Sucralfato. c) Inhibidores de la bomba de protones. d) Tratamiento combinado con anti-H2 y sucralfato.

Se ha demostrado que existe una relación directa entre la duración de la supresión del acido gástrico y la menor acidez del esófago. Los inhibidores de la bomba de protones (IBP) son los agentes preferidos para la curación de las lesiones agudas y para la mantención de remisión. Los bloqueadores H2 presentan como única ventaja su rapidez de acción, pero su potencia es menor. Su uso de rutina asociado con los IBP no se recomienda, ya que los IBP actúan sobre las bombas de ácido activadas y cualquier inhibición de la secreción por otro agente retarda su máximo efecto Los antiácidos son útiles para síntomas ocasionales y su uso no se contrapone con los IBP. De no existir factores significativos modificables en los hábitos o en la anatomía, la ERGE debe considerarse frecuentemente una patología crónica. No existe evidencia de peso que contraindique el uso crónico de IBP. Los pacientes con esofagitis erosiva son los más susceptibles a desarrollar complicaciones que los en GERD (endoscopy negative

Page 60: Examen Subespecialidades

Gastroesophageal reflux disease).Todos los IBP son útiles pero no son necesariamente iguales. Puede haber diferencias étnicas en el número de células parietales o polimorfismo del citocromo p450 con diferente metabolización de los IBP. La infección por Helicobacter pylori confunde cualquier intento de comparar IBP. La gastritis de predominio antral puede producir hipergastrinemia e hipersecreción. Si es de predominio corporal puede disminuir la producción de ácido. Los estudios comparativos deben hacerse en individuos sin esta infección. La duración del efecto de los IBP es importante. Muchas publicaciones demuestran la mayor duración de la acción de la acción del esomeprazol, pero clínicamente doble dosis de los otros IBP pueden tener resultados comparables. Katz y cols compararon los 5 IBP disponibles en pacientes con ERGE, todos resultaron en pH > 4 por al menos 8 a 10 h, pero la duración de acción del esomeprazol fue superior en el rango de los pH intragástricos entre 2 a 6. En los pacientes con enGERD el efecto de los IBP no es tan dramático, puede deberse a que no hay tantos elementos objetivos de juicio como la curación de las erosiones. Los IBP pueden usarse una vez al día, en la mañana y antes del desayuno. Los pacientes con daño extraesofágico o cuadros severos obtienen mejor efecto con doble dosis fraccionada (Antes de desayuno y cena). Esto mejora el control de la acidez nocturna. En los pacientes cuya pHmetría demuestre que persiste escape nocturno algunos favorecen agregar un antagonista H2 al acostarse. Rackoff y cols demostraron que 74% de los pacientes presentaban mejoría de los síntomas nocturnos. Una nueva formulación de liberación y absorción rápida de omeprazol también podría ser útil en estos casos. La ausencia de síntomas no significa ausencia de daño. Los casos graves (y por supuesto el Barrett) deben tener seguimiento endoscópico para asegurar su evolución. Un porcentaje significativo de pacientes sin Barrett continúan presentando RGE patológico y bajo pH intragástrico a pesar de IBP bien llevado con total remisión de síntomas. El primer objetivo es la remisión de los síntomas de reflujo clásico. Esto debe lograrse en la primera semana si la prescripción es adecuada. Las manifestaciones extraesofágicas tardan varios meses en controlarse, pero deben exhibir una mejoría rápida en las primeras semanas que confirme la buena orientación de las medidas indicadas. La estrategia terapéutica de comenzar el tratamiento en forma poco agresiva y escalarlo si no hay respuesta, me parece poco adecuada. Al paciente se le debe insistir que los fármacos son una parte importante del tratamiento pero no la única y que los cambios de hábitos y costumbres, la baja de peso, el ejercicio Etc, son claves para el éxito. Bibliografía: 1.- Miner P, Katz P, Chen Y, Sostek M. Gastric acid control with esomeprazole, lansoprazole, omeprazole, pantoprazole, and rabeprazole: A five-way crossover study. Am J Gastroenterol 2003; 98: 2616-20.

Page 61: Examen Subespecialidades

2. Katz P, Miner P, Chen Y, Sostek M. Effects of 5 marketed proton pump inhibitors on acid suppression relative to a range of pH thresholds. Am J Gastroenterol 2004; 99: S34. 3.- Rackoff A, Agrawal A, Hila A, et al. Histamine-2 receptor antagonists at night improve GERD symptoms for patients on proton pump inhibitor therapy. Am J Gastroenterol 2004; 99: S18. 4.- Castell D, Goldlust B, Morelli G, et al. Omeprazole immediate-release oral suspension is more effective than pantoprazole delayed-release capsules in reducing nighttime gastric acidity in GERD patients. Am J Gastroenterol 2004; 99: S39. 5. Milkes D, Gerson L, Triadafilopoulos G. Complete elimination of reflux symptoms does not guarantee normalization of intraesophageal and intragastric pH in patients with gastroesophageal reflux disease(GERD). Am J Gastroenterol 2004; 99: 991-6. 39.- Se trata de paciente masculino de 72 años que cursa con evento vascular cerebral isquémico ¿El tratamiento de elección en este padecimiento es?

a) Antiagregantes, hemorreologicos, anticonvulsivantes. b) Vasodilatadores, anticonvulsivantes, derivados sanguíneos. c) Medidas generales, hipoglucemiantes, analgésicos. d) Vasodilatadores, analgésicos, sedación.

Tratamiento:

Fase aguda ataque isquémico menor * Estabilización general - 02 por puntas nasales - Control de la TA y alteraciones cardiacas (FA) - Correción de variantes metabólicos (ES, glucosa, etc.) * Con o sin crisis convulsivas - DFH 750 mg. IV impregnación con 125 mg. IV c/8 hr. - Carbamacepina 200 mg. VO cada 8 hrs * Mejorar estado circulatorio - Pentoxifilina 400 VO c/ 8 hrs - ASA 125 mg VO c/ 24 hrs Atacar causa subyacente: - Control adecuado de la TA y la glicemia - Control de hiperlipidemia e hipercolesterolemia - Rehabilitación temprana - Eliminar tabaquismo

Page 62: Examen Subespecialidades

* Vigilancia tomográfica de la evolución * Limitar el daño y prevenir su extensión * Educación al paciente y a la familia 1.Aronson A y cols. Examen clínico neurológico, 3ª Edición. La Prensa Médica Mexicana, México, 1995. 2. Uribe CS, Arana A, Pombo PL. Neurología, 5ª Edición. Corporación para investigaciones biológicas. Colombia, 1996. 3.-Adams R, Víctor M. Principles of Neurology. Mc Graw Hill. 7th ed. USA 2001. 4.-Bradley W.G. Neurology in clinical practice. Butterworth Heinemann. 4th ed. 40.- Se trata de femenino de 31 años, con un cuadro clínico de sangrado menstrual irregular, dismenorrea, dispareunia, y una esterilidad de 3 años de evolución, con resultados de laboratorio hormonal normal y un estudio ecográfico transvaginal que informa de un útero normal y sendas formaciones quística ováricas bilaterales de 4 cms. con signos ecográficos de sospecha. ¿Cuál sería la orientación diagnóstica? a) Hemorragia uterina disfuncional. b) Síndrome del ovario poliquístico. c) Endometriosis. d) Quistes dermoides bilaterales.

Causas y síntomas de endometriosis:

Las causas de la endometriosis aún no se conocen. Las células del revestimiento interno del útero de alguna manera se desplazan hasta zonas externas al mismo y siguen creciendo. Este desplazamiento podría quizás deberse a que pequeños fragmentos del revestimiento uterino, desprendidos durante la menstruación, retrocedan hacia las trompas de Falopio en dirección a los ovarios hasta entrar

Page 63: Examen Subespecialidades

en la cavidad abdominal, en lugar de salir con el flujo menstrual a través de la vagina. La endometriosis causa dolor en la parte inferior del abdomen y la zona pélvica, irregularidades menstruales (como manchar antes de la menstruación) e infertilidad. Algunas mujeres con endometriosis grave no presentan síntomas, mientras que otras con la enfermedad en grado mínimo sufren un dolor invalidante. Con frecuencia, el dolor menstrual debido a la endometriosis no aparece hasta años después de desarrollar la enfermedad. En algunos casos, se constata dolor durante el coito (dispareunia), antes o durante la menstruación. El tejido endometrial adherido al intestino grueso o a la vejiga urinaria puede provocar hinchazón abdominal, dolor durante las deposiciones, hemorragia rectal durante la menstruación o dolor en la parte inferior del abdomen durante la micción. Así mismo, cuando el tejido se localiza en un ovario o una estructura cercana puede dar lugar a la formación de una masa llena de sangre (endometrioma). En ocasiones, el endometrioma se rompe bruscamente o se escapa algo de su contenido, lo que causa un agudo y repentino dolor abdominal. 41.- Masculino de 40 años recién llegado del estado de Chiapas, Méx. Acude a consulta por referir presentar fiebre elevada de tres días de tres días de evolución refiere artralgias francas y cefalea intensa. El mismo día de su visita a urgencias comenzó a presentar un exantema maculo-papuloso pruriginoso. El examen de la sangre mostró los siguientes datos: Valor hematocrito 38%, Leucocitos 3600 p.mm3 con 82% neutrófilos y 12% linfocitos. Plaquetas 115.000 p.mm3. Placa de tórax normal. El paciente fue dado de alta con el diagnóstico de cuadro viral, en tratamiento con paracetamol y antihistamínicos para el picor. A las 48 horas volvió a urgencias con muy mal aspecto: estaba afebril, tenía confusión mental, se observaban petequias en antebrazos y piernas, edema en pies, TA 85/70. Pulso 110 l.p.min. de amplitud pequeño. En los nuevos exámenes de la sangre destacaban: Valor hematocrito 46%, leucocitos 3600 p.mm3 sin cambios en la fórmula y plaquetas 65.000 p.mm3. Glucosa 106 mg.p. dl. Creatinina 1,8 mg.p.dl. Sodio 126 mEq/l. Potasio 4,2 mEq/l. La placa de tórax mostraba un pequeño derrame pleural bilateral. ¿Cuál es, entre los siguientes, el diagnóstico más probable? a) Dengue. b) Meningoencefalitis bacteriana. c) Fiebre tifoidea. d) Neumonía por Legionella Neumophila.

Page 64: Examen Subespecialidades

El dengue es actualmente la más importante arbovirosis que afecta al hombre. Su agente etiológico son los 4 serotipos del virus del dengue (D1-4). Se transmite entre humanos mediante la picadura del mosquito Aedes aegypti. Se estima que el 40% de la población mundial vive en áreas de riesgo de esta entidad (1,2).

La forma clínica más grave, la fiebre hemorrágica de dengue/síndrome de choque del dengue (FHD/SCD), fue casi exclusiva del Sudeste Asiático y el Pacífico Occidental hasta 1981, cuando una gran epidemia de dengue, la mayor de las Américas, que incluyó más de 10.000 casos de FDH/SCD, ocurrió en Cuba. Esta constituyó la primera epidemia de DH en la región (3). Posterior a este brote y hasta la actualidad la FHD continúa presentándose de forma endémica en diferentes países, ocurriendo epidemias de forma frecuente (4).

Cuadro clínico

El curso de la enfermedad, desde el punto de vista clínico, se puede dividir en cuatro fases: Inicial, crítica, de recuperación y de convalecencia.

Fase inicial: En ella el enfermo tiene un síndrome febril sin localización. Es muy

sintomática y además de la fiebre predominan las manifestaciones generales como cefalea, artromialgias, dolor retroocular y malestar, puede aparecer rash.

Suele durar alrededor de 3 días, tras lo cual la fiebre cede y algunos casos de dengue

clásico comienzan a presentar manifestaciones hemorrágicas leves, pero la mayoría tienden a mejorar. Otro grupo menor de pacientes desarrollará la FHD/SCD. Los llamados signos de

alarma que preceden al choque pueden comenzar en esta fase.

Page 65: Examen Subespecialidades

Fase crítica: Transcurre entre el 4to y 7mo día de la enfermedad, en ella se presentan los síntomas que definen al DH, como son la extravasación de plasma, la trombocitopenia, y las manifestaciones hemorrágicas. Algunos pacientes desarrollan el síndrome de choque por dengue que es la forma más severa de la enfermedad. Las manifestaciones hemorrágicas

pueden ser tan leves que solo se hacen evidentes a través de la prueba del torniquete, o tan severas como sangramientos digestivos graves con compromiso hemodinámico.

Los signos de extravasación de líquidos (derrame en serosas, hemoconcentración) son

indispensables para hablar de FHD/SCD, ya que es la fuga de líquido la que casi siempre lleva el paciente al choque y no las hemorragias. Este hecho es de gran importancia pues influenciado por el nombre de fiebre hemorrágica de dengue, en no pocas ocasiones los

médicos asistentes esperan las grandes hemorragias que nunca llegan y el paciente cae en choque por la extravasación de plasma.

Fase de recuperación: Se inicia cuando cesa el escape de líquido y las manifestaciones de sangrado comienzan a disminuir. Pueden aparecer edemas o agravarse los derrames serosos

producto de la sobre hidratación. También se puede apreciar un rash tardío asociado a prurito intenso. Se recuperan el apetito y el número de plaquetas.

Fase de convalecencia: Puede prolongarse hasta más de 6 meses, se caracteriza por cefalea discreta, cansancio fácil y artromialgias. No se presenta en la totalidad de los

casos.

Resumimos los signos de alarma de la siguiente manera: 1. Signos clínicos tempranos del choque. · Caída brusca de la fiebre. · Dolor abdominal intenso. · Irritabilidad, somnolencia u otras alteraciones mentales. · Fatiga extrema. · Lipotimias. · Dolor torácico. 2. Signos clínicos que agravan el choque. · Vómitos frecuentes. · Diarreas frecuentes. 3. Signos de laboratorio e imagenológicos.

Page 66: Examen Subespecialidades

· Aumento progresivo del hematócrito y disminución progresiva del conteo de plaquetas. · Engrosamiento de la pared de la vesícula biliar.

Referencias: 1. Guzman MG, Kouri G. Dengue-an update. The Lancet Inf Dis 2002;2:33-42. 2. Gubler DJ, Clark CG. Dengue/dengue hemorrhagic fever: the emergence of a global health problem. Emerging Infectious Diseases. Atlanta USA. 1995;1:55-57. 3. Kourí GP, Guzmán MG, Bravo JR, Triana C. Dengue haemorrhagic fever/ dengue shock syndrome: lessons from the Cuban epidemic. 1981. Bull World Health Organ 1989;87:375-80. 4. Organización Panamericana de la Salud. 2003: Number of reported cases of dengue and dengue hemorrhagic fever (DHF), region of the Americas (by country and subregion). [citado del 25 de agosto de 2003]. 5. Valdes L, Guzman MG, Kouri G, Delgado J, Carbonell I, Cabrera MV, Rosario D, Vazquez S. La Epidemiología del Dengue en Cuba en 1997. Rev Panam Salud Publica/Pan American Journal of Public Health 1999;6:16-25. 6. Pelaez O et al. Havana dengue 3 epidemic, 2001. Enviada a Emerging Infection Diseases. 7. George R, Lum LCS. Clinical spectrum of dengue infection. In: Gubler DJ, Kuno G, eds. Dengue and dengue hemorrhagic fever. London: CAB International, UK, 1997; 89-113. 8. Kourí G, Guzmán MG, Bravo J. Why dengue haemorrhagic fever in Cuba? 2: An integral analysis. Trans R Soc Trop Med Hyg 1987;81:821-23. 42.- Paciente de 4 años de edad tiene lesiones eczematosas crónicas en flexuras de brazos y piernas que producen intenso picor, asociadas a una queilitis descamativa de labios. ¿Cuál, entre los siguientes, es el diagnóstico más probable? a) Una dermatitis atópica. b) Un eczema seborréico. c) Un prúrigo nodular. d) Un eczema microbiano.

Page 67: Examen Subespecialidades

La dermatitis atópica llamada comúnmente eccema (atópico), es una enfermedad que consiste en un estado reaccional de la piel caracterizado por erupciones pruriginosas y con aspecto de escamas, más frecuente en niños, multifactorial, en la cual intervienen factores tanto ambientales como constitucionales. Las personas con eccema a menudo tienen antecedentes de condiciones alérgicas como asma, fiebre del heno o eccema. La dermatitis atópica fue originalmente conocida como prúrigo de Besnier y eccema constitucional, actualmente también es llamada neurodermatitis diseminada, por las escuelas europeas.

Clínica Las manifestaciones clínicas típicas de la dermatitis atópica se dividen en tres etapas, que suelen denominarse del lactante, infantil y del adulto. Junto a ellas se encuentran otras, con frecuencia llamadas atípicas, a pesar de que muchas, como la xerosis, son muy constantes. Dermatitis atópica del lactante: Suele empezar hacia los cinco meses de vida, pero puede hacerlo antes. Algunos niños desarrollan lesiones de eccema seborreico, que de forma gradual va adquiriendo el aspecto de la dermatitis o eccema atópico. La localización más habitual es en la cara, respetando las zonas alrededor de los ojos, la nariz y la boca (. Son también frecuentes en el cuero cabelludo, las orejas, el dorso de las manos y las zonas de extensión de las extremidades. Las lesiones suelen ser pápulas o placas eritematosas y edematosas, muchas veces con erosiones, exudación y costras. Es muy raro que se aprecien las vesículas características del eccema. El prurito es un síntoma constante. Dermatitis atópica infantil Este periodo suele considerarse con un inicio hacia los dos años y un final entre los siete años y la pubertad. Las lesiones características se observan sobre todo en las flexuras, en especial en los codos y las rodillas (Fig. ), pero pueden aparecer en otras zonas. En esta fase es más fácil ver lesiones eccematosas con vesículas, pero el intenso prurito hace que enseguida se transformen en erosiones, con exudación y formación de costras. Bibliografía:

Page 68: Examen Subespecialidades

1. Bielsa Marsol I. Eccemas (II). En: Ferrándiz C, ed. Dermatología Clínica. Madrid, Mosby/ Doyma Libros 1996, 113-124. 2. Fernández Vozmediano JM y cols. Dermatitis atópica. Madrid, Jarpyo 1994. 3. Fernández Vozmediano JM, Armario Hita JC. Tacrolimus. Piel 2001;16:48-54. 4. Fonseca Capdevila E. Dermatitis atópica. Protocolo terapéutico. (En línea) (14.02.2001). Disponible en www.especialistasdermatologia.com. 5. Fonseca E. Dermatitis atópica en la infancia. Salud Rural 1997;14:92-105. 6. Guerra Tapia A. Dermatitis atópica. En: Fonseca Capdevila E, ed. Dermatología Pediátrica. Madrid, Aula Médica 1999, 83-180. 43.- Un hombre de 55 años acude a consulta por una historia de 2 meses de dificultad para tragar. En un principio, la dificultad era únicamente con bocados grandes de alimentos sólidos, pero ahora tiene problema hasta con los líquidos. Tiene una historia de uso de alcohol y tabaco. La esofagoscopía demuestra una masa polipoide grande e irregular que ocluye casi completamente el tercio superior del esófago. ¿Cuál de los siguientes es el tipo histológico de este tumor? a) Linfoma de células gigantes b) Carcinoma de células pequeñas c) Linfoma de células pequeñas d) Carcinoma de células escamosas Hay varios subtipos, principalmente adenocarcinoma (aproximadamente 50-80% de todos los cánceres del esófago) y el cáncer de células escamosas. El cáncer de células escamosas surge de las células que recubren la parte superior del esófago. El adenocarcinoma se deriva de las células glandulares que están presentes en la unión del esófago y el estómago. Los tumores de esófago por lo general llevan a la disfagia (dificultad para tragar), dolor. Clasificación:

Page 69: Examen Subespecialidades

Los cánceres de esófago son generalmente carcinomas que surgen a partir del epitelio o revestimiento de superficie, del esófago. La mayoría de los cánceres del esófago caen en una de dos clases: Los carcinomas de células escamosas, que son similares a cáncer de cabeza y cuello en su apariencia y su asociación con el tabaco y el consumo de alcohol, y los adenocarcinomas, que se asocian a menudo con antecedentes de enfermedad por reflujo gastroesofágico y esófago de Barrett. Una regla del pulgar general es que un cáncer en la parte superior de dos tercios es un carcinoma de células escamosas y uno en el tercio inferior es un adenocarcinoma. REVISTA ESPAÑOLA DE PATOLOGÍA Vol. 37, n. º 4, 2004 Protocolo e información sistematizada para los estudios histopatológicos relacionados con el carcinoma esofágico Francisco Colina, Guadalupe López Alonso, Carolina Ibarrola. 44.- Masculino de 7 meses de edad que inicia con rechazo al alimento, irritabilidad, llanto constante, inquietud, delirio, convulsiones y estado epiléptico los datos clínicos del menor son compatibles con intoxicación por: a) Capulín tullidor. b) Té de anís estrella. c) Papaver somniferum d) Picadura de alacrán. La intoxicación se presenta en forma crónica. Más frecuente en recién nacidos y lactantes. La sintomatología por intoxicación de té de anís de estrella se caracteriza por el siguiente cuadro clínico.

Gran irritabilidad Llanto constante Inquietud Delirio Convulsiones Rechazo al alimento Estado epiléptico

El tratamiento consiste en:

Page 70: Examen Subespecialidades

Estabilizar signos vitales Ayuno Soluciones parenterales Anticonvulsivos:

Benzodiacepinas ó barbitúricos Estabilizar signos vitales

Ayuno Soluciones parenterales Anticonvulsivos:

Benzodiacepinas ó barbitúricos • 1.- Montoya-Cabrera MA. Intoxicaciones y envenenamientos en niños. México,

Intersistemas, 2000. 2.-Montoya CMA. Toxicología clínica. 2ª. Ed., México, Méndez Editores. 45.- Su frecuencia es de 1/8,000 bajo peso al nacer, occipucio prominente, micrognatia, puños cerrados con cabalgamiento característico de los dedos, pies en mecedora y Divertículo de Meckel:

a) Trisomía 13 b) Trisomía 18 c) Trisomía 8 d) Trisomía 11 TRISOMIA 18 (SINDROME DE EDWARD) Tiene una incidencia de 1/8000 recién nacidos vivos con predominio para el sexo femenino. Es la trisomia más común entre los recién nacidos muertos con malformaciones. Manifestaciones Clínicas: En mas del 50% de los casos presenta: Retraso mental y pondoestatural, hipertonia, occipucio prominente, orejas bajas y malformadas, micrognatia, cuello corto, implantación distal y retroflexión del pulgar, hipoplasia de uñas, defectos articulares de los dedos, dedos de manos “montados”, pie equinovaro, calcaneo prominente (pie en mecedora), abducción limitada de caderas, esternón corto, cardiopatia y criptorquidea. En menos del 50% de los casos se acompaña de hipotonia, epicantus,

Page 71: Examen Subespecialidades

microftalmia, defectos oculares, fisura labial y palatina, hemangiomas capilares, polidactilia, sindactilia, hernias y malformaciones renales. Desde el punto de vista anestésico se debe predecir una difícil intubación y tener especial precaución por las drogas con excreción renal. El pronóstico es muy grave ya que la supervivencia es corta, por el conjunto de malformaciones viscerales graves asociadas ; el 50% fallecen antes de los dos meses, y el 75% antes de los tres meses. En las supervivencias prolongadas se pone de manifiesto el intenso retraso psicomotor y los defectos esqueléticos posturales. En los casos de trisomías parciales por lo general la clínica se atenúa y la supervivencia es prolongada. BADGWELL J : Clinical pediatric anesthesia. Common and uncommon coexisting diseases.

Philadelphia. 399-401, 1997. CRUZ M : Tratado de pediatria. Cromosopatias. Barcelona. 7a edición, 305-314, 1994. MENEGHELLO J : Pediatria. Fundamentos de genetica clinica en pediatria. Buenos aires, 4a edicion, 2004-2005, 1991. HARLEY EH : Neurologic sequelae secondary to atlantoaxial instability in down syndrome. Arch otolaringology, 120(2), 159-165, 1994. 46.- Paciente femenino de 10 años de edad que la llevan sus padres por conducta “rara”. Actualmente se está recuperando de un cuadro diarreico. No presenta rigidez de nuca, náusea, vómito u otra alteración. Se presenta combativa, hiperrefléxica, midriática, mucosas secas y rubicunda. Signos vitales: FC 140 por min, FR 20 por min, TA 110/70 mmHg, Temp. 40°C, peso 35 Kg. ¿Esta paciente presenta un toxíndrome, cual sería? a) Intoxicación por organofosforados. b) Intoxicación por hongos. c) Intoxicación por cáusticos. d) Intoxicación por anticolinérgicos.

ANTICOLINÉRGICOS

El grupo de fármacos de esta clase actúan en el nivel uno es decir a nivel del órgano y son los llamados fármacos anticolinérgicos o antimuscarínicos y a estos fármacos se los define como aquellos que bloquean la acción muscarínica de la Acetilcolina (Ach) es decir son los que inhiben la función del sistema parasimpático.

Page 72: Examen Subespecialidades

ANTICOLINÉRGICOS

Atropina

Antidepresivos tricíclicos

Fenotiacidas

Antihistamínicos

Hongos

Agitación

Alucinaciones

Midriasis

Boca Seca

Taquicardia, arritmias

Retención urinaria

Calor

Piel seca

Movs. Extrapiramidales

47.-Un hombre de 29 años con historia de actividad homosexual hace 5 años y HIV-1 positivo con tratamiento con HAART , el cual hace 8 meses suspende el medicamento por sentirse bien y que ha presentado hace dos meses una neumonía por neumocictis carinii , en caso de presentar una infección del SNC por papovavirus tiene un riesgo de producir : a) Adrenoleucodistrofia. b) Esclerosis múltiple. c) Panencefalitis subaguda esclerosante. (SSPE) d) Leucoencefalopatia multifocal progresiva. (PML)

La leucoencefalopatía multifocal progresiva (LMP) es una enfermedad de etiología viral (virus JC, Papovavirus), que afecta de forma subaguda o crónica la sustancia encefálica provocando una desmielinización progresiva de ésta. Se observa en pacientes jóvenes, portadores de enfermedades inmunosupresoras (infección por VIH etapas finales, enfermedades linfoproliferativas, tratamientos inmunosupresores, etcétera).

Page 73: Examen Subespecialidades

El diagnóstico de LMP se plantea ante un paciente inmunodeprimido, que presenta un cuadro de deterioro cognitivo o déficit focales (hemiparesia, alteraciones campimétricas, alteraciones de la coordinación y del equilibrio, etcétera), o ambos, de curso progresivo, en cuestión de semanas o pocos meses, infrecuentemente acompañado de cefaleas y sin fiebre. Este cuadro lleva inexorablemente a la muerte, aunque se han reportado casos anecdóticos de detención y reversibilidad de la enfermedad (7). La tomografía computarizada (TC) encefálica puede ser normal al inicio de la enfermedad, o mostrar lesiones hipodensas de la sustancia blanca, a menudo confluentes, más frecuentemente ubicadas en las regiones frontales y parietooccipitales que no se realzan con el contraste, no producen efecto de masa y respetan la sustancia gris cortical. Leucoencefalopatía multifocal progresiva Rev Med Uruguay 2003; 19: 78-82. Bibliografía 1 . Corradi H. Leucoencefalopatía multifocal progresiva. In: Salamano R (coord). Temas de Neuroinfectología. Montevideo: Oficina del Libro-AEM, 1998: 105-8. 2 . Astrom KE, Mancall EL, Richardson EP Jr. Progressive Multifocal Encephalopathy. Brain 1958; 81: 930. 3 . Cavanagh JB, Greenbaum D, Marshall AHE, et al. Cerebral Demyelination associated with disorders of reticuloendothelial system. Lancet 1959; 2: 525. 4 . Richardson EP Jr. Progressive Multifocal Leukoencephalopathy. N Engl J Med 1961; 265: 815. 5 . Padgett BL, Walker DL, Zu Rheim GM, Eckroade RJ, Dessel BH. Cultivation of papova-like virus from human brain with progressive multifocal leucoencephalopathy. Lancet 1971; 1: 1257-60. 48. Acude a consulta un hombre de 35 años de edad, soltero, previamente sano. Sus familiares refieren que en los 5 días previos ha presentado fiebre, cefalea, cambios del estado de ánimo, y en el último día, somnolencia que se ha convertido en estupor. A la exploración física no encuentra alteraciones. Realiza una punción lumbar y encuentra como única alteración del líquido cerebroespinal la presencia de eritrocitos. Se solicitó una resonancia magnética de cráneo. Usted esperaría encontrar para apoyar su diagnóstico: a) Afección de ganglios basales b) Afección de lóbulo frontal

Page 74: Examen Subespecialidades

c) Afección de lóbulo temporal d) Lesión intraaxial ocupativa en puente. El cuadro clínico y los hallazgos del líquido cerebroespinal sugieren fuertemente infección por virus del herpes simple. Los cambios en el estudio de imagen más característicos, sobre todo en resonancia magnética, se observan en los lóbulos temporales. Kasper DL, Braunwald E, Fauci AS, Hauser SL, Longo DL, Jameson JL. Harrison´s Principles of Internal Medicine. McGraw Hill. 16 Ed. 2471-2490 p. 49.- Paciente femenino de 5 años quien llega al servicio de urgencias por presentar fiebre, cefalea, náusea, vómito y variabilidad en el estado de despierto. A la exploración física confirma la fiebre en 39°C, FC 110x´, y presenta lesiones purpúricas en extremidades. Usted sospecha meningitis por meningococo y hace una punción lumbar. Cuál de las siguientes opciones sería compatible con su diagnóstico: a) Aspecto de agua de roca, glucorraquía normal y celularidad aumentada con predominio

de linfocitos. b) Aspecto de agua de roca, glucorraquía normal, celularidad aumentada con predominio de

linfocitos y algunas células de aspecto neoplásico. c) Aspecto turbio, glucorraquía normal y celularidad aumentada con predominio de

polimorfonucleares. d) Aspecto turbio, glucorraquía disminuída y celularidad aumentada con predominio de

polimorfonucleares. El cuadro clínico es compatible con una meningitis bacteriana. La primera opción es más compatible con meningitis aséptica, de origen viral, la segunda con una meningitis neoplásica, la tercera son poco probables encontrarlas en la realidad. Kasper DL, Braunwald E, Fauci AS, Hauser SL, Longo DL, Jameson JL. Harrison´s Principles of Internal Medicine. McGraw Hill. 16 Ed. 2471-2490 p. 50.- Durante su servicio social le llevan en la noche a un paciente de 3 días de nacido, cuyo parto fue atendido en el domicilio de su madre por una partera empírica. El niño ha rechazado la alimentación al seno materno en las últimas 24 horas, y se ha mostrado hipoactivo. Al revisarlo encuentra que presenta la fontanela con tensión aumentada y fiebre. Se le pasa por la mente el diagnóstico de meningitis. Uno de los microorganismos más probables es: a) Meningococo. b) Neumococo. c) Estreptococo del grupo B (S agalactiae) d) Listeria monocytogenes

Page 75: Examen Subespecialidades

Los microorganismos más frecuentes en este grupo de edad son el estreptococo del grupo B y la Escherichia coli, con los que se colonizan algunos recién nacidos por su paso a través del canal del parto. Listeria aparece en pacientes de mayor edad, lo mismo que meningo y neumococo. Kasper DL, Braunwald E, Fauci AS, Hauser SL, Longo DL, Jameson JL. Harrison´s Principles of Internal Medicine. McGraw Hill. 16 Ed. 2471-2490 p.

51.- Femenino de 62 años refiere hace unos días dolor brusco en espalda tras un intento de levantar un bulto de aproximados 15 kg. Menopausia a los 50 años. Fumadora. Exploración física normal, salvo dolor a la percusión sobre apófisis espinosas de D5-D6. Calcemia en límite normal alto; fósforo, fosfatasa alcalina y niveles de vitamina D normales. Urea y creatinina en límite superior de la normalidad. Hb 11 g/dl. Proteínas totales 9 g/dl con albúmina 3,8 g/dl. VSG de 90 en primera hora. Proteinuria de 1,2 g (24 horas). Radiología lateral de columna dorsal: acuñamiento posterior de D6, sugerente de osteoporosis generalizada. La paciente tendrá más probablemente: a) Osteoporosis postmenopáusica. b) Osteomalacia. c) Mieloma múltiple. d) Hiperparatiroidismo. Definición OMS: Tumor maligno, que habitualmente muestra compromiso óseo difuso o múltiple, y que se caracteriza por la presencia de células redondas del tipo de las células plasmáticas pero con diversos grados de inmadurez, incluyendo formas atípicas. Presentación clínica: Más frecuente en los hombres 2:1. El 75% de los casos entre 50 y 70 años de edad. Se localiza, preferentemente, en los huesos donde existe médula ósea roja: vértebras, costillas, esternón, pelvis, cráneo y huesos largos (tercio proximal fémur y húmero). Aproximadamente el 80% de pacientes tienen como queja principal el dolor óseo con sensibilidad difusa de hueso, particularmente sobre el esternón, pelvis y columna vertebral. Posteriormente pueden aparecer síntomas radiculares. La sensación de tumor es menos frecuente, ya que a veces los nódulos mielomatosos son pequeños o asientan en huesos profundos, a la palpación suele ser de dos a tres cm. de diámetro El crecimiento de células plasmáticas interfiere con la producción de glóbulos rojos, glóbulos blancos y plaquetas, por lo que se desarrolla anemia, susceptibilidad a infecciones y tendencia al sangrado. También pueden parecer síntomas constitucionales, fiebre, anemia, trombocitopenia, y fracaso renal. Las fracturas patológicas de columna o fémur pueden ser el primer síntoma. Se producen con un traumatismo mínimo o incluso sin trauma. El rango de duración de los síntomas es tan corto como pocas semanas hasta más de 2 años

Page 76: Examen Subespecialidades

Mieloma Múltiple Mieloma múltiple en estadio I

En estadio I se presenta con:

1. Estructura ósea normal. 2. Calcio sérico normal.

3. Hemoglobina superior a 10 g/dL. 4. Producción baja de proteína M indicada por:

a. IgG inferior a 5.0 g/dL b. IgA inferior a 3.0 g/dL

c. kappa o lambda de la orina inferior a 4 g/24 horas.

Masa estimada de las células del mieloma: inferior a 0.6 billones de células por metro cuadrado (carga baja)

Se usa la siguiente subclasificación de estadios:

A. creatinina inferior a 2.0 mg/dL B. creatinina superior o igual a 2.0 mg/dL

La insuficiencia renal empeora el pronóstico independientemente del estadio. Mieloma múltiple en estadio II

En este estadio II el mieloma múltiple se presenta con:

Masa estimada de células del mieloma: 0.6 a 1.2 billones de células por metro cuadrado (carga intermedia)

Se usa la siguiente subclasificación de estadios:

A. creatinina inferior a 2.0 mg/dL B. creatinina superior o igual a 2.0 mg/dL

La insuficiencia renal empeora el pronóstico independientemente del estadio. Mieloma múltiple en estadio III

En estadio III significa que existen uno o más de los siguientes:

1. Hemoglobina inferior a 8.5 g/dL 2. Calcio sérico superior a 12.0 mg/dL

3. Más de 3 lesiones osteolíticas.

Page 77: Examen Subespecialidades

4. Producción alta de proteína M indicada por:

a. IgG superior a 7.0 g/dL b. IgA superior a 5.0 g/dL

c. kappa o lambda de la orina superior a 12.0 g/24 horas

Masa estimada de células del mieloma: mayor de 1.2 billones de células por metro cuadrado (carga alta)

Se usa la siguiente subclasificación de estadios:

A. creatinina inferior a 2.0 mg/dL B. creatinina superior o igual a 2.0 mg/dL

La microglobulina beta-2 sérica ha mostrado ser un indicador confiable para el pronóstico. Puesto que la gran mayoría de pacientes con mieloma sintomáticos son clasificados como

estadio III por los criterios de Durie/Salmon, este sistema de clasificación no ha probado ser muy útil para identificar a los pacientes con pronóstico precario e intermedio. Muchos

investigadores están a favor de un sistema más simple que utilice solamente las concentraciones de microglobulina beta-2 y albúmina para determinar la estadio de los

pacientes. La insuficiencia renal empeora el pronóstico independientemente de la estadio. Las anormalidades de los cromosomas 13 y 11q y la morfología plasmablástica parecen

haber pronosticado un resultado precario en una población de pacientes cuyo mieloma fue tratado con quimioterapia de dosis elevadas y rescate con células madre.

De National Cancer Institute Solitary plasmacytoma of bone and extramedullary plasmacytomas: A clinicopathologic and immunohistochemical study. JM Meis et al. Cancer. Vol 59. 1987. p 1475-1485; 52.- Una mujer de 54 años de edad es llevada a urgencias después de un accidente automovilístico. Al llegar, se encuentra respirando adecuadamente. Presenta múiltiples abrasiones en el tórax y múltiples sitios de dolor a la palpación sobre las costillas. LA RX muestra múltiples fracturas costales, pero el parénquima se observa sin alteraciones y ambos pulmones se encuentran expandidos. Dos días después, presenta dificultad respiratoria y sus pulmones se “emblanquecen” en la RX. ¿Cuál de los siguientes es el diagnóstico más probable? a) Contusión miocárdica b) Contusión pulmonar c) Ruptura traumática de la aorta d) Hemotórax

Page 78: Examen Subespecialidades

Diagnóstico Generalmente se lo hace con base en los datos clínicos, el conocimiento del mecanismo de la lesión y la confirmación mediante la radiografía de tórax y la Tc.

Datos clínicos.- No existen signos físicos específicos de contusión, los datos más típicos son taquipnea, si la contusión es severa y está presente un tórax inestable, o si hay lesiones asociadas como neumotórax, el distrés respiratorio puede ser extremo. La cianosis es rara excepto en casos de lesión de todo el tórax cuando una decoloración cianótica de cabeza y cuello frecuentemente en asociación con petequias es característico.3

Radiología.- La contusión pulmonar se muestra como un infiltrado pulmonar de densidad variable con bordes pobremente definidos en el área de máximo impacto. En el trauma contuso la densidad es periférica y tiende a disminuir hacia el hilio. (Fig. 4 Y 5) Estas lesiones típicamente aparecen 4 a 6 horas después del trauma, clásicamente empeoran en aproximadamente 24 horas y luego tienden a resolverse desapareciendo en 3 a 4 días, aunque la cicatrización posiblemente se complete a los 7 a 10 días.I-3-?

Existe una interrelación entre la extensión de las anormalidades vistas en la radiografía de tórax y la severidad de los datos clínicos. Cuando sobreviene neumonía, frecuentemente esta se desarrolla en el segmento que estuvo contuso y se superpone en el sitio lesionado de forma tal que el infiltrado nunca se aclara totalmente en la placa radiográfica. (Fig. 6) Las fracturas costales generalmente están presentes alrededor del área de contusión señalando el área de máximo impacto.

En el trauma penetrante la contusión corresponde a un cilindro de tejido lesionado alrededor del tracto de la herida o del tejido directamente lesionado cuando hay múltiples fragmentos. Estas imágenes pueden estar ligeramente alteradas debido a hemorragia dentro del trayecto de la herida.

Hemoneumotórax universalmente está presente, en contraste con el trauma contuso en donde a menudo está ausente.

Tanto en el trauma penetrante o contuso las lesiones no tienen bordes anatómicos definidos, en caso de presentarse, es probable que la lesión no corresponda precisamente a una contusión.

Fig. 5. Contusión pulmonar. Imagen obtenida de la misma paciente luego de 8h del trauma.

Page 79: Examen Subespecialidades

Nótese las fracturas costales alIado izquierdo y la imagen de condensación compatible con

contusión pulmonar.

Bibliografía l. Cohn SM. Contusion pulmonary. Review of the Clinical Entity. J Trauma 1997; 42:973-979. 2. Velmahos GC. Traumatismos contusos de tórax. En: Naude GP, Bongar FS, Demetriades D. Secretos del traumatismo. Editorial McGraw-Hill Interamericana. México DF-México 1999:97-101. 3. Lewis FR. Pulmonary Co~tusion. In: Callaham MI.. Current Therapy in Emergency Medicine. BC. Decker Inc. Toronto - Philadelphia. 1987: 123-126. 4. Thomson FG. Notes on penetrating chest wounds. BMJ 1940; 1 :44. 5. Wintermark M., Schnyder P. The Macklin Effect. A frequent Etiology forPneumomediastinum in Severe Blunt Chest Trauma. Chest 2001; 120:543-547. 6. Hooker DR. Physiological effects of air concussion. Am J Physiol 1924; 67:219 7. Moseley RV, Vemick JJ, Doty DE. Response to blunt chest injury: a new experimental model. J Trauma 1970; 10:673.

53.- A un lactante de 8 meses le fue administrada metoclopramida en forma repetida por haber presentado vómitos es llevado al servicio de urgencias porque presenta irritabilidad, espasticidad y opistotonos, es este momento se le debe de administrar: a) Bicarbonato de Sodio b) Difenhidramina c) Atropina d) Naloxona e) Diazepam METOCLOPRAMIDA: Es un medicamento con acciones colinérgicas y antidopaminérgicas. A nivel gastrointestinal, la metoclopramida incrementa el tono del esfínter esofágico inferior, mejora el tono gástrico, incrementa los movimientos peristálticos, mejora la coordinación antro-duodenal y relaja el esfínter pilórico. Gracias a estos efectos farmacológicos, la metoclopramida acelera el vaciamiento gástrico y disminuye el tiempo de tránsito intestinal a través del duodeno, yeyuno e íleon. Por otra parte, la metoclopramida es un fármaco que cruza la barrera hemato-encefálica y por consiguiente tiene acciones a nivel del sistema nervioso central, principalmente como antagonista a nivel de los receptores dopaminérgicos ATROPINA (ATROPINA®) Se utiliza en la intoxicación por agentes colinérgicos, los cuales actúan por inhibición de la acetilcolinesterasa. El acúmulo de acetilcolina resultante provoca diversos efectos que constituyen el síndrome colinérgico: muscarínico, nicotínico y central. Existen diversos agentes colinérgicos, entre los que se incluyen: insecticidas tipo carbamatos y organofosforados (OP) y agentes nerviosos OP. Mientras que los carbamatos se unen de forma reversible a la acetilcolinesterasa, la unión de los organofosforados se hace irreversible con el tiempo. La atropina, antagonista competitivo de la acetilcolina, revierte los efectos muscarínicos (salivación, lagrimeo, incontinencia urinaria, diarrea, sudoración, vómitos, broncorrea,

Page 80: Examen Subespecialidades

broncoconstricción, bradicardia y miosis); sin embargo, no contrarresta los efectos nicotínicos (debilidad muscular, fasciculaciones, parálisis, hipertensión arterial, taquicardia y midriasis), ni los centrales (delirio, convulsiones, coma). La sintomatología nicotínica de los OP se trata con pralidoxima (oxima), que reactiva la acetilcolinesterasa. En la intoxicación por carbamatos, la pralidoxima no está indicada, ya que la enzima se regenera de forma rápida y espontánea. Antídotos más utilizados en intoxicaciones pediátricas 55 Indicaciones: 1. Intoxicaciones sintomáticas por insecticidas y herbicidas (OP o carbamatos). 2. Intoxicaciones por OP incluidos en agentes nerviosos usados en guerras químicas (sarin, soman, tuban, VX). 3. Intoxicaciones por sustancias colinérgicas: setas, fisostigmina, metacolina, neostigmina, pilocarpina. 4. Conducción auriculoventricular alterada por: digital, beta-bloqueantes, antagonistas del calcio. Dosis iv/im: 0,05-0,1 mg/kg/dosis (mín. 0,1 mg, máx. 5 mg), seguida de dosis repetidas cada 2-10 minutos o en infusión continua (0,025 mg/kg/h), hasta que aparezcan los signos de atropinización: desaparición de la broncorrea y del broncoespasmo, taquicardia, piel seca y rubicunda. La midriasis no es un signo confiable de atropinización, ya que su aparición no es constante. Pueden necesitarse varias horas de tratamiento. Se utilizan dosis más elevadas en las intoxicaciones más graves. Efectos secundarios: midriasis, taquicardia, fiebre, visión borrosa, sequedad de boca, íleo, retención urinaria. Además, si se administra vía iv, puede provocar arritmias en el paciente hipóxico, por lo que se aconseja corregir la hipoxia antes de su administración o si no es posible, administrarla vía im. El tratamiento de la intoxicación por agentes colinérgicos se basa en la administración de atropina (para proteger los receptores muscarínicos), oximas (para acelerar la reactivación de la AChE inhibida, en la intoxicación por OP) y benzodiazepinas (para evitar convulsiones). La administración de fosfotriesterasas (PTE, enzimas implicadas en la detoxificación de OP por hidrólisis) ha demostrado ser un tratamiento muy eficaz frente a intoxicaciones por insecticidas OP y agentes nerviosos de guerra OP. 54.- A la probabilidad de que una persona que en realidad padece la enfermedad de interés tenga un resultado positivo en su prueba diagnóstica se le conoce como: a) Sensibilidad b) Especificidad c) Valor predictivo positivo d) Valor predictivo negativo

Page 81: Examen Subespecialidades

La sensibilidad se define como la capacidad que tiene una prueba diagnóstica para clasificar correctamente al enfermo como enfermo o como la probabilidad de tener un resultado positivo dado que se está enfermo.

Ruiz M. A. Epidemiología Clínica, Panamericana, 1ª. Ed. 2004; pág: 116

55.-El diagnóstico mas probable en un paciente femenino de 23 años que presenta en una radiografía con datos de tumoración multiloculada en la zona epifisiaria del extremo distal de fémur izquierdo, es el siguiente: a) Sarcoma osteogénico. b) Tumor de células gigantes. c) Sarcoma de Ewing. d) Mieloma múltiple INTRODUCCIÓN: El tumor de células gigantes es un tumor raro, constituye el 5% de los tumores óseos primarios (1). Ocurre en pacientes entre los 20 y 40 años, siendo raro en los menores de 10 y en mayores de 50 años. Tiene un ligero predominio del sexo femenino y es más frecuente en países orientales que occidentales (2). Habitualmente se localizan en huesos largos, con afectación de la epífisis y de localización excéntrica, pueden propagarse a metáfisis, provocar destrucción cortical y extensión eventual a tejidos blandos y al espacio articular. Los tres sitios más habituales de localización (3), por orden de frecuencia, son: 1º Extremo distal del fémur 2º Extremo proximal de tibia 3º Extremo distal del radio. Clínicamente, el dolor es el signo más frecuente de presentación, tanto si se asocia o no a fractura patológica (4). Localmente la piel puede estar hiperérmica, eritematosa y si el tumor crece puede aparecer circulación colateral con presencia de masa palpable. La clínica sistémica es infrecuente y si el TCG tiene localización raquídea o sacra presenta signos y síntomas neurológicos (5). El diagnóstico del TCG suele realizarse por las manifestaciones clínicas y radiográficas (tabla I), teniendo siempre en cuenta los posibles diagnósticos diferenciales (tabla II). Como pruebas complementarias, la RMN es el método más provechoso para determinar la extensión y el estadiaje, la Gammagrafía es utilizada para detectar TCG multicéntricos que son raros (< 1%) (6) y los parámetros de laboratorio suelen ser normales.

Page 82: Examen Subespecialidades

Tabla I: Características radiológicas del TCG �Lesión osteolítica, excéntrica y epifisaria �Bordes generalmente bien definidos pero no esclerosos �Afectación parcial metafisaria �Expansión a hueso subcondral, adelgazamiento y abombamiento cortical (ruptura) y expansión a tejidos blandos �Mínima o ninguna trabeculación Pals, S.D., and Wilkins, R.M.:Giant cell tumor of bone treated by curettage, cementation, and bone grafting. Orthopedics. 1992. 15: 703-708

56.-Femenino de 22 años, que presenta una tumoración de 2 cm de diámetro en el cuadrante ínfero-externo de la mama izquierda, indolora, de consistencia firme, superficie lisa, forma ovoidea, móvil y bien delimitada del parénquima vecino, sin antecedentes de derrame por el pezón, sin “piel de naranja” ni retracción del pezón, ¿cuál sería su diagnóstico de presunción?:

a) Fibroadenoma. b) Carcinoma. c) Ectasia de los conductos mamarios. d) Quiste solitario.

FIBROADENOMA MAMARIO Tumor benigno más frecuente en las mujeres entre los 20 y 35 años. ETIOLOGIA Existen múltiples teorías siendo la más aceptada la hormonal, generalmente son únicos, solo el 20% son múltiples o bilaterales. De tamaño variable hasta de 10 cm. Ocupa el 13.6% de la patología mamaria benigna. CUADRO CLÍNICO Lesión nodular de consistencia dura, de larga evolución y no dolorosa. Normalmente llegan a los 3 cm. De diámetro. Durante la fase tardía del ciclo menstrual el tumor suele presentar un leve aumento de tamaño. Durante la menopausia presentan regresión hasta la calcificación (signo de palomitas de maíz). DIAGNOSTICO

Page 83: Examen Subespecialidades

Es clínico, se presenta como un tumor bien delimitado, desplazable, no adherido a piel ni a planos profundos, liso o multilobulado en ocasiones. Se localiza frecuentemente en cuadrantes externos. EXAMENES DIAGNOSTICOS: ULTRASONIDO MAMARIO .- Identifica un nódulo sólido, bien delimitado de bordes regulares . TRATAMIENTO: Conservador con vigilancia estrecha dependiendo del tamaño y en caso de ser necesario exéresis del nódulo para estudio histopatológico hospitalgeneral.salud.gob.mx BIBLIOGRAFIA 1. Sánchez BC. Tratado de Enfermedades de la glándula mamaria. Ed. Manual Moderno. Cap. 13- 15. 2.- De Vita V. Cancer of the Breast. In Cancer: Principles and Practice of Oncology: Fifth Ed. Philadelphia: Lippincott-Raven, Chapter 36; pp: 1521-1616. 3.-Consenso Nacional Acerca del Tratamiento de Cáncer de Mama. En Tumores de mama: Diagnóstico y Tratamiento. 2ª Ed. McGraw-Hill Interamericana; pp: 119-126. 4.-Eberlein T. Current management of carcinoma of the breast. Ann Surgery 1994; 220: 121-136. 5. Encyclopedie Medico. Chirurgicale Praxis Medica, Editions Techiques de México, tomo 5, año 2005.

57.- Se presenta a consulta paciente de 25 años de edad refiere que desde hace varios días ha presentado flujo vaginal cuyas características son: blanquecino, grumoso, sin mal olor, muy irritante ¿El agente causal más probable en esta patología es?

a) Gardnerella b) Tricomonas c) Cándida albicans d) Gonococo

Page 84: Examen Subespecialidades

Leucorreas micóticas: Los hongos dan un flujo abundante, blanco, con grumos (aspecto como de quesillo cortado) sin mal olor y que es muy irritante de la piel de la región genital y por lo tanto genera gran ardor vaginal y prurito (picazón). Son muy frecuentes y por lo general son producidas por el hongo Candida albicans.

Tabla I. Características del flujo vaginal según la causa

Cantidad Color Consistencia Olor

Candidiasis Escasa-moderada

Blanco-amarillento Grumosa Indiferente

Tricomonas Aumentada Amarillo-verdoso Espumosa Maloliente

Vaginosis Moderada Blanco-grisáceo

Homogéneo-adherente Maloliente

Speroff Leon and Fritz Marc A. Clinical Gynecologic endocrinology and infertility. 7ª ed. Philadelphia: Lippincott Williams and Wilkins, 2005. p. 25 – 44.

Page 85: Examen Subespecialidades

58.- Se trata de masculino de 39 años que sufre colisión automovilística de frente, poli contundido y con fx expuesta de fémur llama la atención una equimosis periorbitaria por el trauma facial, este signo traduce fractura de: a) Piso posterior b) Piso medio c) Piso anterior d) Macizo facial

En la valoración ocular se deben tener en cuenta las heridas de los tejidos blandos de párpado, córnea y conjuntiva. El signo de mapache (equimosis periorbitaria bilateral) se encuentra frecuentemente asociado a las fracturas de la base anterior del cráneo. (Figura 3).

Figura 3. Equimosis periorbitaria luego de trauma facial contundente.

En las heridas del párpado la localización es de vital importancia y es necesario anotar cuidadosamente el sitio, si se compromete el borde libre o si se lesiona el lugar de paso de la vía lagrimal.

El tono ocular ayudará a definir si hubo herida abierta del ojo.

La presencia de enoftalmo (hundimiento del globo ocular) alertará sobre la posibilidad de una herida abierta del ojo o una fractura de las paredes orbitarias. El hipoftalmos (descenso del ojo en el eje vertical) puede estar relacionado con fractura del piso de la

Page 86: Examen Subespecialidades

órbita o con fractura en el sitio donde se insertan los ligamentos suspensorios del globo ocular (tubérculo de Whitnall, ligamento de Lookwood).

H. Ric Harnsberger. 2004. ISBN 848174753x Translation of PocketRadiologist - Head & Neck: Top 100 Diagnoses. 59.- Un varón de 42 años con antecedente de trastornos convulsivos experimenta una crisis de gran mal. Sus pruebas de laboratorio, tomadas poco después, revelan lo siguiente: Sodio sérico a 140 meq/L; potasio sérico, 4.1 meq/L; cloruro sérico, 97 meq/L; bicarbonato plasmático (HCO3-), 16 meq/L; pH arterial, 7.15 y Paco2, 46 mmHg. ¿Cuál de los siguientes describe mejor la alteración acidobásica?

a) Acidosis respiratoria b) Acidosis metabólica más acidosis respiratoria c) Acidosis respiratoria más alcalosis respiratoria d) Alcalosis mixta

Allen R. M. MMS Medicina Interna. 5ª. Edición. National Medical Series. Mc. Graw Hill. 2006. (capítulo 6, parte II: IV B-D). El paciente tiene acidemia importante, (bajo pH arterial), relacionada con bajo Hco3- sérico; por tanto, debe haber acidosis metabólica. Este trastorno se debe a acumulación de lactato debida a actividad convulsiva. Otro dato es la gran brecha aniónica. Además, el paciente también tiene acidosis respiratoria, como se manifiesta por PaCO2 alta. Con frecuencia, la hiperventilación acompaña a convulsiones de gran mal. 60.- Los anti inflamatorios no esteroideos son fármacos muy utilizados, se absorben fácilmente en el tracto gastrointestinal, una consecuencia idiosincrásica importante que puede ocurrir aún en pacientes previamente sanos, que es reversible si se detecta a tiempo y que debe considerarse con más frecuencia en el caso del ibuprofeno es:

a) Elevación de AST y ALT. b) Cefalea. c) Vitiligo.

Page 87: Examen Subespecialidades

d) Nefrítis intersticial aguda.

Ruiz A, Álvaro-Gracia J. Ma. Manual S.E.R. de las enfermedades reumáticas. Sociedad Española de Reumatología. Editorial Panamericana 2006. Págs. 183 – 188. Los anti inflamatorios no esteroideos poseen vida media y potencia distinta, esas características son expresiones de cada una de las formas moleculares, aquellos de vida media corta actúan menos de seis horas, los de mayor tiempo de acción superan esta cifra. Entre las reacciones adversas más frecuentes se encuentran las gastrointestinales, seguidas por aquellas en hígado, riñón, piel y sistema nervioso central. La nefritis intersticial aguda acompañada de proteinuria masiva corresponde a una reacción idiosincrásica que es más frecuente con el uso de fenoprofeno. 61.-Un varón de 30 años presenta dolor y tumefacción del testículo derecho. Su médico solicita un ultrasonido que revela una masa testicular de 2 x 2.5 cm. Se realizan una exploración inguinal y una orquiectomía. El estudio histopatológico reveló un seminoma puro. Una tomografía computadorizada de tórax, abdomen y pelvis mostró dos ganglios retroperitoneales de 3cm que están aumentados de tamaño. La biometría hemática, la química sanguínea y los marcadores tumorales están todos dentro de los límites normales. ¿Cuál de los siguientes sería la mejor conducta? a) Extirpación quirúrgica de toda la enfermedad. b) Radioterapia. c) Quimioterapia. d) Observación.

Page 88: Examen Subespecialidades

Allen R. M. MMS Medicina Interna. 5ª. Edición. National Medical Series. Mc. Graw Hill. 2006. (capítulo 4IX G 2). El cáncer testicular es el más común en varones adultos jóvenes variedades más frecuentes son seminomas y tumores de células germinales no seminomatosos; ambos sor rabies aun en etapas avanzadas. Los seminomas son muy sensibles a la radioterapia; por tanto, los pacientes enfermedad de etapa II (la limitada a testículo y a ganglios por abajo del diafragma) pueden tratarse con < bajas de radiación. Debido a la toxicidad de la médula ósea producida por la radioterapia mediastínica, y está indicada la radiación profiláctica del mediastino. En este contexto, los ganglios linfáticos retroperitoneales aumentados de tamaño denotan la existencia de enfermedad metastásica y está indicado el tratamiento. 62.- En un paciente del sexo masculino de 35 años de edad con una dermatosis diseminada a codos y rodillas con placas eritemato-escamosas ¿Cuál de los siguientes diagnósticos es el de mayor probabilidad? a) Dermatitis atópica b) Dermatitis seborreica c) Psoriasis d) Dermatitis de contacto La psoriasis es una enfermedad crónica que evoluciona en brotes de causa desconocida, que se caracteriza por placas eritemato escamosas en diferentes partes de la piel. Se presenta por igual en hombres y en mujeres, en todas las edades, predominando en jóvenes y más frecuentemente en personas de piel blanca (parece que la presencia de melanina protege contra la enfermedad. También es ampliamente reconocido que la psoriasis es una enfermedad familiar y hereditaria (diátesis psoriásica). Los sitios de predilección para que aparezcan las lesiones, son los salientes óseos como codos, rodillas, y la piel cabelluda así como la región sacrocoxígea.

Page 89: Examen Subespecialidades

Christophers E, Krueger G G. Psoriasis. En: Fitzpatrick TB, EISEN AZ, Wolff K. Dermatología en Medicina General Buenos Aires: Editorial Panamericana 1988. P. 585-591

Christophers E, Schubert C, Schröder J M. Psoriasis. Dermatología. 1992; 45. 63.- El encontrar elevación de a-fetoproteína en sangre de una madre que cursa con aproximadas 14 semanas de gestación, nos hace pensar en: a) El feto padece un retraso del crecimiento. b) El feto es portador de una trisomía 21. c) El feto tiene una hernia diafragmática. d) El feto tiene un defecto del cierre del tubo neural.

Los Defectos de Cierre del Tubo Neural (DTN) son problemas congénitos severos en el cierre del tejido nervioso, del que se forma el cerebro y el cordón espinal. La Anencefalia es el caso más extremo de los DTN. La bóveda craneana está ausente y sólo queda la porción basal de los huesos frontal parietal y occipital. Su defecto máximo lo constituye la Craneorraquisquisis.

El Encefalocele es la hernia del tejido cerebral a través de un defecto en el cráneo (3), constituyendo el segundo tipo y la Espina Bífida es el DTN menos severo. Consiste en la existencia de uno o más arcos vertebrales defectuosos a través del cual protruye meninges (Meningocele) o meninges, médula espinal y nervios (Mielomeningocele). El grado más leve lo constituye la Espina Bífida Oculta.

Estas afecciones parecen ser de origen multifactorial. La incidencia varía según la localización geográfica, edad materna, características raciales y étnicas, sexo y el nivel socioeconómico, además de la hipertermia y los factores nutricionales. En Estados Unidos de América, la frecuencia es de 1 por cada 1000 recién nacidos vivos aproximadamente.

Page 90: Examen Subespecialidades

Diagnóstico prenatal

1. La elevación de alfafetoproteínas, tanto séricas como en LCR, es un indicador precoz.

2. Ecografía, permite planificar conducta y ofrecer la mejor opción en resguardo de la vida del binomio madre – hijo; en aquellos pacientes que presenten mielomeningocele no sindromático, se contacta con los neurocirujanos previo a interrupción del embarazo que en este caso se planificará por vía alta.

3. Estudio citogenético, en algunos pacientes el DTN corresponde a manifestaciones de alteraciones cromosómicas, como trisomía 18, que deben ser evidenciadas en el período prenatal y evitar acciones innecesarias al momento del nacimiento; es fundamental además para las parejas estar informados del pronóstico real.

Referencias

1. Jones K L , 1997 Smith’s recognizable patterns of human malformation , 5th edition Saunders Philadelpia.

2. Emery’s Mueller RF Genética Médica 10 ° Ed Marban S L 2001. Madrid, España ”Genética y anomalías congénitas” pág. 223 - 243.

3. Plaguicidas en Chile La Guerra Química y sus víctimas Ma. Elena Rosas 1995.

Whittle MJ, Connors JM Prenatal diagnosis in obstetric practice. Blackwell Scientific Publications, Oxford,1989.

Page 91: Examen Subespecialidades

64.-Masculino de 67 años con cáncer de próstata, que presenta hemorragia gingival, hematomas, no hay pulsos distales, acrocianosis y datos de isquemia miocárdica, por lo que usted sospecha: a) Púrpura No Trombocitopénica b) Coagulación Intravascular Diseminada c) Púrpura Trombocitopénica d) Hemofilia La coagulación intravascular diseminada consiste en una activación excesiva de la coagulación, lo que ocasiona trombosis. Como consecuencia el consumo progresivo de los factores de coagulación y plaquetas en las fases finales, produce el fenómeno opuesto, con hemorragias generalizadas. Las causas son: infecciones sobre todo sepsis por gram negativos, problemas obstétricos (abruptio, retención de feto muerto, embolismo de líquido amniótico, aborto séptico, toxemia) neoplasias (leucemias promielocíticas, cáncer de páncreas, próstata, pulmón y esófago), fenómenos autoinmunes y traumas masivos. En la CID aguda debida a complicaciones obstétricas o traumatismo predominan la diátesis hemorrágica, mientras que en la CID crónica se manifiesta inicialmente por complicaciones trombóticas.

1. Cotran R, Kumar V. Robbins Patología estructural y funcional. 6° edición. Mc Graw Hill. Colombia 2003673.

65.-La triada de Beck realizada para el diagnóstico de un tamponade cardiaco en un paciente femenino de 44 años consiste en: a) Hipotensión, PVC elevada, RsCs disminuidos o velados b) Hipotensión, PVC elevada, disminución del tamaño del complejo. c) Hipotensión, PVC elevada, ingurgitación yugular. d) Hipotensión, PVC elevada, taquicardia. El taponamiento cardíaco se define como la compresión del corazón que resulta de la acumulación de líquido en el saco pericárdico y que produce un severo trastorno hemodinámico. El efecto principal de la compresión cardíaca es una alteración en el llenado de las cavidades durante la diástole, lo cual lleva a una disminución del gasto cardíaco y de

Page 92: Examen Subespecialidades

la presión arterial en un espectro variable en el que el paciente severamente comprometido puede estar en shock cardiogénico. •Signos Agudos:-Triada de Beck: Presión arterial baja, elevación de la presión venosa central, Ruidos cardiacos apagados (corazón quieto)-La injurgitaciónyugular puede estar ausente debido a hipovolemia.-Paciente estuporoso, agitado-Extremidades frías y húmedas-Taquipnea y taquicardia.

1. Cheitlin MD, Abbott JA. Urgencias cardíacas. En: Diagnóstico y Tratamiento de Urgencias. Editado por MT. Ho y CE Saunders. Editotial El Manual Moderno. México DF, 1991

2. Hancock EW. Cardiac Tamponade. 3. Med Clin North Am 63:1, 1979 4. Herrera Márquez JH. Pericarditis. En: Cardiología Fundamentos de Medicina. H

Vélez, J Borrero, J Restrepo editores. Corporación para Investigaciones Biológicas. CIB. Medellín, 1981.

66.- Un varón de 71 años presenta gota de inicio agudo. Este problema ha recurrido durante varios años y por lo común se manifiesta como artritis monoarticular aguda que afecta la primera articulación intertarsal proximal metatarsiana. También tiene antecedentes de larga duración de insuficiencia renal crónica con cifras de creatinina sérica de 4 a 6 mg/100 mi en los últimos cinco años e hipertensión de mucho tiempo antes tratada con diversos productos como diuréticos y bloqueadores adrenérgicos. En la exploración física su presión sanguínea es de 170/105 mmHg, su pulso es de 72/min, su frecuencia respiratoria de 15/min y su temperatura de 37°C. En la exploración subsiguiente se encuentra cardiomegalia moderada con galope con tercer ruido cardíaco (S3) y cuarto ruido cardíaco (S4), así como tumefacción e hipersensibilidad de la primera articulación metatarsiana derecha. En los estudios de laboratorio se encuentra lo siguiente: nitrógeno ureico sanguíneo (BUN) de 63 mg/100 mi; creatinina de 5.1 mg/100 mi; sodio sérico de 136 meq/L; potasio sérico de 5.9 meq/L; cloruro sérico de 100 meq/L; C02 de 19 meq/L y ácido úrico de 9.3 mg/100 mi. ¿Cuál de las siguientes es la causa más probable del trastorno de este paciente? a) Nefropatía crónica por plomo b) Exceso de producción primaria de ácido úrico c) Nefritis intersticial crónica relacionada con abuso de analgésicos d) Nefropatía hipertensiva

Page 93: Examen Subespecialidades

Allen R. M. MMS Medicina Interna. 5ª. Edición. National Medical Series. Mc. Graw Hill. 2006. (capítulo 6, parte I: XII B 2 b). La nefropatía crónica por plomo es la causa más probable del trastorno de este paciente; coexiste con afección de la excreción de ácido úrico y también con síndrome clínico de gota. Si bien ésta y la insuficiencia renal crónica de otras causas pueden coexistir, las prueba; actuales sugieren que la toxicidad por plomo es la causa de gran parte de los casos en que concurren ambas enfermedades. El abuso de analgésicos no debe relacionarse con una mayor incidencia de gota, aunque los dato: renales pueden ser semejantes. La nefropatía hipertensiva también puede explicar los datos, aunque no la aparición de gota en este paciente. Podría haber hipertensión renovascular en este enfermo, pero se relaciona con hipertensión más grave cuando hay insuficiencia renal. Además, aquélla coexiste con gota. Sin embargo, se necesitan imágenes del árbol vascular renal para descartar por completo este diagnóstico. 67.- Masculino de 46 años de edad que inicia con dolor intenso en fosa renal izquierda ante la sospecha de litiasis renoureteral se realizan estudios de laboratorio y gabinetem, los Rx demuestran cálculos radio lúcidos, los cuales están relacionados a: a) Calcio b) Estruvita c) Ácido úrico d) Cistina -Bruce E. Jarrell, R. Anthony Carabasi, Nacional Medical Series for Independent Study. Wiliams & Wilkins, 3rd Edition: 451-475. Los cálculos que se pueden presentar en la vía urinaria son formados por calcio, ácido úrico y cistina, los únicos radiolucidos son los de ácido úrico que representan un reto diagnóstico.

68.- Al existir la sospecha de líquido libre en la cavidad peritoneal y después de analizar los hallazgos de manera conjunta de las placas de abdomen y en el ultrasonido abdomino-pélvico, ¿Dónde se deberán de buscar los cambios de densidad de manera inicial?

a) Infradiafragmático b) Fondos de saco de Douglas y de Morrison c) Transcavidad de los epiplones

Page 94: Examen Subespecialidades

d) Correderas parieto-cólicas

Pedrosa C, Casanova R. Diagnóstico por imágen. Mc Graw Hill 2001 p. 209, 210. La diseminación del líquido intraperitoneal está producida por la gravedad y la presión hidrostática secundaria al movimiento diafragmático.la pelvis es la parte más dependiente de la cavidad peritoneal, su capacidad es de 300 cc, a partir de lo cual el líquido asciende por las correderas parietocólicas. El líquido pélvico desplaza las asas intestinales que rellenan los espacios pélvicos, el íleon del lado derecho y el colon sigmoides en el izquierdo, lo que resulta de la parición de una densidad homogénea en al pelvis menor, en contra del patrón poco homogéneo de las asas intestinales que tienen gas y líquido. En la pelvis femenina el líquido se acumula en los recesos laterales de la vejiga y al útero, y en el hombre puede verse en la fosa vesicorrectal y los recesos pélvicos laterales. En el ultrasonido el diagnóstico de pequeñas cantidades de líquido debe hacerse fundamentalmente en el saco de Douglas, en la bolsa de Morrison y en el receso yuxtaesplénico. 69.- En la hiponatremia intensa (inferiores 120 mEq/L se debe de tratar con soluciones salinas hipertónicas mediante el calculo del déficit, reponiendo de la siguiente manera: a) Pasar el déficit de sodio en las primeras 6 hr. b) Pasar el déficit de sodio en las primeras 12 hr. c) Pasar el déficit de sodio, la mitad en las primeras 12-24 hr y se revalora. d) Pasar el déficit de sodio en las primeras 3 hr y después se revalora. Farreas. Medicina Interna. Cap. 222, Decimoquinta edición 2004, Pp. 1844-1847. La hiponatremia con volumen extracelular disminuido se trata con la administración de soluciones salinas isotónicas (al 0.9%). La cantidad necesaria de miliequivalentes de sodio se calcula de acuerdo a la siguiente formula: Na (mEq)= Na x agua total corporal, es decir Na (mEq)= (140- Na actual) x (0.6 x peso en Kg) Donde: Agua total corporal= 60% del peso = 0.6 x peso en kg

Page 95: Examen Subespecialidades

En la practica, suele administrase la mitad de esa cantidad en el transcurso de las primeras 12- 24 h y luego se efectúan reevaluaciones correctoras. 70.- Aunque la mayor parte de los casos de hepatitis vírica aguda cede espontáneamente, pueden sobrevenir complicaciones. ¿Cuál de las siguientes aseveraciones describe mejor las complicaciones de la infección por virus de hepatitis B (HBV)? a) El estado de portador crónico coexiste con mayor riesgo de hepatoma b) Por lo común la hepatitis persistente crónica provoca deterioro progresivo del

funcionamiento hepático y debe tratarse enérgicamente c) La hepatitis activa crónica puede diagnosticarse en dos a cuatro semanas de la

infección aguda con HBV d) La hepatitis activa crónica se caracteriza en la biopsia hepática por infiltrado

linfocítico periportal sin fibrosis o extensión extraportal Allen R. M. MMS Medicina Interna. 5ª. Edición. National Medical Series. Mc. Graw Hill. 2006. (capítulo 5IX A1 f (1 )-(3)). El estado de portador crónico del virus de hepatitis B (HBV) coexiste con mayor riesgo de hepatoma y se encuentra en 0.2% de la población estadounidense. No es posible diagnosticar hepatitis crónica activa hasta por lo menos seis meses después de la infección aguda con HBV. En la hepatitis activa crónica, la inflamación y la fibrosis rebasan el área portal y por tanto, se correlacionan con deterioro profuso de la función hepática, que puede resultar en cirrosis o insuficiencia hepática. Se presenta hepatitis fulminante en alrededor de 1 a 2% de los casos de HBV, hepatitis C (HCV) y hepatitis no A, no B y no C. Esta complicación rara por lo regular se relaciona con baja de las concentraciones de transaminasa conforme es destruido el tejido hepático y disminuye el tamaño del hígado.

71.- Se trata de paciente femenino de la tercera edad que acude por presentar durante las últimas 3 semanas astenia, febrícula vespertina, cefalea global y, durante los últimos días, le han notado confusión intermitente y somnolencia progresiva. A la exploración se observa una temperatura de 38ºC, somnolencia, dudosa rigidez de nuca y paresia de VI par derecho. La placa de tórax muestra un discreto engrosamiento pleural apical derecho. La biometría hemática y química sanguinea se encuentra normal. Se realiza una punción lumbar con los siguiente resultados en LCR: presión de apertura 170mmHg, proteínas 140 mg/dL,

Page 96: Examen Subespecialidades

glucosa 42 mg/dL, 270 células (70% mononucleares), tinción de gram, de Zielhl-Nielsen, así como investigación de antígenos bacterianos son negativos. ¿Que diagnóstico es el más probable?: a) Meningitis tuberculosa. b) Encefalitis herpética. c) Meningitis bacteriana. d) Meningitis vírica.

Meningitis TB

Causas, incidencia y factores de riesgo

La meningitis tuberculosa es un trastorno muy poco común, causado por la diseminación de

la Mycobacterium tuberculosis al cerebro, desde otro sitio en el cuerpo. Generalmente, sus

síntomas se inician gradualmente. Entre los factores de riesgo se pueden mencionar

antecedentes de tuberculosis pulmonar, consumo de alcohol en forma excesiva, SIDA u

otros trastornos que comprometen el sistema inmunitario.

MANIFESTACIONES CLINICAS.

Los fenómenos clínicos evolucionan en varios periodos o etapas cuya duración y contenido semiológico son muy variables, pero que tienen unas características generales que permiten definirlos. En 1948 el British Medical Council definió los tres estadios evolutivos de la enfermedad.(7)

Estadio I: consciente, síntomas inespecíficos, ausencia de afectación neurológica.

Estadio II: cierto grado de afectación de la conciencia, aparición de signos de afectación neurológica.

Estadio III: afectación profunda del estado de conciencia, convulsiones, signos de focalidad neurológica.

La recuperación total es la regla en los casos en estadio I mientras que las secuelas y muerte son muy elevadas en los estadios II y III. En esta última etapa el porcentaje de fallecimientos puede ser superior al 25%.(8,9).

Estadio I

En esta fase, los síntomas son muy inespecíficos, sin que puedan atribuirse a alteración del SNC, salvo por la aparición de alteraciones del carácter, que si bien no son específicas si son bastante constantes.

Page 97: Examen Subespecialidades

Fiebre, generalmente de escasa entidad. Otros síntomas de este periodo son cefalea moderada, apatía, pérdida de la alegría, anorexia, y con mucha frecuencia en el niño dolores abdominales, acompañados o no de vómitos, estos últimos sin características que sugieran un origen central. Los síntomas de esta etapa en el lactante pueden pasar desapercibidos.

Estadio II

Junto a la persistencia de otros signos, entre ellos la fiebre, se ponen de manifiesto los signos de afectación neurológica, coincidiendo con los signos meníngeos.

Al principio predominan la cefalea y los vómitos, estos últimos sobre todo en menores de tres años. Aparece rigidez de nuca, que puede ser muy intensa, y signos de Kernig y Brudzinski.

En más de 1/3 de pacientes con meningitis tuberculosa hay afectación de pares craneales. El sexto par, es el que se afecta con más frecuencia, seguido por los pares III y IV, de forma uni o bilateral. El facial se afecta más raramente, así como otros pares craneales. Puede afectarse el nervio óptico, conduciendo a la atrofia óptica y ceguera.

Las convulsiones son en el niño una manifestación habitual y precoz de la enfermedad.(10)Los signos de hipertensión endocraneal se van acentuando, apreciándose una fontanela tensa en el lactante, aumento del perímetro craneal, edema de papila en el niño mayor, diplopia y visión borrosa.

Loa déficits motores que pueden presentarse a lo largo de la evolución son secundarios a los fenómenos vasculares descritos, que conducen a isquemia e infarto. Puede verse hemiplejía por afectación del territorio de la arteria cerebral media o carótida interna.

Síntomas de afectación medular pueden existir en casos muy evolucionados, con presentación progresiva de paraplejía.

Durante este estadio las alteraciones del sensorio aun son discretas, y su empeoramiento es el que define el paso al siguiente periodo.

Estadio III.

Presidido por la alteración profunda del estado de conciencia, pasando de la apatía e irritabilidad, a la confusión, estupor y coma profundo en los casos avanzados. Los casos terminales se caracterizan por coma profundo, rigidez de descerebración, siendo la muerte inminente.

Page 98: Examen Subespecialidades

Encefalopatía tuberculosa.

En los niños hay un cuadro, denominado encefalopatía tuberculosa por Udani en 1958 (11), que se caracteriza por la presencia de signos de afectación difusa con convulsiones estupor o coma, y signos moderados de afectación meníngea. El coma sobreviene precozmente, y la muerte sobreviene entre 1 y 2 meses después del comienzo, aun con terapéutica específica. Dominan el cuadro los fenómenos de hipertensión intracraneal.

Anatomopatológicamente el cuadro está constituido por severo edema de la sustancia blanca, sin presencia de hemorragias. Microscópicamente hay una pérdida difusa de mielina en la sustancia blanca, atribuyéndose la causa a fenómenos de hipersensibilidad frente a la tuberculoproteinas

Es una infección de las meninges, las membranas que recubren el cerebro y la médula

espinal, causada por la bacteria Mycobacterium tuberculosis que produce la tuberculosis.

Signos y exámenes

Para cualquier paciente con meningitis, es importante llevar a cabo una punción lumbar:

• Tinción del líquido cefalorraquídeo (LCR) positiva para Mycobacterium. • LCR con alto nivel de proteínas, bajo nivel de glucosa y aumento en el número de

linfocitos.

• Cultivo de LCR que muestra la proliferación de Mycobacterium tuberculosis

• Reacción en cadena de la polimerasa (PCR) que muestra LCR positivo para M. tuberculosis.

• Prueba cutánea positiva para tuberculosis.

• Biopsia cerebral o meníngea que muestra la presencia de M. tuberculosis.

BIBLIOGRAFIA

Page 99: Examen Subespecialidades

1. Sudre P,Dam G,Kochi A. La tuberculose aujourd'hui dans le monde.WHO Bull 1992;70:297-308.

2. Dolin PJ,Raviglione MC,Kochi A. Global tuberculosis incidence and mortality during 1990-2000.WHO Bulletin 1994;72:213-220.

3. March Ayuela P. Trend in tuberculous meningitis in Barcelona in children aged 0-4 years:correlation with the annual risk of tuberculous infection. Tubercle and Lung Disease 1994;75:423-428.

4. Barry R Bloom.Tuberculosis: Pathogenesis, Protection, and Control. Tuberculosis. American Society for Microbiology. Washington DC 2005.

5. Anggard E. Nitric oxide: Mediator, murderer ,and medicine. Lancet. 1994;343: 1199-206.

6. Jinkins JR,Gupta R,Hyun Chang K,Rodriguez-Carvajal J.MR imaging of cental nervous system tuberculosis. Radiol Clin North Am 1995;33:771-786.

7. Medical Research Council.Streptomycin in Tuberculosis Trials Committee: Streptomycin treatment of tuberculous meningitis.Lancet. 1948;1:582-596.

72.- En la utilización de medios de contraste iodados por vía intravascular. ¿Cuál es la característica que produce reacciones adversas?

a) La ionicidad

b) La hiperosmolaridad

d) La elevación de azoados

e) La radiopacidad

Sánchez AC, Casanova R. Pedrosa. Diagnóstico por imágen. Tratado de Radiología Clínica. McGraw Hill. México 1997. p 80. Las reacciones adversas al contraste pueden ser de dos tipos: yodosincráticas o quimiotóxicas. La mayoría son de origen yodosincrático, es decir anafilácticas y ocurren independientemente de la dosis administrada y la concentración del producto. Las de origen quimiotáxico se deben a mecanismos fisicoquímicos específicos sobre los órganos que prefunden, directamente dependientes de la dosis y la concentración inyectada. Los factores potencialmente responsables del efecto incluyen: la

Page 100: Examen Subespecialidades

hiperosmolaridad, concentración de cationes y capacidad para ligarse al calcio. La administración IV de contraste iónico de alta osmolaridad produce 5 a 125 de las reacciones agudas. 73.- Su tutor en el curso de infectología de pregrado le pide que revise a un paciente al cual se le ha diagnosticado endocarditis en la válvula mitral. Como parte de su exploración física usted revisa el fondo de ojo en busca de: a) Nódulos de Osler b) Mancha de Roth c) Lesiones de Janeway d) Hemorragias en astilla

Las manchas de Roth son hemorragias retinales (retinianas) con centro pálido o de color blanco compuestas y rodeadas de fibrina coagulada o cúmulos de hematíes. Estas manchas pueden observarse en el fondo de ojo mediante el uso de un oftalmoscopio.1

Las manchas de Roth generalmente son causadas vasculitis mediada por inmunocomplejos debido a la endocarditis bacteriana, también pueden ser observadas en leucemia, diabetes, anemia perniciosa, isquemia y raramente en retinopatía por VIH.

Fowler VG Jr, Scheld WM, Bayer AS. Endocarditis and Intravascular Infections. In: Mandell GL, Bennett JE, Dolin R, eds. Principles and Practice of Infectious Diseases. 7th ed. Philadelphia, Pa: Elsevier Churchill Livingstone; 2009; chapt 77.

Page 101: Examen Subespecialidades

74.- Masculino de 36 años acude al servicio por presentar fiebre, escalofríos, cefalea, mialgias y esplenomegalia, como antecedentes refiere que realizó un viaje a Chiapas, se realiza examen de “gota gruesa” observando formas asexuadas con tinción de Wright. El laboratorio reporta, leucopenia con presencia de formas jóvenes, trombocitopenia y elevación de transaminasas. El tratamiento de elección en este paciente es: a) Cloroquina b) Sulfato de quinina y doxiciclina c) Mefloquina d) Cloroquina y primaquina

Medidas específicas de prevención y control

9.1 Tratamiento médico del paludismo. Se emplea la combinación de dos medicamentos, la cloroquina que elimina las formas sanguíneas del P. vivax y del P. falciparum, excepto los gametocitos de este último y la primaquina que elimina los hipnozoítos del P. vivax y los gametocitos del P. falciparum. Se ministrarán diferentes esquemas de acuerdo al objetivo que se persiga.

9.1.1 Recomendaciones Generales. La cloroquina y primaquina son medicamentos que en personas sensibles causan:

9.1.1.1 Irritación gástrica. Siempre deberán administrarse cuando la persona a tratar haya ingerido algún alimento y con abundantes líquidos. Si el paciente vomita 30 minutos después de recibirlo, debe recibir una segunda dosis completa. Si vomita entre 30 y 60 minutos después de una dosis, se le debe dar media dosis adicional. En lo sucesivo pueden fraccionarse las dosis diarias en varias tomas al día.

9.1.1.2 Primaquina. No debe ministrarse a menores de seis meses de edad, mujeres embarazadas y madres en periodo de lactancia por sus efectos adversos en hígado y la interacción con hemoglobinas fetales; en estos casos se emplea solamente cloroquina a dosis supresivas cada 21 días y una vez que las condiciones antes mencionadas desaparezcan, se ministrará tratamiento completo.

9.1.1.3 Control médico. Cualquier tratamiento antipalúdico y en particular para menores de seis meses de edad, mujeres embarazadas, madres en periodo de lactancia, enfermos hepáticos y otros padecimientos intercurrentes graves, queda siempre sujeta al criterio y vigilancia médica.

Page 102: Examen Subespecialidades

9.1.2 Tratamiento supresivo. Aquel que se administra sólo para eliminar el ciclo eritrocítico; utilizar la cloroquina como medicamento de elección en caso de infecciones de paludismo vivax, y cloroquina más primaquina en regiones con paludismo falciparum.

9.1.2.1 Indicación. A toda persona sospechosa de padecer paludismo se le ministra al momento de tomar la muestra hemática, una dosis única de cloroquina a dosis de 10 mg por kg de peso o según edad y en áreas especiales (de falciparum) se agrega primaquina a dosis de 0.25 mg por kg de peso (a los menores de 6 meses y embarazadas no se les ministra este último).

9.1.2.2 Tratamiento supresivo en áreas con P. vivax. Se ministra sólo cloroquina, indicado para áreas de baja transmisión o hipoendémicas, en las mesoendémicas e hiperendémicas el tratamiento de primera elección es la dosis única, posterior a la toma de muestra hemática.

Tabla 5 Tratamiento supresivo en áreas con transmisión de P. vivax.

GRUPO DE EDAD No. COMPRIMIDOS CLOROQUINA 150 mg

< 6 meses 1/4

6 meses a 1 año 1/2

2 a 5 años 1

6 a 12 años 2

13 años y más,

con menos de 60 kg de peso 3

13 años y más, con más de 60 kg de peso 4

9.1.2.3 Tratamiento supresivo en áreas P. falciparum. Al momento de tomar la muestra de sangre, administrar cloroquina y primaquina a las dosis mencionadas en el apartado 9.1.2.1 Indicación. La dosificación se refiere en la "Tabla 6".

Tabla 6 Tratamiento supresivo en áreas P. falciparum

Page 103: Examen Subespecialidades

GRUPO DE EDAD No. COMP.

CLOROQUINA DE 150 mg

No. COMP. PRIMAQUINA

DE 5 mg

No. COMP. PRIMAQUINA

DE 15 mg

< 6 meses 1/4 0 0

6 meses a 1 año 1/2 1/2 0

2 a 5 años 1 1 0

6 a 12 años 2 2 0

13 años y más, con menos de 60 kg de peso 3 0 1

13 años y más, con más de 60 kg de peso 4 0 1

9.1.3 Tratamiento de cura radical (TCR). La cura radical se logra aplicando diferentes esquemas terapéuticos y su efectividad varía de acuerdo a lo referido en la "Tabla 7".

Tabla 7 Tratamiento de cura radical

Esquema Objetivo/Indicación Eficacia estimada

Dosis única (TDU) con cloroquina y

primaquina) con periodicidad mensual

Eliminar fuentes de infección para mosquitos. En distribución masiva o a los casos conocidos de

los últimos tres años y sus convivientes, disminuye cargas parasitarias en la población.

Útil en caso de brotes o en áreas especiales de transmisión persistente y focalizada,

actualmente en áreas de alta transmisión se indica como primera elección al momento de

tomar la gota gruesa y sustituye al supresivo-presuntivo

< 50%

TCR a 5 días (cloroquina y primaquina)

Tratamiento en casos detectados en áreas hipo, meso e hiperendémicas. Puede administrarse en forma masiva o a casos y convivientes pero no

como medida única de control

80%

TCR a 14 días (cloroquina y primaquina)

Tratamiento alternativo para los casos de áreas hipoendémicas y cuando los dos esquemas de

TCR anteriores no han sido suficientes 90%

Dosis única (TDU con Su mayor utilidad es en casos repetidores. 95%

Page 104: Examen Subespecialidades

cloroquina y

primaquina) 3x3x3 o seguimiento por 3

años que equivale a 18 dosis mensuales cada 8 a 21 días durante 8

tomas

Elimina fuentes de infección para mosquitos

Los cuatro esquemas empleados para cura radical mencionados en la "Tabla 7" son:

9.1.3.1 Tratamiento en dosis única (TDU). Se aplica para disminuir, rápida y drásticamente la densidad de parásitos circulantes en un área específica.

Referencia: NORMA Oficial Mexicana NOM-032-SSA2-2002, Para la vigilancia epidemiológica, prevención y control de enfermedades transmitidas por vector. 75.- Masculino que es ingresado al servicio de urgencias por haber sufrido quemaduras en el cuerpo, durante las primeras 24 hrs. después de la lesión la resucitación de líquidos en éste tipo de pacientes incluye: a) Infusión inicial de 2 ml de coloides / % quemado / Kg. Peso. b) Infusión de cristaloides a razón de mantener un gasto urinario de 1 ml / hr. /

Kg. peso en adultos. c) Infusión de cristaloides a razón de obtener 45 ml de orina / hr. en paciente de 28 Kg. d) Uso de diuréticos para tratar oliguria ( 15 ml de orina / 2 hrs.) en ausencia de

hemocromatógenos urinarios. REANIMACION HIDRICA

– Fórmula de Parkland • Se utiliza en las primeras 24 hr. • Con cristaloides

4 ML / KG / % SCT Quemada

• La mitad del déficit total en las primeras 8 hrs. • La segunda mitad en las 16 hrs restantes.

Page 105: Examen Subespecialidades

• Calcular volumen circulante 60cc x kg.

– Mantener adecuada PVC ( 8 - 10 ) – Mantener TAM > 60 mmHg – Gasto urinario 0.5 - 1.0ml / kg / hr – Evitar administración excesiva de líquidos

• Uresis (2ml / kg / hr ) • Na sérico c 8 hrs

VIGILAR HIPOTERMIA 76.- Un niño de 6 años con hemofilia tipo A (actividad del factor VIII del 10%) se da un golpe en el codo y sufre, como consecuencia, inflamación e hipersensibilidad en el lugar del traumatismo. ¿Cuál es el tratamiento apropiado? a) Infusión de desmopresina b) Infusión de plasma fresco congelado c) Infusión de concentrado de factor VIII d) Inyección de vitamina K Paul H. Dwaorkin. Nacional Medical Series for independent study. Pediatrics. Mc Graw Hill. 4th ed. 2000. Capitulo 15. VI D 2 a (2). Con un nivel de factor VIII del 10%, el paciente tiene una hemofilia A leve. En ausencia de hemorragia potencialmente letal o de cirugía mayor, estos pacientes pueden tratarse con desmopresina (dDAVP), que elevara el nivel de factor VIII dentro del ámbito terapéutico sin exponer al paciente al riesgo de infección vírica transmitida por la sangre. Aunque el factor VIII recombinante, también está libre de contaminantes virales, son mucho más caros que la dDAVP. La vitamina K no tendría utilidad alguna en esta situación, ya que el factor VIII no es vitamina k- dependiente.

Page 106: Examen Subespecialidades

77.- Hipotonía muscular, hiperlaxitud articular, braquicefalia, fisuras palpebrales oblicuas, puente nasal deprimido, macroglosia relativa, clinodactilia del 5° dedo, persistencia de conducto arterioso son características clínicas de: a) Trisomía 21 b) Trisomía 13 c) Trisomía 18 d) Trisomía 8

CONCEPTO El Síndrome de Down (SD), también llamado trisomía 21, es la causa más frecuente de retraso mental identificable de origen genético. Se trata de una anomalía cromosómica que tiene una incidencia de 1 de cada 800 nacidos, y que aumenta con la edad materna. Es la cromosomopatía más frecuente y mejor conocida. CLÍNICA Los niños con SD se caracterizan por presentar una gran hipotonía e hiperlaxitud ligamentosa. Fenotípicamente presentan unos rasgos muy característicos. CABEZA y CUELLO: leve microcefalia cuello es corto. CARA: los ojos son “almendrados”, y si el iris es azul suele observarse una pigmentación moteada, son las manchas de B r u s h f i e l d. Las hendiduras palpebrales siguen una dirección oblicua hacia arriba y afuera y presentan un pliegue de piel que cubre el ángulo interno y la carúncula del ojo (epicanto). La nariz es pequeña con la raíz nasal aplanada. La boca también es pequeña y la protusión lingual característica. Las orejas son pequeñas con un helix muy plegado y habitualmente con ausencia del lóbulo. El conducto auditivo puede ser muy estrecho. MANOS Y PIES: manos pequeñas y cuadradas con metacarpianos y falanges cortas (braquidactilia) y clinodactilia por hipoplasia de la falange media del 5º dedo. Puede observarse un surco palmar único. En el pie existe una hendidura entre el primer y segundo dedo con un aumento de la distancia entre los mismos (signo de la sandalia).

Page 107: Examen Subespecialidades

GENITALES: el tamaño del pene es algo pequeño y el volumen testicular es menor que el de los niños de su edad, una criptorquídia es relativamente frecuente en estos individuos. PIEL y FANERAS: la piel es redundante en la región cervical sobretodo en el período fetál y neonatal. Puede observarse livedo reticularis (cutis marmorata) de predominio en extremidades inferiores. Con el tiempo la piel se vuelve seca e hiperqueratósica. El retraso mental es constante en mayor o menor grado. BIBLIOGRAFÍA: 1. Cooley WC, Graham JM. Down syndrome: an update and review for the primary pediatrician. Clin Pediatr. 1991;30:233-253. 2. American Academy of Pediatrics. Committee on Genetics Health Guidelines for Children with Down syndrome.. Pediatrics 1994;93:855-859. 3. Cronk C, Crocker Ac, Pueschel SM, et al. Growth charts for children with Down syndrome: 1 month to 18 years of age. Pediatrics 1988;81:102-10. 78.- Se trata de recién nacido pretermino de 30 semanas de gestación, peso de 1200 gr. Cursó con enfermedad de membranas hialinas por lo que se manejo con una dosis de surfactante y manejo con ventilación convencional por 3 días. Posteriormente inició con alimentación enteral por sonda orogástrica con leche humana, sin tolerancia a la misma (residuo gástrico y distensión abdominal), existe la sospecha sospecha diagnóstica de enterocolitis necrotizante Los signos y síntomas esperados son:

a) Sangre macroscópica en heces, cólico intestinal, diarrea b) Sangre microscópica en heces, ictericia, neumatosis intestinal c) Distensión abdominal, doble burbuja gástrica d) Distensión abdominal, radiografía normal La enterocolitis necrotizante (ECN) es una enfermedad grave que afecta a recién nacidos, en especial prematuros, con una incidencia y morbimortalidad elevados. Constituye la urgencia gastrointestinal más frecuente en las UCI neonatales. Se presenta como un síndrome gastrointestinal y sistémico que comprende síntomas variados y variables, como distensión e hipersensibilidad abdominal, sangre en heces, intolerancia a la alimentación, apnea, letargia, y en casos avanzados acidosis, sepsis, CID y shock.

Page 108: Examen Subespecialidades

El síndrome clínico ha sido clasificado en estadios por Bell y col. (1978) y modificado por Walsh y Klegman (1986) para incluir hallazgos sistémicos, intestinales y radiológicos.

• A. Estadio I : sospecha de enterocolitis necrotizante

• Los hallazgos sistémicos son inespecíficos.

• Los hallazgos intestinales incluye el residuo gástrico y heces guayaco –positivas.

• Los hallazgos radiológicos son normales e inespecíficos.

• B. Estadio II A: enterocolitis necrotizante leve

• Los hallazgos sistémicos son similares al estadio I.

• Los hallazgos intestinales incluyen distensión abdominal prominente con hipersensibilidad a la palpación o sin ella, ruidos hidroaéreos ausentes, sangre macroscópica en materia fecal.

• Los hallazgos radiológicos, íleo con asas dilatadas con áreas focales de neumatosis intestinal.

• C. Estadio II B: enterocolitis necrotizante moderada

• Los hallazgos sistémicos incluyen acidosis leve y trombocitopenia.

• Los hallazgos intestinales incluyen edema de la pared abdominal e hipersensibilidad a la palpación con una masa palpable o sin ella.

• Los hallazgos radiológicos incluyen neumatosis extensa y ascitis temprana.

• Puede haber gas en la vena porta intrahepática.

• D. Estadio IIIA: enterocolitis necrotizante avanzada:

• Los hallazgos sistémicos incluyen acidosis respiratoria y metabólica, ventilación asistida por apnea, hipotensión arterial, oliguria, neutropenia y coagulación intravascular diseminada.

• Los hallazgos intestinales incluyen edema que disemina, eritema e induración del abdomen.

• Los hallazgos radiológicos incluyen ascitis prominente y asa centinela persistente sin perforación.

• E. Estadio IIIB: enterocolitis necrotizante avanzada:

• Los hallazgos sistémicos revelan signos vitales e índices de laboratorio en deterioro, síndrome de shock y desequilibrio electrolítico.

• Los hallazgos intestinales y radiológicos muestran evidencias de perforación.

Page 109: Examen Subespecialidades

Hallazgos radiológicos en la radiografía de abdomen

• Distensión abdominal generalizada

• Íleo paralítico

• Neumatosis intestinal quistoide (patognomónico)

La neumatosis intestinal quistoide se la puede visualizar de varias maneras, cúmulos de gas lineales, curvilíneos, esponjosos y espumosos.

Este último debe distinguirse de materia fecal o meconio mezclado con aire.

Cualquier lactante con sospecha de enterocolitis necrotizante en el que se encuentren radiográficamente colecciones de aire lineales, curvilíneas esponjosas o espumosas debe considerarse que tiene neumatosis intestinal quistoide hasta que se demuestre lo contrario.

La neumatosis intestinal quistoide suele verse con mayor frecuencia en el colon, pero puede ocurrir desde estomago hasta recto.

1. Hartmann G. E., Drugas G. T., Shochat S. J. Post-necrotizing enterocolitis strictures presenting with sepsis of perforation: risk of clinical observation. J. Pediatr. Surg. 1988; 23: 562-6.

2. Kosloske A. M., Burstein J., Bartow S. A. Intestinal obstruction due to colonic stricture following neonatal necrotizing enterocolitis. Ann Surg. 1980 Aug;192 (2): 202-7.

3. Schwartz M. Z., Hayden C. K., Richardson C. J., Tyson K. R., Lobe T. E. A prospective evaluation of intestinal stenosis following necrotizing enterocolitis. J. Pediatr. Surg. 1982 Dec; 17 (6): 764-70.

4. Bell M. J., Ternberg J. L., Askin F. B. Intestinal stricture in necroting enterocolitis. J. Pediatr. Surg. 1976; 11: 319-27.

5. Pokorny W. J., Harr V. L., McGill, C. W., et al; Intestinal stenosis resulting from necrotizing enterocolitis. Am J. Surg 1981 42: 721-724.

6. Schimpl G., Hollwarth M. E., Fotter R., Becker H. Late intestinal strictures following successful treatment of necrotizing enterocolitis. Acta Paediatr. Suppl. 1994; 396: 80-3.

7. Bütter A., Flageole H., Laberge J. M. The Changing face of Surgical Indication for Necrotizing Enterocolitis J. Pediatr. Surg. 2002; 37: 469- 499.

Gobet R. , Sacher P. , Schwobel M. G. Surgical procedures in colonic strictures after necrotizing enterocolitis. Acta Paediatr. Suppl. 1994;396:77-9.

Page 110: Examen Subespecialidades

79. - Which one of the following conditions results in prologation of the partial thromboplastin time (PTT), but not the prothrombin time (PT)? a) Varicela hemorrhage as a result of cirrosis b) Menorrhagia resulting from von Willebrand’s disease c) Therapy with broad-spectrum antibiotics d) Therapy with coumarin for phlebitis

Enf. Von Willebrand

Manifestaciones

Epistaxis 60 %

Hemorragia transvaginal 50 %

Equimosis 40 %

Gingivorragias 35 %

Hematomas 5 %

Hemartrosis 3 %

EvW

Pruebas de escrutinio

T. Hemorragia Prolongado

C. Plaquetaria Normales, excepto 2B

TTPa Normal o prolongado

TP Normal

Gpo AB0 25 % bajo en “0”

Page 111: Examen Subespecialidades

• 1.-Lee GR, Foerster J, Lukens J, Paraskevas F, Greer JP and Rodgers GM.

Wintrobe’s clinical haematology; 10th Edition, Lippincott Williams & Wilkins, United States of America, 1999.

• 2.-Williams WJ. Manual Williams de hematología 5a Edición McGraw-Hill Interamericana, México, 1997. • 3.-Beutler E, Lichtman MA, Coller BS, Kipps T. Hematology. 5th International

Edition. United Stated of America, 1995. • 4.-Ruiz Argüelles GJ. Fundamentos de hematología; 2ª. Edición, Editorial Médica

Panamericana, México, 1998. 80.- Paciente obesa, con fractura de cadera que será intervenida quirúrgicamente para reemplazo total de cadera se considera: a) Paciente de riesgo bajo para trombosis venosa profunda. b) Paciente de riesgo alto para trombosis venosa profunda. c) Paciente con riesgo de trombosis arterial. d) Paciente con riesgo muy bajo de TEP.

La fisiopatología de la TVP se resume mediante la tríada de Virchow: Estasis sanguíneo, daño endotelial e hipercoagulabilidad. Estas 3 circunstancias aisladamente o en asociación, intervienen en el desarrollo de un trombo. Los factores de riesgo enumerados a continuación aumentan la probabilidad de desarrollar trombosis mediante uno o más de los mecanismos de la tríada (Kahn S, 1998):

1. Cirugía mayor, especialmente la cirugía ortopédica, pero también los pacientes sometidos a cirugía abdominal, neurocirugía.

2. Neoplasias malignas, el riesgo aumenta en los pacientes que reciben quimioterapia activa.

Page 112: Examen Subespecialidades

3. Infarto Agudo de Miocardio. 4. Síndrome Nefrótico. 5. Ictus isquémico, preferentemente en el miembro hemipléjico. 6. Inmovilización prolongada. A mayor tiempo de inmovilización mayor riesgo, aunque

se ha documentado una incidencia de TVP del 13% en pacientes encamados durante 8 días.

7. TVP o Embolismos Pulmonares (EP) previos. El riesgo se debe probablemente a la persistencia de obstrucción al flujo y/o daño en las válvulas venosas tras la TVP anterior.

8. Embarazo y postparto. 9. Anticoncepción oral y Terapia Hormonal Sustitutiva (THS). 10. Alteraciones congénitas o adquiridas que produzcan hipercoagulabilidad:

o Deficiencia congénita de Proteína C, Proteína S y Antitrombina III. o Resistencia a la Proteína C activada: Esta alteración se encuentra en el 5%

de la población general y en el 20-40% de los enfermos con TVP. o Hiperhomocisteinemia o Disfibrinogenemia o Presencia de Anticuerpos Antifosfolípido

Estos trastornos se asocian a TVP recurrentes, o en localizaciones atípicas o de aparición a edades tempranas (antes de los 45 años). Bibliografía:

1. Ebell MH. Evaluation of the patient with suspected deep vein thrombosis. J Fam Pract. 2001 Feb;50(2):167-71 [PubMed] [Texto completo]

2. Gabriel Botella F, Labiós Gómez M, Brasó Aznar JV. Trombosis venosa profunda:

presente y futuro. Med Clin 2000; 114: 584-596. [PubMed]

3. Gorman WP, Davis KR, Donnelly R. ABC of arterial and venous disease. Swollen limb-1: General assessment and deep vein thrombosis. BMJ 2000; 320: 1453-1456. [PubMed] [Texto completo]

4. Green L, Fay W, Harrison V, Kleaveland M, Wahl R, Wakefield T, Weg J, Williams

D. Venous thromboembolism (VTE) [Internet]. Ann Arbor (MI): University of Michigan Health System 2004 [acceso 18/3/2007] Disponible en: http://cme.med.umich.edu/iCME/vte04/ [NGC]